Download as pdf or txt
Download as pdf or txt
You are on page 1of 204

= Item _1 of 30

Question Id: 18804


■ t7' Mark <:::
Previous
C>
Next
~~
Full Screen
®
Tutorial
il
Lab Values

Notes
....I
Calculator
,
Reverse Color
~
Text Zoom
©
Settings

A 45-year-old woman comes to the office due to progressive visual difficulty with night driving and reading road
signs and fine print. The patient has systemic lupus erythematosus, which is treated with hydroxychloroquine and
prednisone. She avoids sun exposure due to photosensitivity and has vitamin D deficiency for which she takes a
vitamin D supplement. Examination shows opacities on the posterior side of the lenses in both eyes. Optic discs
are normal. Ocular movements are full. The remainder of the examination shows no abnormalities. Which of the
following is the most significant contributing factor for the eye findings in this patient?

0 A . Hydroxychloroquine

0 8 . Lack of sun exposure

0 C. Prednisone

0 D. Systemic lupus erythematous

0 E. Vitamin D deficiency

Submit

Block Time Elapsed: 00:00:31 ~ https://t.me/USMLEWorldStep1 II;! rvo Cl @) 0


My Notebook Flashcards Feedback Suspend End Block
= Item _1 of 30
Question Id: 18804
■ t7' Mark <:::
Previous
C>
Next
~~
Full Screen
®
Tutorial
il
Lab Values

Notes
....I
Calculator
,
Reverse Color
~
Text Zoom
©
Settings

A 45-year-old woman comes to the office due to progressive visual difficulty with night driving and reading road
signs and fine print. The patient has systemic lupus erythematosus, which is treated with hydroxychloroquine and
prednisone. She avoids sun exposure due to photosensitivity and has vitamin D deficiency for which she takes a
vitamin D supplement. Examination shows opacities on the posterior side of the lenses in both eyes. Optic discs
are normal. Ocular movements are full. The remainder of the examination shows no abnormalities. Which of the
following is the most significant contributing factor for the eye findings in this patient?

0 A . Hydroxychloroquine (35%)

X ® 8 . Lack of sun exposure (2%>)

✓ Q C. Prednisone (47%)

0 D. Systemic lupus erythematous (12%)

0 E. Vitamin D deficiency (1%)

Incorrect
Correct answer I 1,1 47% fi"\ 44 secs
I.!!.!.!. Answered correctly \.:.J Time Spent
C

Explanation

Cataracts

Pathogenesis • Oxidative damage & opacification of the lens

• Age >60
• Diabetes mellitus
- -
Block Time Elapsed: 00:00:44 ~ https://t.me/USMLEWorldStep1 II;! rvo Cl @) 0
My Notebook Flashcards Feedback Suspend End Block
= Item _1 of 30
Question Id: 18804
■ t7' Mark <:::
Previous
C>
Next
~~
Full Screen
®
Tutorial
il
Lab Values
~-
Notes
....I
Calculator
,
Reverse Color
~
Text Zoom
©
Settings

Cataracts

Pathogenesis • Oxidative damage & opacification of the lens

• Age >60
• Diabetes mellitus
Risk factors • Chronic sunlight exposure
• Tobacco use
• lmmunosuppression (eg, HIV, corticosteroids)

• Painless, progressive, bilateral vision loss


• Difficulty with nighttime driving
Clinical features
• Loss of red reflex
• Opacified lens

Treatment • Artificial lens implantation

This patient has bilateral opacities in the ocular lenses, findings consistent with cataracts. Cataracts can cause
difficulty with reading and fine visual tasks; patients often see excessive glare at night and halos around bright
lights due to light scattering in the lens. Examination findings include white or gray cloudiness of the lens,
decreased visualization of retinal detail, and loss of the red reflex.

Cataracts are generally related to chronic photooxidative injury; the prevalence increases with age, and most
patients with cataracts first develop symptoms at age >60. However, cataracts can develop earlier in individuals,
such as this patient, with exposure to systemic or ophthalmic glucocorticoids (eg, prednisone). This may be due
to glucocorticoid-induced transcription of genes in the lens epithelial cell that may alter lens homeostasis or
increase the susceptibility to oxidative injury. Other causes of premature cataract formation include diabetes
mellitus, ocular trauma, and external radiation exposure.
- -

Block Time Elapsed: 00:00:44 ~ https://t.me/USMLEWorldStep1 II;! rvo Cl @) 0


My Notebook Flashcards Feedback Suspend End Block
= Item _1 of 30 ■ t7' Mark <::: C> ~~ ® il ~• , ~ ©
- Question Id: 18804
-. - - - - .- - . Previous Next Full Screen Tutorial Lab Values Notes Calculator Reverse Color Text Zoom Settings

This patient has bilateral opacities in the ocular lenses, findings consistent with cataracts. Cataracts can cause
difficulty with reading and fine visual tasks; patients often see excessive glare at night and halos around bright
lights due to light scattering in the lens. Examination findings include white or gray cloudiness of the lens,
decreased visualization of retinal detail, and loss of the red reflex.

Cataracts are generally related to chronic photooxidative injury; the prevalence increases with age, and most
patients with cataracts first develop symptoms at age >60. However, cataracts can develop earlier in individuals,
such as this patient, with exposure to systemic or ophthalmic glucocorticoids (eg, prednisone). This may be due
to glucocorticoid-induced transcription of genes in the lens epithelial cell that may alter lens homeostasis or
increase the susceptibility to oxidative injury. Other causes of premature cataract formation include diabetes
mell itus, ocular trauma, and external radiation exposure.

(Choices A and D) The increased risk of premature cataract formation in patients with systemic lupus
erythematosus (SLE) is due primarily to chronic/frequent glucocorticoid exposure, not to the disease itself.
Patients with SLE are often treated with hydroxychloroquine; long-term use can cause retinopathy, which presents
with decreased central visual acuity, photopsia (flashing lights), and central macular degeneration.

(Choice B) Excessive, not limited , lifetime sun exposure is associated with an increased risk of cataracts.

(Choice E) Common manifestations of vitamin D deficiency incl ude hypocalcemia, bone pain, and accelerated
bone loss. Cataracts are not a major consequence of vitamin D deficiency.

Educational objective:
Cataracts are related primarily to chronic photooxidative injury. Most patients with cataracts first develop
symptoms at age >60 but exposure to systemic or ophthalmic glucocorticoids can cause cataracts at an early age.
Other causes of premature cataract formation include diabetes mell itus, ocular trauma, and external radiation
exposure.

;;:--- . -

Block Time Elapsed: 00:00:44 ~ https://t.me/USMLEWorldStep1 II;! rvo Cl @) 0


My Notebook Flashcards Feedback Suspend End Block
Normal eyes & white reflex

Thi
diffi
ligh
dee
Normal eyes
Cat Red reflexes & corneal light reflexes are equal.

pati
sue
tog
mer
mell

(Ch
eryt
Absent reflex
Pati White reflex on abnormal eye can result from opacities of the lens
(eg , cataract) or tumor (eg, retinoblastoma).
with
euwoo1d

(Ch ©_ Zoom In 0_ Zoom Out C Reset Cf;, New I lf-J Existi ng (!;I My Notebook
(Ch
bone loss. Cataracts are not a major consequence of vitamin D deficiency.

Block Time Elapsed: 00:00:44 ~ https://t.me/USMLEWorldStep1 11;1 rvo O @) 0


My Notebook Flashcards Feedback Suspend End Block
= ltem~of30
Question Id: 863
■ t7' Mark <:::
Previous
C>
Next
~~
Full Screen
®
Tutorial
il
Lab Values

Notes
....I
Calculator
,
Reverse Color
~
Text Zoom
©
Settings

A 3-month-old boy is brought to the office by his mother for a routine checkup. He is found to have a right-sided
white pupillary reflex. His father was diagnosed with a retinal neoplasm during childhood and had his left eye
enucleated . This child is at the greatest risk of developing which of the following neoplasms?

0 A . Acute lymphoblastic leukemia

0 8 . Neuroblastoma

0 C. Ewing sarcoma

0 D. Osteosarcoma

0 E. Medulloblastoma

Submit

Block Time Elapsed: 00:00:45 ~ https://t.me/USMLEWorldStep1 II;! rvo Cl @) 0


My Notebook Flashcards Feedback Suspend End Block
= ltem~of30
Question Id: 863
■ t7' Mark <
Previous
C>
Next
~~
Full Screen
®
Tutorial
il
Lab Values

Notes
....I
Calculator
,
Reverse Color
~
Text Zoom
©
Settings

A 3-month-old boy is brought to the office by his mother for a routine checkup. He is found to have a right-sided
white pupillary reflex. His father was diagnosed with a retinal neoplasm during childhood and had his left eye
enucleated . This child is at the greatest risk of developing which of the following neoplasms?

0 A . Acute lymphoblastic leukemia (2%)

X ® 8 . Neuroblastoma (22%)

0 C. Ewing sarcoma (5%)

✓ Q D. Osteosarcoma (62%>)

0 E. Medulloblastoma (6%>)

Incorrect
Correct answer I 1,1 62% fi"\ 05 secs
~ Answered correctly \.::,;Time Spent
D

Explanation

Knudson's 2-hit hypothesis

Both copies of the gene must be knocked out to promote malignancy.

Sporadic cancer:
2 acquired mutations

https://t.me/USMLEWorldStep1
C
D
Knudson's 2-hit hypothesis

Both copies of the gene must be knocked out to promote malignancy.


E
Sporadic cancer:
2 acquired mutations

Tumor

Hereditary cancer:
1 inherited & 1 acquired mutation

Tumor
iC)UWorld

©_ Zoom In 0_ Zoom Out C Reset Cf;, New I lf-J Existi ng (!;I My Notebook

https://t.me/USMLEWorldStep1
:= ltem~of30
Question Id: 863
■ t7' Mark <:::
Previous
C>
Next
~~
Full Screen
®
Tutorial
il
Lab Values
~•
Notes Calculator
,
Reverse Color
~
Text Zoom
©
Settings

Retinoblastoma is the most common ocular tumor of childhood. It usually presents with a white pupillary reflex
(leukocoria) in children <5 years old. Approximately 60% of retinoblastomas are sporadic (usually unilateral), while
40% are familial (often, but not always, bilateral).

Familial retinoblastoma occurs due to a germline mutation that affects the Rb tumor suppressor gene on
chromosome 13; affected children carry one copy of the mutation in all their cells. However, there are two copies
of the Rb gene (one allele from each parent), and malignancy does not occur while at least one copy of the gene is
still functional. A somatic second mutation acquired early in life is the "second hit" that leads to retinoblastoma, as
cells with two inactivated Rb genes proliferate uncontrollably and give rise to malignancy.

Children with familial retinoblastoma often acquire other "second hit" mutations later in life, leading to additional
tumors, usually sarcomas such as osteosarcoma. In contrast, patients with sporadic retinoblastoma are not at
risk for other malignancies, as the chance of two acquired somatic mutations occuring multiple times in different
cell lineages is very low.

(Choice A) Patients with Down syndrome, ataxia-telangiectasia, and neurofibromatosis type 1 have an increased
risk of development of acute lymphoblastic leukemia, which is not associated with familial retinoblastoma.

(Choice B) Neuroblastoma is a common childhood cancer that arises from sympathetic nervous tissue, most
commonly in the adrenals. Most cases are sporadic.

(Choice C) Ewing sarcoma is an undifferentiated neoplasm of mesenchymal origin that most commonly occurs in
the long and flat bones of children and adolescents. It occurs sporadically.

(Choice E) Medulloblastoma is the most common malignant brain tumor of childhood, typically arising in the
cerebellum.

Educational objective:
Familial retinoblastoma occurs as a result of mutations of each of the two Rb genes ("two hits"). These patients
have an increased risk of secondary tumors, especially osteosarcomas, later in life.

Block Time Elapsed: 00:00:49 ~ https://t.me/USMLEWorldStep1 II;! rvo Cl @) 0


My Notebook Flashcards Feedback Suspend End Block
= ltem~of30
Question Id: 863
■ t7' Mark <:::
Previous
C>
Next
~~
Full Screen
®
Tutorial
il
Lab Values

Notes
....I
Calculator
,
Reverse Color
~
Text Zoom
©
Settings

Retinoblastoma is the most common ocular tumor of childhood. It usually presents with a white pupillary reflex
(leu ' Exhibit Display Bl x
40°/c

Fa

Retinoblastoma white reflex


still
cell

Chil
tum
risk
cell

(Ch
risk

(Ch
co
Retinoblastoma of the left
(Ch eye with white reflex
C)UWorld
the

(Ch

Ed
©_ Zoom In 0_ Zoom Out C Reset Cf;, New I lf-J Existi ng (!;I My Notebook

Fa
have an increased risk of secondary tumors, especially osteosarcomas, later in life.

Block Time Elapsed: 00:00:49 ~ https://t.me/USMLEWorldStep1 11;1 rvo O @) 0


My Notebook Flashcards Feedback Suspend End Block
= ltem_3of30
Question Id: 106294
■ t7' Mark <:::
Previous
C>
Next
~~
Full Screen
®
Tutorial
il
Lab Values

Notes
....I
Calculator
,
Reverse Color
~
Text Zoom
©
Settings

A 32-year-old woman comes to the office to establish medical care. The patient reports no symptoms. Medical
history is significant for migraine headaches for which she takes acetaminophen and naproxen as needed. Vital
signs are within normal limits. Physical examination is unremarkable except for anisocoria . Pupillary examination
findings in normal, bright, and dim light are shown in the image below:

Right Left

Room light

Bright light

Dim light

This patient's pupillary abnormality is likely due to a defect in which of the following pathways?

0 A . Optic nerve of the left eye

Block Time Elapsed: 00:00:53 ~ https://t.me/USMLEWorldStep1 II;! rvo Cl @) 0


My Notebook Flashcards Feedback Suspend End Block
Bright light

Dim light

This patient's pupillary abnormality is likely due to a defect in which of the following pathways?

0 A . Optic nerve of the left eye

0 B. Optic nerve of the right eye

0 C. Parasympathetic innervation of the left eye

0 D. Parasympathetic innervation of the right eye

0 E. Sympathetic innervation of the left eye

O F. Sympathetic innervation of the right eye

Submit

Block Time Elapsed: 00:00:57 ~ https://t.me/USMLEWorldStep1 II;! rvo Cl @) 0


My Notebook Flashcards Feedback Suspend End Block
Bright light

Dim light

This patient's pupillary abnormality is likely due to a defect in which of the following pathways?

0 A . Optic nerve of the left eye ( 1°/o)

0 B. Optic nerve of the right eye (11 %)

X ® C. Parasympathetic innervation of the left eye (4%)

✓0 D. Parasympathetic innervation of the right eye (69%)

0 E. Sympathetic innervation of the left eye (2%)

O F. Sympathetic innervation of the right eye ( 11%)

Incorrect
Correct answer I 1,1 69% f'i\ 12 secs
~ Answered correctly \..::.J Time Spent
D

Block Time Elapsed: 00:01:01 ~ https://t.me/USMLEWorldStep1 II;! rvo Cl @) 0


My Notebook Flashcards Feedback Suspend End Block
= ltem_3of30
Question Id: 106294
.- - -
■ t7' Mark <
Previous
C>
Next
~~
Full Screen
®
Tutorial
il
Lab Values

Notes
....I
Calculator
,
Reverse Color
~
Text Zoom
©
Settings

Exhibit Display Bl x

Pupillary asymmetry
In room light

Sympathetic defect (right eye) Parasympathetic defect (left eye)


Increases in dim light Increases in bright light

Decreases in bright light Decreases in dim light

Thi
sym
exa

• ©uworld

©_ Zoom In 0_ Zoom Out C Reset Cf;, New I lf-J Existi ng (!;I My Notebook

• In dim light, the sympathetic pathway becomes dominant. With sympathetic defects, pupillary dilation

Block Time Elapsed: 00:01:01 ~ https://t.me/USMLEWorldStep1 11;1 rvo O @) 0


My Notebook Flashcards Feedback Suspend End Block
=
-
ltem_3of30
Question Id: 106294
■ t7' Mark <:::
Previous
C>
Next
~~
Full Screen
®
Tutorial
il
Lab Values
~•
Notes Calculator
,
Reverse Color
~
Text Zoom
©
Settings

This patient has incidental anisocoria (asymmetric pupils). Pupil size is governed by the balance between ocular
sympathetic (dilation) and parasympathetic (constriction) efferents to the iris. These responses are tested by
examining the pupils in bright and dim light:

• In bright light, the parasympathetic pathway is dominant. With parasympathetic defects, pupillary
constriction occurs in the "good" eye but not the "bad," causing the difference between pupils to widen (ie,
anisocoria increases) in bright light. The larger pupil is abnormal (Choice C).

• In dim light, the sympathetic pathway becomes dominant. With sympathetic defects, pupillary dilation
occurs in the "good" eye but not the "bad," causing anisocoria to increase in the dark. The smaller pupil is
abnormal.

This patient's anisocoria increases in bright light, consistent w ith a parasympathetic defect causing her right pupil
to not constrict properly in response to light; the unaffected left pupil constricts normally. In the dark, both pupils
dilate because sympathetic input is intact bilaterally; a small degree of anisocoria will still be present because basal
parasympathetic activity is reduced in the left eye.

(Choices A and B) The visual pathway carries afferent signals from the retina to the visual cortex. Lesions
preceding the optic chiasm (eg, retina, optic nerve [CN II]) cause monocular vision loss; however, pupils remain
symmetric (no anisocoria) with ambient lighti ng because the efferent sympathetic or parasympathetic tracts are not
involved . Instead, prechiasmatic visual pathway lesions cause a relative afferent pupillary defect: When light is
shone in the affected eye, no signal is transmitted , so pupillary constriction is equally impaired bilaterally.

(Choices E and F) With sympathetic defects (eg, damage to cervical sympathetic chain or ganglia), the affected
pupil cannot properly dilate, so anisocoria increases in dim light. In response to bright light, both pupils constrict
because parasympathetic innervation is intact bilaterally, and the anisocoria is reduced.

Educational objective:
Anisocoria (asymmetric pupils) is caused by a lesion in the ocular parasympathetic (constriction) or sympathetic

Block Time Elapsed: 00:01:01 ~ https://t.me/USMLEWorldStep1 II;! rvo Cl @) 0


My Notebook Flashcards Feedback Suspend End Block
:= ltem_3of30
Question Id: 106294
■ t7' Mark <:::
Previous
C>
Next
~~
Full Screen
®
Tutorial
il
Lab Values
~-
Notes
....I
Calculator
,
Reverse Color
~
Text Zoom
©
Settings

examining the pupils in bright and dim light:

• In bright light, the parasympathetic pathway is dominant. With parasympathetic defects, pupillary
constriction occurs in the "good" eye but not the "bad," causing the difference between pupils to widen (ie,
anisocoria increases) in bright light. The larger pupil is abnormal (Choice C).

• In dim light, the sympathetic pathway becomes dominant. With sympathetic defects, pupillary dilation
occurs in the "good" eye but not the "bad," causing anisocoria to increase in the dark. The smaller pupil is
abnormal.

This patient's anisocoria increases in bright light, consistent with a parasympathetic defect causing her right pupil
to not constrict properly in response to light; the unaffected left pupil constricts normally. In the dark, both pupils
dilate because sympathetic input is intact bilaterally; a small degree of anisocoria will still be present because basal
parasympathetic activity is reduced in the left eye.

(Choices A and B) The visual pathway carries afferent signals from the retina to the visual cortex. Lesions
preceding the optic chiasm (eg, retina, optic nerve [CN II]) cause monocular vision loss; however, pupils remain
symmetric (no anisocoria) with ambient lighti ng because the efferent sympathetic or parasympathetic tracts are not
involved . Instead, prechiasmatic visual pathway lesions cause a relative afferent pupillary defect: When light is
shone in the affected eye, no signal is transmitted , so pupillary constriction is equally impaired bilaterally.

(Choices E and F) With sympathetic defects (eg, damage to cervical sympathetic chain or ganglia), the affected
pupil cannot properly dilate, so anisocoria increases in dim light. In response to bright light, both pupils constrict
because parasympathetic innervation is intact bilaterally, and the anisocoria is reduced.

Educational objective:
Anisocoria (asymmetric pupils) is caused by a lesion in the ocular parasympathetic (constriction) or sympathetic
(dilation) pathways. In this patient, anisocoria increases in bright light, indicating that the larger right pupil is unable
to constrict due to a defect in the right parasympathetic pathway.

Block Time Elapsed: 00:01:01 ~ https://t.me/USMLEWorldStep1 II;! rvo Cl @) 0


My Notebook Flashcards Feedback Suspend End Block
= ltem_3of30
Question Id: 106294
■ t7' Mark <
Previous
C>
Next
~~
Full Screen
®
Tutorial
il
Lab Values

Notes
....I
Calculator
,
Reverse Color
~
Text Zoom
©
Settings

Sympathetic defect (right eye) Parasympathetic defect (left eye)

Exhibit Display Bl x

Autonomic innervation of the eye

Afferent retinal Optic nerve


nerve fibers
Long ciliary
nerves

Thi
Edinger-Westphal
sym nucleus

exa


Oculomotor
nerve (CN 111)
nerves
Ciliary I Trigeminal
• gangli~ nerve (CN V)
■ Sensory
■ Parasympathetic
■ Sympathetic Carotid plexus

Thi
ton
dila
©_ Zoom In 0_ Zoom Out C Reset Cf;, New I lf-J Existi ng (!;I My Notebook

• •
Block Time Elapsed: 00:01:01 ~ https://t.me/USMLEWorldStep1 11;1 rvo O @) 0
My Notebook Flashcards Feedback Suspend End Block
Pupillary light reflex

Thi Ciliary ganglion -


sym
exa

Oculomotor
nerve (CN Ill)

Thi
ton
dila
/
Lateral geniculate
nucleus Edinger-Westphal
par nudeus
Pretectal nucleus
0UWo<1d

(Ch ©. Zoom In 0. Zoom Out a Reset q;, New I lf-J Existi ng I!;! My Notebook

involved . Instead, prechiasmatic visual pathway lesions cause a relative afferent pupillary defect: When light is
Block Time Elapsed: 00:01:01 ~ https://t.me/USMLEWorldStep1 11;1 rvo O @) 0
My Notebook Flashcards Feedback Suspend End Block
= ltem~of30
Question Id: 799
■ t7' Mark <:::
Previous
C>
Next
~~
Full Screen
®
Tutorial
il
Lab Values

Notes
....I
Calculator
,
Reverse Color
~
Text Zoom
©
Settings

A 4-week-old boy is being evaluated in the neonatal intensive care unit. The patient was born at 29 weeks
gestation . Immediately before delivery, the mother received a dose of terbutaline in an attempt to stop the
contractions and a dose of antenatal corticosteroids to help improve fetal lung maturity. Shortly after birth, the
patient developed severe respiratory distress, requiring surfactant administration and supplemental oxygen. He
was weaned off oxygen therapy after a week. The patient also received routine prophylactic medications, including
erythromycin ophthalmic ointment and intramuscular vitamin K, at birth. Ophthalmic examination today shows
abnormal retinal vascularization that extends into the vitreous. The retinal findings in this patient are most likely
related to which of the following?

0 A . Maternal corticosteroid therapy

0 8 . Maternal terbutaline treatment

0 C. Neonatal ophthalmic antibiotic treatment

0 D. Neonatal oxygen supplementation

O E. Neonatal surfactant therapy

Submit

Block Time Elapsed: 00:01:02 ~ II;! rvo Cl @ 0


My Notebook Flashcards Feedback Suspend End Block

"' https://t.me/USMLEWorldStep1 Show all X


= ltem~of30
Question Id: 799
■ t7' Mark <:::
Previous
C>
Next
~~
Full Screen
®
Tutorial
il
Lab Values

Notes
....I
Calculator
,
Reverse Color
~
Text Zoom
©
Settings

A 4-week-old boy is being evaluated in the neonatal intensive care unit. The patient was born at 29 weeks
gestation . Immediately before delivery, the mother received a dose of terbutaline in an attempt to stop the
contractions and a dose of antenatal corticosteroids to help improve fetal lung maturity. Shortly after birth, the
patient developed severe respiratory distress, requiring surfactant administration and supplemental oxygen. He
was weaned off oxygen therapy after a week. The patient also received routine prophylactic medications, including
erythromycin ophthalmic ointment and intramuscular vitamin K, at birth. Ophthalmic examination today shows
abnormal retinal vascularization that extends into the vitreous. The retinal findings in this patient are most likely
related to which of the following?

0 A . Maternal corticosteroid therapy ( 10%>)

X ® 8 . Maternal terbutaline treatment (7°/o)

0 C. Neonatal ophthalmic antibiotic treatment (3%)

✓0 D. Neonatal oxygen supplementation (74%)

O E. Neonatal surfactant therapy (2%)

Incorrect
Correct answer I 1,1 74% fi"\ 04 secs 2023
~ Answered correctly \..::.;Time Spent Version
D

Explanation

Retinopathy of prematurity

Block Time Elapsed: 00:01:05 ~ II;! rvo Cl @ 0


My Notebook Flashcards Feedback Suspend End Block

"' https://t.me/USMLEWorldStep1 Show all X


Retinopathy of prematurity

C
D
Term birth

E
In utero
Low 0 2 tension --+ t VEGF
Retinal vascularization compl~

Relative hyperox,a .... I VEGF Relative hypoxia .... t VEGF

Normal vessel growth

T metabolic
I Preterm birth

demand

Abnormal retinal vascularizatlon


Cessation of vessel growth
(retlnopathy of prematurity)
<C)~d

©_ Zoom In 0_ Zoom Out C Reset Cf;, New I lf-J Existi ng (!;I My Notebook

https://t.me/USMLEWorldStep1 Show all X


:= ltem~of30
Question Id: 799
■ t7' Mark <:::
Previous
C>
Next
~~
Full Screen
®
Tutorial
il
Lab Values
~•
Notes Calculator
,
Reverse Color
~
Text Zoom
©
Settings

This infant born at 29 weeks gestation has abnormal retinal vascularization extending into the vitreous, findings
diagnostic of retinopathy of prematurity (ROP). This condition, exclusive to premature neonates, is a leading
cause of severe visual impairment and blindness.

In utero, the relatively low oxygen tension normally causes upregulation of proangiogenic factors, such as vascular
endothelial growth factor (VEGF); these growth factors stimulate normal vessel growth. Premature birth disrupts
this process as follows:

• The relatively hyperoxic environment (even in room air) after delivery leads to downregulation of VEGF
and slowing or arrest of retinal vascular growth.

• Once the retina begins to mature (over subsequent weeks), increased metabolic demand causes relative
retinal hypoxia, which again stimulates VEGF. This often results in aberrant new vessel formation
(neovascularization), which can lead to retinal scarring and/or detachment if severe.

Although prematurity alone can lead to ROP, the risk is increased in premature neonates with significantly elevated
arterial oxygen tension because hyperoxia exacerbates the initial VEGF suppression and increases risk of
subsequent aberrant neovascularization. In this case, the patient had neonatal respiratory distress syndrome and
received supplemental oxygen, which likely contributed to disrupted retinal angiogenesis.

(Choice A) Corticosteroids are often administered to women in preterm labor to induce surfactant production by
the fetal lungs and decrease the risk or severity of neonatal respiratory distress syndrome. This therapy is not
associated w ith neonatal retinal neovascularization.

(Choice B) Terbutaline is a beta agonist used to delay preterm labor and delivery by suppressing uterine
contractions. Fetal risks include tachycardia and hypoglycemia (due to maternal hyperglycemia causing fetal
hyperinsulinism), not ROP.

Block Time Elapsed: 00:01:05 ~ II;! rvo Cl @ 0


My Notebook Flashcards Feedback Suspend End Block

"' https://t.me/USMLEWorldStep1 Show all X


:= ltem~of30 ■ t7' Mark <::: C> ~~ ® il ~• , ~ ©
Question Id: 799
. • -- . --- .- ' - - . Previous
..
Next
. • - -- Full Screen Tutorial Lab Values Notes Calculator Reverse Color Text Zoom Settings

(neovascularization), which can lead to retinal scarring and/or detachment if severe.

Although prematurity alone can lead to ROP, the risk is increased in premature neonates with significantly elevated
arterial oxygen tension because hyperoxia exacerbates the initial VEGF suppression and increases risk of
subsequent aberrant neovascularization. In this case, the patient had neonatal respiratory distress syndrome and
received supplemental oxygen, which likely contributed to disrupted retinal angiogenesis.

(Choice A) Corticosteroids are often administered to women in preterm labor to induce surfactant production by
the fetal lungs and decrease the risk or severity of neonatal respiratory distress syndrome. This therapy is not
associated w ith neonatal retinal neovascularization.

(Choice B) Terbutaline is a beta agonist used to delay preterm labor and delivery by suppressing uterine
contractions. Fetal risks include tachycardia and hypoglycemia (due to maternal hyperglycemia causing fetal
hyperinsulinism), not ROP.

(Choice C) Neonatal ophthalmic antibiotic treatment (ie, erythromycin) prevents gonococcal conjunctivitis and can
cause chemical conjunctivitis in rare cases; however, retinal changes would not occur.

(Choice E) Exogenous surfactant decreases alveolar surface tension and alveolar collapse (ie, atelectasis),
improving ventilation/perfusion mismatch. This therapy allows for supplementation of oxygen at decreased
concentrations, which helps prevent ROP.

Educational objective:
The pathogenesis of retinopathy of prematurity involves an initial downregulation of proangiogenic factors (eg,
VEGF) due to hyperoxia following delivery. Subsequently, increased metabolic demand causes relative retinal
hypoxia, which stimulates a pathological increase in VEGF leading to aberrant vessel formation. Neonatal oxygen
supplementation increases the risk in premature infants.

Block Time Elapsed: 00:01:05 ~ II;! rvo Cl @ 0


My Notebook Flashcards Feedback Suspend End Block

"' https://t.me/USMLEWorldStep1 Show all X


= ltem_5of30
Question Id: 12083
■ t7' Mark <
Previous
C>
Next
~~
Full Screen
®
Tutorial
il
Lab Values

Notes
....I
Calculator
,
Reverse Color
~
Text Zoom
©
Settings

A 68-year-old woman comes to the emergency department due to sudden-onset double vision that started
immediately after she woke up. The patient notices that her vision worsens when looking to the right. She has no
headache, weakness, or numbness. The patient has a prolonged history of hypertension , hyperlipidemia, and type
2 diabetes mellitus. Blood pressure is 160/90 mm Hg and pulse is 86/min and regular. Neurologic examination,
including visual acuity, is normal with the exception of the ocular findings shown in the image below.

Rightward
gaze

Neutral

Leftward

https://t.me/USMLEWorldStep1 Show all X


=
-
ltem5of30
Question Id: 12083
■ t'.7
\ Mark
<
Previous
C>
Next
~~
Full Saeen
®
Tutorial
il
Lab Values
~-
Notes
~
Calculator
,
Reverse Color
~
Text Zoom
©
Settings

A6 ••
1m Exhibit Display ========-- - - - - - -- Bl x
hea
2 di Rightward
incl gaze

Neutral

Leftward
gaze

Convergence

© UWorld

©_ Zoom In 0, Zoom Out C Reset Cf;, New I lf-J Existi ng (!;I My Notebook

https://t.me/USMLEWorldStep1 Show all X


Convergence

© UWorld
A lesion involving which of the following structures is most likely causing the observed ocular findings in this
patient?

0 A . Abducens nerve

0 8 . Lateral geniculate nucleus

0 C. Medial longitudinal fasciculus

0 D. Occipital lobe

0 E. Oculomotor nerve

0 F. Superior colliculus

Submit

Block Time Elapsed: 00:01:16 ~ II;! rvo Cl @ 0


My Notebook Flashcards Feedback Suspend End Block

"' https://t.me/USMLEWorldStep1 Show all X


Convergence

© UWorld
A lesion involving which of the following structures is most likely causing the observed ocular findings in this
patient?

0 A . Abducens nerve (10%)

X ® 8 . Lateral geniculate nucleus (5%)

✓0 C. Medial longitudinal fasciculus (67%)

0 D. Occipital lobe (0%)

0 E. Oculomotor nerve (14%)

0 F. Superior colliculus (1%>)

Incorrect
Correct answer I 1,1 67% ti\ 14 secs
l!!!!. Answered correctly \::.J Time Spent
C

Block Time Elapsed: 00:01:19 ~ II;! rvo Cl @ 0


My Notebook Flashcards Feedback Suspend End Block

"' https://t.me/USMLEWorldStep1 Show all X


=
-
ltem5of30
Question Id: 12083
■ t'.7
\ Mark
<
Previous
C>
Next
~~
Full Saeen
®
Tutorial
il
Lab Values

Notes
~
Calculator
,
Reverse Color
~
Text Zoom
©
Settings

E Exhibit Display Bl x
Gaze palsy with left internuclear ophthalmoplegia

(D Palie<il attempts
right gaze

@ Left MLF lesK>n


prevents activation
of left oculomotor
nucleus

'
'
,
Right , Left

Functioning
lateral rectus

-
(full abduction)

MLF . medSI IOngilUOinal la&dculut.

©_ Zoom In 0_ Zoom Out C Reset Cf;, New I lf-J Existi ng (!;I My Notebook

https://t.me/USMLEWorldStep1 Show all X


:= ltem_5of30
Question Id: 12083
■ t7' Mark <:::
Previous
C>
Next
~~
Full Screen
®
Tutorial
il
Lab Values
~-
Notes
....I
Calculator
,
Reverse Color
~
Text Zoom
©
Settings

This patient has internuclear ophthalmoplegia, a disorder of impaired horizontal gaze caused by damage to the
medial longitudinal fasciculus (MLF). The frontal eye field initiates horizontal gaze through the contralateral
abducens (CN VI) nucleus, which communicates with the ipsilateral oculomotor (CN Ill ) nucleus by way of the
MLF. This allows for simultaneous activation of the lateral rectus muscle in the contralateral eye (abduction) and
the medial rectus muscle in the ipsilateral eye (adduction), resulting in coordinated conjugate horizontal gaze.

Unilateral MLF lesions typically occur in older patients with lacunar strokes in the dorsal pons or midbrain
(pontine artery distribution) and result in impaired adduction of the ipsilateral eye during horizontal gaze,
causing diplopia. Abduction in the contralateral eye is preserved (but often with gaze-evoked nystagmus) . The
pathways for convergence and the pupillary light reflex bypass the MLF lesion , so these reflexes are generally
preserved .

{Choice A) The abducens nerves innervate the lateral rectus muscles, which abduct the eyes. A lesion involving
an abducens nerve would result in impaired abduction of the ipsilateral eye due to lateral rectus muscle palsy.

(Choice B) The lateral geniculate nucleus is located in the thalamus and relays visual information to the ipsilateral
primary visual cortex. Damage to this structure would result in a contralateral homonymous hemianopia.

(Choice D) Unilateral infarction of the primary visual cortex in the occipital lobe (eg , posterior cerebral artery
occlusion) typically results in contralateral homonymous hemianopia with macular sparing as the macula receives
collateral circulation from the middle cerebral artery.

(Choice E) Oculomotor nerve palsy (eg, ipsilateral mydriasis, ptosis, "down and out" deviation of the eye) may
occur with microvascular nerve ischemia (eg, diabetes mellitus) or nerve compression (eg , posterior
communicating artery aneurysm).

(Choice F) The superior colliculus controls vertical gaze. Damage to the superior colliculus in the dorsal midbrain

Block Time Elapsed: 00:01:19 ~ II;! rvo Cl @ 0


My Notebook Flashcards Feedback Suspend End Block

"' https://t.me/USMLEWorldStep1 Show all X


=
-
ltem_5of30
Question Id: 12083
■ t7' Mark <:::
Previous
C>
Next
~~
Full Screen
®
Tutorial
il
Lab Values
~•
Notes Calculator
,
Reverse Color
~
Text Zoom
©
Settings

Unilateral MLF lesions typically occur in older patients with lacunar strokes in the dorsal pons or midbrain
(pontine artery distribution) and result in impaired adduction of the ipsilateral eye during horizontal gaze,
causing diplopia. Abduction in the contralateral eye is preserved (but often with gaze-evoked nystagmus) . The
pathways for convergence and the pupillary light reflex bypass the MLF lesion , so these reflexes are generally
preserved .

(Choice A) The abducens nerves innervate the lateral rectus muscles, which abduct the eyes. A lesion involving
an abducens nerve would result in impaired abduction of the ipsilateral eye due to lateral rectus muscle palsy.

(Choice B) The lateral geniculate nucleus is located in the thalamus and relays visual information to the ipsilateral
primary visual cortex. Damage to this structure would result in a contralateral homonymous hemianopia.

{Choice D) Unilateral infarction of the primary visual cortex in the occipital lobe (eg , posterior cerebral artery
occlusion) typically results in contralateral homonymous hemianopia with macular sparing as the macula receives
collateral circulation from the middle cerebral artery.

(Choice E) Oculomotor nerve palsy (eg, ipsilateral mydriasis, ptosis, "down and out" deviation of the eye) may
occur with microvascular nerve ischemia (eg, diabetes mellitus) or nerve compression (eg , posterior
communicating artery aneurysm).

(Choice F) The superior colliculus controls vertical gaze. Damage to the superior colliculus in the dorsal midbrain
causes Parinaud syndrome (eg, upward gaze palsy, absent pupillary light reflex, impaired convergence).

Educational objective:
Internuclear ophthalmoplegia is a disorder of impaired horizontal gaze caused by a lesion in the medial longitudinal
fasciculus (MLF). The affected eye (ipsilateral to the lesion) is unable to adduct while the contralateral eye is able
to abduct (often with associated nystagmus). Convergence and the pupillary light reflex are preserved because
these pathways bypass the MLF.

Block Time Elapsed: 00:01:19 ~ II;! rvo Cl @ 0


My Notebook Flashcards Feedback Suspend End Block

"' https://t.me/USMLEWorldStep1 Show all X


Conjugate horizontal gaze

© Patient attempts Left frontal


right gaze eye field

Thi
...- --+- -. -- Left oculomotor
nudeus

the Right abducens nudetls - ----::.-=o.!

(D Righi PPRF ------,,-.- , - -


~.,...-~
Uni activates right
abducens & 1en
ocutomotor
(po

pat
pre R.ight

(Ch
Lateral rectus Medial r
an (abduC1ion) (adduction

Mlf • meCll8' 10rlgituell'la1 l&WCl;IUS.. PPRF • perameOian POl'llln& ret!Ct.181' tormatlon.


CUWorld

pn ©_ Zoom In 0_ Zoom Out C Reset Cf;, New I lf-J Existi ng (!;I My Notebook

occlusion) typically results in contralateral homonymous hemianopia with macular sparing as the macula receives
Block Time Elapsed: 00:01:19 ~ II;! rvo Cl @ 0
My Notebook Flashcards Feedback Suspend End Block

"' !!I 51549.mp4


"' https://t.me/USMLEWorldStep1 Show all X
Right abducens nerve palsy

Rightward gaze

the

Uni
(po

Neutral
pat
pre

(Ch
an

(Ch Leftward gaze


pn

(Ch
©UWOl1d
occl
©_ Zoom In 0_ Zoom Out C Reset Cf;, New I lf-J Existi ng (!;I My Notebook
coll

e .1 I II e. • I'"'&'"' 1 - u .. -1 I .1-'"' U'"' I l'"'a'"' IHI'"' II '"'I II '"'II

Block Time Elapsed: 00:01:19 ~ II;! rvo Cl @ 0


My Notebook Flashcards Feedback Suspend End Block

"' !!I 51549.mp4


"' https://t.me/USMLEWorldStep1 Show all X
= ltem_6of30
Question Id: 1646
■ t7' Mark <:::
Previous
C>
Next
~~
Full Screen
®
Tutorial
il
Lab Values

Notes
....I
Calculator
,
Reverse Color
~
Text Zoom
©
Settings

A 43-year-old man comes to the emergency department due to painless, progressive visual impairment. The
patient was diagnosed with sexually acquired HIV infection 4 years ago. He was previously started on
antiretroviral therapy but has been noncompliant with his prescribed medications. His most recent CD4 count was
37 cells/mm3 . Physical examination shows no skin rash or corneal lesions. Funduscopy reveals the findings seen
in the image below.

Which of the following medications is the best initial therapy for this patient?

- ' •
Block Time Elapsed: 00:01:22 ~ II;! rvo Cl @ 0
My Notebook Flashcards Feedback Suspend End Block

"' !!I 51549.mp4


"' https://t.me/USMLEWorldStep1 Show all X
= ltem_6of30
Question Id: 1646
■ t7' Mark <
Previous
C>
Next
~~
Full Saeen
®
Tutorial
il
Lab Values

Notes
....I
Calculator
,
Reverse Color
~
Text Zoom
©
Settings

37

©_ Zoom In 0_ Zoom Out C Reset Cf;, New I lf-J Existi ng (!;I My Notebook
Whi

- '

Block Time Elapsed: 00:01:25 ~ II;!


My Notebook
rvo
Flashcards
Cl
Feedback
@
Suspend
0
End Block

"' !!I 51549.mp4


"' https://t.me/USMLEWorldStep1 Show all X
Which of the following medications is the best initial therapy for this patient?

0 A . Acyclovir

0 8 . Amphotericin B

0 C. Clarithromycin

0 D. Flucytosine

0 E. Ganciclovir

0 F. Penicillin G

Submit

Block Time Elapsed: 00:01:30 ~ II;! rvo Cl @ 0


My Notebook Flashcards Feedback Suspend End Block

"' !!I 51549.mp4


"' https://t.me/USMLEWorldStep1 Show all X
Which of the following medications is the best initial therapy for this patient?

0 A . Acyclovir (10%)

X ® 8 . Amphotericin B (6%>)

0 C. Clarithromycin (3%)

0 D. Flucytosine (3°/o)

✓ Q E. Ganciclovir (74%)

0 F. Penicillin G (1 %)

Incorrect
Correct answer I 1,1 74% f'i\ 15 secs
I.!!.!!. Answered correctly \..::,; Time Spent
E

Block Time Elapsed: 00:01:34 ~ II;! rvo Cl @ 0


My Notebook Flashcards Feedback Suspend End Block

"' !!I 51549.mp4


"' https://t.me/USMLEWorldStep1 Show all X
Cytomegalovirus (CMV) retinitis

C
E ow-w 1te.
areas of r

•often following retinal vessels in a "brush-fire· pattern C UWorld

©_ Zoom In 0_ Zoom Out C Reset Cf;, New I lf-J Existi ng (!;I My Notebook

!!I 51549.mp4
https://t.me/USMLEWorldStep1 Show all X
:= ltem_6of30
Question Id: 1646
■ t7' Mark <:::
Previous
C>
Next
~~
Full Screen
®
Tutorial
il
Lab Values
~•
Notes Calculator
,
Reverse Color
~
Text Zoom
©
Settings

Patients with untreated AIDS who have CD4 counts <50/mm3 are at high risk for cytomegalovirus (CMV)
retinitis. Suspicion is often raised when a patient with advanced A IDS develops slowly progressive blurred
vision, scotoma (blind spots), floaters, or photopsia (flashing lights). Although most cases are thought to arise due
to hematogenous dissemination of CMV, serum testing for the virus is often (-50%>) negative. Therefore, the
diagnosis is typically established when funduscopy reveals yellow-white, fluffy retinal lesions near the retinal
vessels associated with hemorrhage.

Treatment is required to prevent progression and blindness. Ganciclovir, a guanine nucleoside analogue that is
structurally similar to acyclovir but expresses greater activity against CMV DNA polymerase, is first-line therapy.
Patients are also typically started on antiretroviral therapy to increase CD4 count. A major complication of CMV
retinitis is retinal detachment due to the replacement of damaged retina with thin, atrophic scar tissue (which is
prone to tear).

(Choice A) Acyclovir is used to treat herpes simplex (HSV) and varicella zoster virus (VZV) infections. Although
these infections occasionally cause acute retinal necrosis, most cases arise in older, immunocompetent individuals
(not patients with advanced A IDS). In addition, HSV retinitis and VZV retinitis are usually associated with
significant vitreal inflammation, spotty peripheral retinal lesions, and infrequent hemorrhage.

(Choice B) Candida endophthalmitis is treated with amphotericin B. Funduscopy usually reveals focal, white,
mound-like lesions on the retina. Most cases occur in the setting of an indwell ing central venous catheter or total
parenteral nutrition.

(Choice C) Clarithromycin is used in conjunction with other antibiotics for the treatment of Mycobacterium avium
complex infection, which usually causes fever, night sweats, abdominal pain, and diarrhea.

(Choice D) Cryptococcus neoformans infections are treated with flucytosine, a fluorinated analog of cytosine, in
b. r
·th •h t • • B Alth •h •t d •hth I l • f t "th d d
Block Time Elapsed: 00:01:34 ~ II;! rvo Cl @ 0
My Notebook Flashcards Feedback Suspend End Block

"' !!I 51549.mp4


"' https://t.me/USMLEWorldStep1 Show all X
:= ltem_6of30 ■ t7' Mark <::: C> ~~ ® il ~- ....I , ~ ©
-
Question Id: 1646
- - .- - - .. .
- - -. -- - . ·- .. -. -
Previous
-'' Next
- .. - ,- ''
Full Screen Tutorial Lab Values Notes Calculator Reverse Color Text Zoom Settings

prone to tear).

(Choice A) Acyclovir is used to treat herpes simplex (HSV) and varicella zoster virus (VZV) infections. Although
these infections occasionally cause acute retinal necrosis, most cases arise in older, immunocompetent individuals
(not patients with advanced A IDS). In addition, HSV retinitis and VZV retinitis are usually associated with
significant vitreal inflammation, spotty peripheral retinal lesions, and infrequent hemorrhage.

(Choice B) Candida endophthalmitis is treated with amphotericin B. Funduscopy usually reveals focal , white,
mound-like lesions on the retina. Most cases occur in the setting of an indwell ing central venous catheter or total
parenteral nutrition.

(Choice C) Clarithromycin is used in conjunction with other antibiotics for the treatment of Mycobacterium avium
complex infection, which usually causes fever, night sweats, abdominal pain, and diarrhea.

{Choice D) Cryptococcus neoformans infections are treated with flucytosine, a fluorinated analog of cytosine, in
combination with amphotericin B. Although cryptococcal endophthalmitis can occur in patients with advanced
AIDS, it is much less common than CMV retinitis, and patients often have other manifestations of cryptococcal
infection such as skin lesions or meningoencephalitis.

(Choice F) Penicillin G is used to treat ocular (and neuro-) syphilis. Ocular syphilis often presents as uveitis but
can cause retinitis, which usually appears as ground-glass lesions w ith significant vitreal inflammation. Most
patients also have manifestations of syphilitic meningitis (eg, headache, vomiting).

Educational objective:
Cytomegalovirus retinitis is the most common cause of ocular disease in patients with untreated AIDS who have
CD4 counts <50/mm3. Diagnosis is made by funduscopy, which typically reveals yellow-white, fluffy retinal lesions
near the retinal vessels w ith associated hemorrhage. Treatment w ith ganciclovir is required to prevent blindness.

Block Time Elapsed: 00:01:34 ~ II;! rvo Cl @ 0


My Notebook Flashcards Feedback Suspend End Block

"' !!I 51549.mp4


"' https://t.me/USMLEWorldStep1 Show all X
= ltem_7of30
Question Id: 15838
■ t7' Mark <:::
Previous
C>
Next
~~
Full Screen
®
Tutorial
il
Lab Values

Notes
....I
Calculator
,
Reverse Color
~
Text Zoom
©
Settings

A 28-year-old man comes to the emergency department after sustaining an accidental penetrating injury to the left
eye. Examination shows left globe perforation with decreased visual acuity. The right eye is normal. The patient
is otherwise healthy. Surgical treatment is performed with subsequent improvement in vision . Two months later,
the patient experiences pain, photophobia, and diminished vision in the right eye. Evaluation shows leukocytes in
the anterior chamber and vitreous humor and choroidal deposits consistent with granulomatous panuveitis.
Disruption of which of the following immune processes is most likely responsible for this patient's current condition?

0 A . Complement regulation

0 8 . Immune privilege

0 C. Immune surveillance

0 D. lmmunoglobulin class switching

0 E. Positive selection

Submit

Block Time Elapsed: 00:01:35 ~ II;! rvo Cl @ 0


My Notebook Flashcards Feedback Suspend End Block

"' !!I 51549.mp4


"' https://t.me/USMLEWorldStep1 Show all X
= ltem_7of30
Question Id: 15838
■ t7' Mark <:::
Previous
C>
Next
~~
Full Screen
®
Tutorial
il
Lab Values

Notes
....I
Calculator
,
Reverse Color
~
Text Zoom
©
Settings

A 28-year-old man comes to the emergency department after sustaining an accidental penetrating injury to the left
eye. Examination shows left globe perforation with decreased visual acuity. The right eye is normal. The patient
is otherwise healthy. Surgical treatment is performed with subsequent improvement in vision. Two months later,
the patient experiences pain, photophobia, and diminished vision in the right eye. Evaluation shows leukocytes in
the anterior chamber and vitreous humor and choroidal deposits consistent with granulomatous panuveitis.
Disruption of which of the following immune processes is most likely responsible for this patient's current condition?

x ® A . Complement regulation (10%)

✓ Q 8 . Immune privilege (46%)

0 C. Immune surveillance (26°/o)

0 D. lmmunoglobulin class switching (6%)

0 E. Positive selection (10%)

Incorrect
Correct answer I 1,1 46% ri\ 08 secs
~ Answered correctly ~ Time Spent
B

Explanation

This patient with a traumatic ocular injury has developed granulomatous inflammation of the contralateral eye,
a condition known as sympathetic ophthalmia. This occurs when there is a robust T-cell response to previously
sequestered antigens in the eye, an area that displays immune privilege.

-· . . ... - -- - - - - . - ··- -· .... ·-. --- . . . . . .... .. ... -·.


Block Time Elapsed: 00:01:42 ~ II;! rvo Cl @ 0
My Notebook Flashcards Feedback Suspend End Block

"' !!I 51549.mp4


"' https://t.me/USMLEWorldStep1 Show all X
:= ltem_7of30
Question Id: 15838
■ t7' Mark <:::
Previous
C>
Next
~~
Full Screen
®
Tutorial
il
Lab Values
~•
Notes Calculator
,
Reverse Color
~
Text Zoom
©
Settings

This patient with a traumatic ocular injury has developed granulomatous inflammation of the contralateral eye,
a condition known as sympathetic ophthalmia. This occurs when there is a robust T-cell response to previously
sequestered antigens in the eye, an area that displays immune privilege.

Certain anatomic sites (eg , eyes, testes) have inherent immune privilege, in which inflammation is inhibited by
multiple mechanisms (eg, physical barriers, lack of lymphatics, low expression of MHC class la); this helps limit
potential organ damage that may result from a robust inflammatory response.

Self-antigens located in immune-privileged sites can be recognized by T cells that escape negative selection in the
thymus. Therefore, if these self-antigens are released into the lymphatic system as a result of trauma, T cells may
recognize these self-antigens as foreign and mount a response in both the injured eye and the contralateral
eye. Bacterial introduction and exposure during trauma may upregulate the inflammatory process, increasing the
chances that these autoreactive T cells w ill develop.

Because of this potential sight-threatening immune response, if an eye is severely injured with no possibility that
vision will be restored, it is enucleated (ie, surgically removed) to prevent blindness in the uninjured eye.

(Choice A) Disorders of complement regulation can lead to autoimmune disease (eg, paroxysmal nocturnal
hemoglobinuria) or increased susceptibility to infections with encapsulated bacteria (eg , Neisseria), but these
disorders are typically genetic conditions rather than the result of trauma .

(Choice C) Immune surveillance is a process by which cytotoxic lymphocytes and natural killer cells can
recognize neoplastic cells either by their expression of tumor antigens or by a lack of expression of self-antigens
(eg, MHC class I).

(Choice D) lmmunoglobulin class switching is the process through which activated B lymphocytes switch
production from lgM immunoglobulins during the primary immune response to lgG and lgA isotypes, which are

Block Time Elapsed: 00:01:42 ~ II;! rvo Cl @ 0


My Notebook Flashcards Feedback Suspend End Block

"' !!I 51549.mp4


"' https://t.me/USMLEWorldStep1 Show all X
:= ltem_7of30
Question Id: 15838
■ t7' Mark <:::
Previous
C>
Next
~~
Full Screen
®
Tutorial
il
Lab Values
~•
Notes Calculator
,
Reverse Color
~
Text Zoom
©
Settings
eye. Bacterial introduction and exposure during trauma may upregulate the inflammatory process, increasing the
chances that these autoreactive T cells w ill develop.

Because of this potential sight-threatening immune response, if an eye is severely injured with no possibility that
vision will be restored, it is enucleated (ie, surgically removed) to prevent blindness in the uninjured eye.

(Choice A) Disorders of complement regulation can lead to autoimmune disease (eg, paroxysmal nocturnal
hemoglobinuria) or increased susceptibility to infections with encapsulated bacteria (eg , Neisseria), but these
disorders are typically genetic conditions rather than the result of trauma .

(Choice C) Immune surveillance is a process by which cytotoxic lymphocytes and natural killer cells can
recognize neoplastic cells either by their expression of tumor antigens or by a lack of expression of self-antigens
(eg, MHC class I).

(Choice D) lmmunoglobulin class switching is the process through which activated B lymphocytes switch
production from lgM immunoglobulins during the primary immune response to lgG and lgA isotypes, which are
active during the secondary response. This occurs in the germinal centers of lymph nodes and requires interaction
of the CO40 receptor on B cells with the CD40 ligand expressed by activated T cells.

(Choice E) Positive selection of T cells occurs prior to negative selection and is responsible for development of a
T cell repertoire that can recognize and bind to self-MHC molecules. T cells expressing a T-cell receptor that is
able to bind self-MHC are allowed to survive (although if that response is too strong, they will usually be eliminated
through negative selection).

Educational objective:
Traumatic injury to the eye, an area of the body that has inherent immune privilege, can lead to the release of
previously sequestered antigens that T cells recognize as foreign. This can lead to subsequent inflammation and
blindness in both the injured and uninjured eye.

Block Time Elapsed: 00:01:42 ~ II;! rvo Cl @ 0


My Notebook Flashcards Feedback Suspend End Block

"' !!I 51549.mp4


"' https://t.me/USMLEWorldStep1 Show all X
= --
ltem_7of30
Question Id: 15838
--·
■ t7' Mark
- -.-,- - - .,- -- ,-.,-.-"
<
Previous
C>
Next
- -· .- .. ,
~~
Full Saeen
®
Tutorial
il
Lab Values

Notes
....I
Calculator
,
Reverse Color
~
Text Zoom
©
Settings

mu -
_ Exhibit Display Bl x
pot

Self
thy
Paroxysmal nocturnal
Normal
rec hemoglobinuria

eye. Complement
Lysis & release of
components
cha
, y free hemoglobin
I
Bee
VISI

(Ch
he
diso

(Ch
re
Absence of COSS & COS9
(eg, GPI anchors COSS & COS9 to proteins allows terminal
RBC membrane, inhibiting MACs to form
(Ch complement deposition

GPI • glycosylphosphaticfy1oositol: MAC • membrane a n a c k c o m p l e •: R B C • red blood cell. ~ UWorld

oft
©_ Zoom In 0_ Zoom Out C Reset Cf;, New I lf-J Existi ng (!;I My Notebook
(Ch
Tee
-- - - - -- -- . , - - - - -- - - - - - -- -- -- ------- - -- - --- -- - - -- - -- -- --
Block Time Elapsed: 00:01:42 ~ II;! rvo Cl @ 0
My Notebook Flashcards Feedback Suspend End Block

"' !!I 51549.mp4


"' https://t.me/USMLEWorldStep1 Show all X
= ltem_Sof30
Question Id: 11551
■ t7' Mark <:::
Previous
C>
Next
~~
Full Screen
®
Tutorial
il
Lab Values

Notes
....I
Calculator
,
Reverse Color
~
Text Zoom
©
Settings

A 22-year-old woman comes to the office for a routine prenatal visit. She is 16 weeks pregnant with her first child.
Two months ago, she had a mononucleosis-like illness with fever, myalgia, and fatigue, and she was diagnosed
with cytomegalovirus infection . The patient recovered with symptomatic treatment and is currently asymptomatic.
Her other medical problems include episodic migraine headaches and benign choroidal nevus. She takes prenatal
vitamins and does not use tobacco, alcohol , or illicit drugs. The patient has a family history of hypertension and
glaucoma. Physical examination findings and prenatal ultrasound are unremarkable. This patient's unborn infant
is at greatest risk for which of the following eye conditions?

0 A . Chorioretinitis

0 8 . Choroidal nevus

0 C. Congenital cataract

0 D. Glaucoma

0 E. Inclusion conjunctivitis

0 F. Retinitis pigmentosa

0 G. Retinoblastoma

Submit

Block Time Elapsed: 00:02:01 ~ https://t.me/USMLEWorldStep1 II;! rvo Cl @) 0


My Notebook Flashcards Feedback Suspend End Block
= ltem_Sof30
Question Id: 11551
■ t7' Mark <:::
Previous
C>
Next
~~
Full Screen
®
Tutorial
il
Lab Values

Notes
....I
Calculator
,
Reverse Color
~
Text Zoom
©
Settings

A 22-year-old woman comes to the office for a routine prenatal visit. She is 16 weeks pregnant with her first child .
Two months ago, she had a mononucleosis-like illness with fever, myalgia, and fatigue, and she was diagnosed
with cytomegalovirus infection . The patient recovered with symptomatic treatment and is currently asymptomatic.
Her other medical problems include episodic migraine headaches and benign choroidal nevus. She takes prenatal
vitamins and does not use tobacco, alcohol , or illicit drugs. The patient has a family history of hypertension and
glaucoma. Physical examination findings and prenatal ultrasound are unremarkable. This patient's unborn infant
is at greatest risk for which of the following eye conditions?

✓ Q A . Chorioretinitis (60°/o)

X ® 8 . Choroidal nevus (2%)

0 C. Congenital cataract (18%)

0 D. Glaucoma (3%)

0 E. Inclusion conjunctivitis (8%)

0 F. Retinitis pigmentosa (5%)

0 G. Retinoblastoma (1%)

Incorrect
Correct answer I 1,1 60% fi"\ 22 secs 2023
~ Answered correctly \.:.JTimeSpent Version
A

Explanation

Up to 5% of women develop a primary cytomegalovirus (CMV) infection (typically a mononucleosis-like illness


with fever and mild hepatitis) during pregnancy. Vertical transmission to the fetus occurs in approximately one-third

Block Time Elapsed: 00:02:04 ~ https://t.me/USMLEWorldStep1 II;! rvo Cl @) 0


My Notebook Flashcards Feedback Suspend End Block
=
-
ltem_Sof30
Question Id: 11551
■ t7' Mark <:::
Previous
C>
Next
~~
Full Screen
®
Tutorial
il
Lab Values
~•
Notes Calculator
,
Reverse Color
~
Text Zoom
©
Settings

Explanation

Up to 5% of women develop a primary cytomegalovirus (CMV) infection (typically a mononucleosis-like illness


with fever and mild hepatitis) during pregnancy. Vertical transmission to the fetus occurs in approximately one-third
of cases, with the highest risk of transmission occurring in the first trimester. CMV-related complications
observed in infants exposed to the virus in utero include chorioretinitis (most common eye-related problem),
sensorineural deafness, seizures, jaundice, hepatomegaly, splenomegaly, and microcephaly.

(Choice B) Choroidal nevus (eye freckle) is a nonhereditary, usually benign pigmented area underneath the retina
and is not associated with in-utero CMV infection.

(Choice C) Congenital cataract can be seen with in-utero rubella infection .

(Choice D) Glaucoma can be present at birth and may be primary or secondary (eg , tumor). However, it is not a
known complication of CMV infection .

(Choice E) Inclusion conjunctivitis is seen with Chlamydia trachomatis infection of the newborn .

(Choice F) Retinitis pigmentosa is an inherited degenerative eye disease that leads to severe vision impairment
and is not a complication of in-utero CMV infection.

(Choice G) Retinoblastoma is the most common malignant eye cancer in children and is caused by a
chromosomal mutation that is hereditary or sporadic. CMV infection in utero is not associated with causing
retinoblastoma.

Educational objective:
The most common eye-related complication of congenital cytomegalovirus infection is chorioretinitis.

References
• Ocular abnormalities in congenital cytomegalovirus infection.

Block Time Elapsed: 00:02:04 ~ https://t.me/USMLEWorldStep1 II;! rvo Cl @) 0


My Notebook Flashcards Feedback Suspend End Block
= ltem_9of30
Question Id: 16172
■ t7' Mark <:::
Previous
C>
Next
~~
Full Screen
®
Tutorial
il
Lab Values

Notes
....I
Calculator
,
Reverse Color
~
Text Zoom
©
Settings

A 52-year-old man comes to the office after his wife noted that his eyes appeared different. The patient reports no
symptoms. Medical history is significant for hypertension and type 2 diabetes mellitus. He has smoked 1.5 packs
of cigarettes daily for 25 years. Blood pressure is 140/86 mm Hg and pulse is 84/min. On physical examination,
the right pupil is 3 mm and the left pupil is 5 mm. When the light in the examination room is dimmed, the
anisocoria increases. Which of the following is the most likely location of the lesion?

0 A . Left oculomotor nerve

0 B. Left oculosympathetic pathway

0 C. Left optic nerve

0 D. Right oculomotor nerve

0 E. Right oculosympathetic pathway

0 F. Right optic nerve

Submit

Block Time Elapsed: 00:02:07 ~ https://t.me/USMLEWorldStep1 II;! rvo Cl @) 0


My Notebook Flashcards Feedback Suspend End Block
= ltem_9of30
Question Id: 16172
■ t7' Mark <:::
Previous
C>
Next
~~
Full Screen
®
Tutorial
il
Lab Values

Notes
....I
Calculator
,
Reverse Color
~
Text Zoom
©
Settings

A 52-year-old man comes to the office after his wife noted that his eyes appeared different. The patient reports no
symptoms. Medical history is significant for hypertension and type 2 diabetes mellitus. He has smoked 1.5 packs
of cigarettes daily for 25 years. Blood pressure is 140/86 mm Hg and pulse is 84/min . On physical examination,
the right pupil is 3 mm and the left pupil is 5 mm. When the light in the examination room is dimmed, the
anisocoria increases. Which of the following is the most likely location of the lesion?

0 A . Left oculomotor nerve (18°/o)

x ® B. Left oculosympathetic pathway (14%)

0 C. Left optic nerve (6%)

0 D. Right oculomotor nerve (12%)

✓0 E. Right oculosympathetic pathway (40%)

0 F. Right optic nerve (7%)

Incorrect
Correct answer I 1,1 40% f"i\ 06 secs
~ Answered correctly \..::.JTimeSpent
E

Explanation

Pupillary asymmetry
In room light

Block Time Elapsed: 00:02:10 ~ https://t.me/USMLEWorldStep1 II;! rvo Cl @) 0


My Notebook Flashcards Feedback Suspend End Block
C
E
Pupillary asymmetry
E In room light

Sympathetic defect (right eye) Parasympathetic defect (left eye)


Increases in dim light Increases in bright light

Decreases in bright light Decreases in dim light

C)UWortd

©_ Zoom In 0_ Zoom Out C Reset Cf;, New I 1f-J Existi ng (!;I My Notebook

This patient has anisocoria (ie, pupillary asymmetry), which indicates a unilateral defect in input from either the

Block Time Elapsed: 00:02:10 ~ https://t.me/USMLEWorldStep1 11;1 rvo Cl @) 0


My Notebook Flashcards Feedback Suspend End Block
=
-
ltem_9of30
Question Id: 16172
■ t7' Mark <:::
Previous
C>
Next
~~
Full Screen
®
Tutorial
il
Lab Values
~•
Notes Calculator
,
Reverse Color
~
Text Zoom
©
Settings

This patient has anisocoria (ie, pupillary asymmetry), which indicates a unilateral defect in input from either the
ocular sympathetic (pupil dilation) or parasympathetic (pupil constriction) pathway. In response to dim light, input
from the oculosympathetic pathway initiates pupillary dilation , allowing more light to reach the retina. Under
bright light, parasympathetic input from the ipsilateral oculomotor nerve (CN Ill ) initiates pupillary constriction,
limiting the amount of light received by the retina.

Determining which pathway is affected in anisocoria can be accomplished by observing the change in pupillary
discrepancy in both bright and dim light:

• Asymmetry that increases in dim light indicates that the smaller pupil is unable to dilate due to loss of
sympathetic nerve input. Under bright light, the asymmetry will decrease because parasympathetic input
(pupillary constriction) is unaffected .

• Asymmetry that increases under bright light indicates that the larger pupil is unable to constrict due to loss of
parasympathetic nerve input. In dim light, the asymmetry will decrease because sympathetic input (pupillary
dilation) is unaffected.

This patient has increased asymmetry in a dim room, indicating that the smaller (right) pupil is unable to dilate
due to loss of sympathetic nerve input secondary to a lesion in the right oculosympathetic pathway.

(Choices A and D) An oculomotor nerve lesion would disrupt parasympathetic innervation of the ipsilateral pupil,
resulting in anisocoria that increases under bright light because the abnormal larger pupil is unable to constrict in
response to light.

(Choice B) Damage to the left oculosympathetic pathway would cause loss of sympathetic input to the left eye,
resulting in a fixed , constricted pupil in the left eye (with increased asymmetry in dim light) rather than in the right.

(Choices C and F) The optic nerves carry the afferent portions of the pupillary reflex from each eye. A unilateral
optic nerve lesion would not cause pupillary asymmetry, but instead monocular vision loss and decreased bilateral
pupillary constriction in response to light entering the affected eye (ie, relative afferent pupillary defect).

Block Time Elapsed: 00:02:10 ~ https://t.me/USMLEWorldStep1 II;! rvo Cl @) 0


My Notebook Flashcards Feedback Suspend End Block
:= ltem_9of30
Question Id: 16172
■ t7' Mark <:::
Previous
C>
Next
~~
Full Screen
®
Tutorial
il
Lab Values
~-
Notes
....I
Calculator
,
Reverse Color
~
Text Zoom
©
Settings

Determining which pathway is affected in anisocoria can be accomplished by observing the change in pupillary
discrepancy in both bright and dim light:

• Asymmetry that increases in dim light indicates that the smaller pupil is unable to dilate due to loss of
sympathetic nerve input. Under bright light, the asymmetry will decrease because parasympathetic input
(pupillary constriction) is unaffected .

• Asymmetry that increases under bright light indicates that the larger pupil is unable to constrict due to loss of
parasympathetic nerve input. In dim light, the asymmetry will decrease because sympathetic input (pupillary
dilation) is unaffected.

This patient has increased asymmetry in a dim room, indicating that the smaller (right) pupil is unable to dilate
due to loss of sympathetic nerve input secondary to a lesion in the right oculosympathetic pathway.

(Choices A and D) An oculomotor nerve lesion would disrupt parasympathetic innervation of the ipsilateral pupil,
resulting in anisocoria that increases under bright light because the abnormal larger pupil is unable to constrict in
response to light.

(Choice B) Damage to the left oculosympathetic pathway would cause loss of sympathetic input to the left eye,
resulting in a fixed , constricted pupil in the left eye (with increased asymmetry in dim light) rather than in the right.

(Choices C and F) The optic nerves carry the afferent portions of the pupillary reflex from each eye. A unilateral
optic nerve lesion would not cause pupillary asymmetry, but instead monocular vision loss and decreased bilateral
pupillary constriction in response to light entering the affected eye (ie, relative afferent pupillary defect).

Educational objective:
Pupillary asymmetry (ie, anisocoria) is caused by a lesion in the ocular parasympathetic (pupillary constriction) or
sympathetic (pupillary dilation) pathways. In this patient, the pupillary asymmetry increases in a dim room,
indicating that the smaller right pupil is unable to dilate due to a lesion in the right oculosympathetic pathway.

Block Time Elapsed: 00:02:10 ~ https://t.me/USMLEWorldStep1 II;! rvo Cl @) 0


My Notebook Flashcards Feedback Suspend End Block
= ltem_9of30
Question Id: 16172
■ t7' Mark <:::
Previous
C>
Next
~~
Full Screen
®
Tutorial
il
Lab Values

Notes
....I
Calculator
,
Reverse Color
~
Text Zoom
©
Settings

from the oculosympathetic pathway initiates pupillary dilation , allowing more light to reach the retina. Under
brig Exhibit Display Bl X
limit
Pupillary light reflex
Det
disc

Ciliary ganglion -

This Oculomotor
nerve (CN Ill)
due

(Ch
res
res

res /
Lateral geniculate
nucleus Edinger-Westphal
(Ch Pretectal nucleus nucleus
OUVVortd

opti
pupi
©. Zoom In 0. Zoom O ut a Reset q;, New I lf-J Existi ng I!;! My Notebook

•,.11. uu-11 --•••· • ·••--••••-• . ,•• 111.1- 11.a: •• •••

Block Time Elapsed: 00:02:10 ~ https://t.me/USMLEWorldStep1 11;1 rvo Cl @) 0


My Notebook Flashcards Feedback Suspend End Block
= ltem_10of30
Question Id: 8594
■ t7' Mark <:::
Previous
C>
Next
~~
Full Screen
®
Tutorial
il
Lab Values

Notes
....I
Calculator
,
Reverse Color
~
Text Zoom
©
Settings

A 57-year-old man is brought to the emergency department following a generalized tonic-clonic seizure. His w ife
reports that he has no history of seizures. However, she says that he has been complaining of intermittent
headaches, memory loss, and vision disturbances for the past 2 weeks. Brain imaging shows a solitary mass
within the right temporal lobe. Which of the following visual field defects (areas of vision loss in black) is most likely
present in this patient?

Left Right

Q A.

QB.

QC.

Q D.


Block Time Elapsed: 00:02:13 ~ II;! rvo Cl @ 0
My Notebook Flashcards Feedback Suspend End Block

)4
"' https://t.me/USMLEWorldStep1 Show all X
=. ltem_10of30
Question Id: 8594
- - .- ■ t7' Mark
-
<:::
Previous
C>
Next
~~
Full Screen
®
Tutorial
il
Lab Values

Notes
....I
Calculator
,
Reverse Color
~
Text Zoom
©
Settings

Left Right

QA.

0 8.

QC.

Q D.

Q E.

Submit

Block Time Elapsed: 00:02:21 ~ II;! rvo Cl @ 0


My Notebook Flashcards Feedback Suspend End Block

)4
"' https://t.me/USMLEWorldStep1 Show all X
= ■ t'.7 <::: C> ~~ ® il ~· ~ , ~ ©
. .-
ltem10of30
- Question Id: 8594 \ Mark Previous Next Full Screen Tutorial Lab Values Notes Calculator Reverse Color Text Zoom Settings
- - -

Left Right

Q A.

X (!) 8 . (60/o)

Q C. (24%)

✓ Q D. (57%)

Q E. (7%)

Incorrect

)4
https://t.me/USMLEWorldStep1 Show all X
Optic radiations

C
D Superior optic radiation: : J
Information from inferior visual field
(superior retina)
P------,
E ( Parietal lobe

Lateral geniculate nucleus

Inferior optic radiations


(ie, Meyer loop)
Information from superior visual field
(inferior retina)

© uWorld

©. Zoom In 0. Zoom Out a Reset q;, New I lf-J Existi ng I!;! My Notebook

)4
https://t.me/USMLEWorldStep1 Show all X
= Item _10 of 30
Question Id: 8594
■ t7' Mark <:::
Previous
C>
Next
~~
Full Screen
®
Tutorial
il
Lab Values
~-
Notes
....I
Calculator
,
Reverse Color
~
Text Zoom
©
Settings

After traversing the lateral geniculate nucleus, visual information travels to the primary visual cortex in the occipital
lobe by way of the optic radiations. The optic radiations fan out as they proj ect posteriorly into 2 major pathways:

• Superior optic radiations carry information from the superior retina/inferior visual field. They travel through the
parietal lobe, so lesions in that area result in contralateral inferior quadrant visual field defects (ie, contralateral
homonymous inferior quadrantanopia) (Choice E).

• Inferior optic radiations carry information from the inferior retina/superior visual field. They travel through the
temporal lobe (Meyer loop), so lesions in that area (such as this patient's temporal lobe mass) result in
contralateral superior quadrant visual field defects (ie, contralateral homonymous superior
quadrantanopia).

Therefore, this patient's tumor in the right temporal lobe is likely impacting the inferior optic radiations, causing a
left homonymous (both eyes the same) superior quadrantanopia.

(Choice A) Monocular vision loss results from a lesion anterior to the optic chiasm, including lesions to the optic
nerve, retina, or globe.

(Choice B) In the optic chiasm, fibers from the nasal retina (which process the temporal visual field) cross so that
the hemivisual fields from both eyes are processed in the same contralateral hemisphere. A lesion that affects
these crossing fibers in the middle of the optic chiasm can produce bitemporal hemianopia.

(Choice C) Left homonymous hemianopia may occur with lesions to the right optic tract or the right occipital
cortex. It may also occur with lesions to the optic radiations, but only if they affect both the superior and inferior
optic radiations. This is less likely to occur with a mass lesion localized within the temporal lobe.

Educational objective:
The inferior optic radiations travel through the temporal lobe in the Meyer loop. They carry information from the

Block Time Elapsed: 00:02:24 ~ II;! rvo Cl @ 0


My Notebook Flashcards Feedback Suspend End Block

)4
"' https://t.me/USMLEWorldStep1 Show all X
= Item _10 of 30
Question Id: 8594
■ t7' Mark <:::
Previous
C>
Next
~~
Full Screen
®
Tutorial
il
Lab Values
~-
Notes
....I
Calculator
,
Reverse Color
~
Text Zoom
©
Settings

• Superior optic radiations carry information from the superior retina/inferior visual field. They travel through the
parietal lobe, so lesions in that area result in contralateral inferior quadrant visual field defects (ie, contralateral
homonymous inferior quadrantanopia) (Choice E) .

• Inferior optic radiations carry information from the inferior retina/superior visual field . They travel through the
temporal lobe (Meyer loop), so lesions in that area (such as this patient's temporal lobe mass) result in
contralateral superior quadrant visual field defects (ie, contralateral homonymous superior
quadrantanopia).

Therefore, this patient's tumor in the right temporal lobe is likely impacting the inferior optic radiations, causing a
left homonymous (both eyes the same) superior quadrantanopia.

(Choice A) Monocular vision loss results from a lesion anterior to the optic chiasm, including lesions to the optic
nerve, retina, or globe.

(Choice B) In the optic chiasm, fibers from the nasal retina (which process the temporal visual field) cross so that
the hemivisual fields from both eyes are processed in the same contralateral hemisphere. A lesion that affects
these crossing fibers in the middle of the optic chiasm can produce bitemporal hemianopia.

(Choice C) Left homonymous hemianopia may occur with lesions to the right optic tract or the right occipital
cortex. It may also occur with lesions to the optic radiations, but only if they affect both the superior and inferior
optic radiations. This is less likely to occur with a mass lesion localized within the temporal lobe.

Educational objective:
The inferior optic radiations travel through the temporal lobe in the Meyer loop. They carry information from the
inferior retina, which covers the superior visual field. Therefore, temporal lobe lesions can result in contralateral
homonymous superior quadrantanopia.

Block Time Elapsed: 00:02:24 ~ II;! rvo Cl @ 0


My Notebook Flashcards Feedback Suspend End Block

)4
"' https://t.me/USMLEWorldStep1 Show all X
Visual pathways from above

Optic
nerve

Optic
chiasma

Optic
Afte tract

lob

• F. Left homonymous superior


~ n o p i ~ the sky")

Lateral
geniculate G. Left homonymous inferior
nucleus quadrantanopia ("pie on the floor")

Dorsal optic
radiation
~~
H. Left homonymous hemianopia
i )lars~

H
CUWorld

The ©_ Zoom In 0_ Zoom Out C Reset Cf;, New I lf-J Existi ng (!;I My Notebook
left

Block Time Elapsed: 00:02:24 ~ II;! rvo Cl @ 0


My Notebook Flashcards Feedback Suspend End Block

)4
https://t.me/USMLEWorldStep1 Show all X
= ltem_11 of30
Question Id: 11841
■ t7' Mark <:::
Previous
C>
Next
~~
Full Screen
®
Tutorial
il
Lab Values

Notes
....I
Calculator
,
Reverse Color
~
Text Zoom
©
Settings

A 58-year-old airline captain comes to the office after he was found to have abnormal vision testing during a pilot
medical certification examination . The patient feels well and has no ocular symptoms. Vision testing shows
normal visual acuity but moderate loss of peripheral vision in both eyes. Results of the funduscopic examination
are shown in the image below.

.·- ·- ... - ••
Block Time Elapsed: 00:02:28
- ...
~
··•• - - - •• • - ...... - - . - •• -. - • - -- -• -· .. - - ••·.
II;!
My Notebook
rvo
Flashcards
Cl
Feedback
@
Suspend
0
End Block

)4
"' https://t.me/USMLEWorldStep1 Show all X
= ltem_11 of30
Question Id: 11841
■ t7' Mark <
Previous
C>
Next
~~
Full Saeen
®
Tutorial
il
Lab Values
~-
Notes
....I
Calculator
,
Reverse Color
~
Text Zoom
©
Settings

AS
me

are

©_ Zoom In 0, Zoom Out C Reset Cf;, New I lf-J Existi ng (!;I My Notebook

. ·- ....... -••
Block Time Elapsed: 00:02:31
- ...
~
··••-• - -•• •- ...... - - ...... _ -• - _... -•-· .. - -••·.
II;!
My Notebook
rvo
Flashcards
Cl
Feedback
@
Suspend
0
End Block

)4
"' https://t.me/USMLEWorldStep1 Show all X
Other neurologic and cardiopulmonary examination findings are unremarkable. He is started on latanoprost
eyedrops. The medication is most likely to improve this patient's condition through which of the following
mechanisms?

O A . Decreased bicarbonate formation in ciliary body

0 8 . Decreased formation of abnormal blood vessels in retina

0 C. Decreased secretion of aqueous humor

0 D. Increased outflow of aqueous humor

0 E. Vasoconstriction in ciliary body

Submit

Block Time Elapsed: 00:02:35 ~ II;! rvo Cl @ 0


My Notebook Flashcards Feedback Suspend End Block

)4
"' https://t.me/USMLEWorldStep1 Show all X
Other neurologic and cardiopulmonary examination findings are unremarkable. He is started on latanoprost
eyedrops. The medication is most likely to improve this patient's condition through which of the following
mechanisms?

O A . Decreased bicarbonate formation in ciliary body (2%)

X® 8 . Decreased formation of abnormal blood vessels in retina (10°/o)

0 C. Decreased secretion of aqueous humor (21%)

✓0 D. Increased outflow of aqueous humor (61%)

0 E. Vasoconstriction in ciliary body (4%)

Incorrect
Correct answer I 1,1 61% ti\ 14 secs
l!!!!. Answered correctly \::.J Time Spent
D

Block Time Elapsed: 00:02:38 ~ II;! rvo Cl @ 0


My Notebook Flashcards Feedback Suspend End Block

)4
"' https://t.me/USMLEWorldStep1 Show all X
= ltem_11 of30
Question Id: 11841
■ t7' Mark <
Previous
C>
Next
~~
Full Saeen
®
Tutorial
il
Lab Values

Notes
....I
Calculator
,
Reverse Color
~
Text Zoom
©
Settings

Exhibit Display Bl x

Glaucoma medications

Cornea
Trabecular
meshwork

Trabecular outflow

Schlemm
canal

Sciera-- queous umor m o


• Beta blockers
• Alpha-2 AR agonists

Alpha-2 AR agonlsts = alpha-2 adrenerglc reeeptor agonists: CAls = carbonic anhydrase inhibitors.

Aq ©_ Zoom In 0_ Zoom Out C Reset Cf;, New I lf-J Existing (!;I My Notebook
flow
ne I • I - ~ .. - .. f - I I I .. I I I -"1 I - f I .. • - I - .. .. I ...., I f ..

Block Time Elapsed: 00:02:38 ~ II;! rvo Cl @ 0


My Notebook Flashcards Feedback Suspend End Block

)4
"' https://t.me/USMLEWorldStep1 Show all X
= ltem_11 of30
Question Id: 11841
■ t7' Mark <:::
Previous
C>
Next
~~
Full Screen
®
Tutorial
il
Lab Values
~-
Notes
....I
Calculator
,
Reverse Color
~
Text Zoom
©
Settings

Aqueous humor is secreted by epithelial cells of the ciliary body into the posterior eye chamber. The fluid then
flows through the pupil into the anterior chamber to the iridocorneal angle. Here, it diffuses through a trabecular
network into Schlemm's canal (scleral venous sinus) and subsequently drains into episcleral and conjunctiva!
veins. A small amount of aqueous can also diffuse through the ciliary muscle and adjacent sclera (uveoscleral
pathway).

Open-angle glaucoma is characterized by increased intraocular pressure due to increased secretion or


decreased outflow of aqueous humor. It is a form of optic neuropathy and causes progressive loss of ganglion cell
axons, which may be visualized as a pale optic disc and enlarged optic cup (compared to normal retina).
Symptoms evolve over decades, with progressive loss of peripheral visual fields.

Glaucoma is managed with agents that either decrease production or increase outflow of aqueous humor.
Latanoprost is a topical prostaglandin used in the treatment of glaucoma. It is applied as a prodrug and
converted to the active form by esterases in the cornea. Prostaglandins are the preferred first-line agents for the
treatment of glaucoma, and although the exact mechanism of action is uncertain, they have been found to
decrease the collagen content in the uveoscleral outflow pathway and increase outflow of aqueous humor. They
are also known to cause increased pigmentation in the iris and eyelashes.

(Choices A, C, and E) Carbonic anhydrase inhibitors (eg, dorzolamide) decrease formation of bicarbonate in the
ciliary body, leading to decreased sodium and fluid transport and subsequently decreased production of aqueous
humor. Alpha-adrenergic agonists (eg, brimonidine) inhibit production of aqueous humor due to vasoconstriction in
the ciliary body. Beta blockers (eg, timolol) also decrease production of aqueous humor. Prostaglandins do not
affect production of aqueous humor but are often used in combination with these other agents.

(Choice B) Increased formation of new blood vessels at the optic disc is the characteristic feature of proliferative
diabetic retinopathy. Strict control of diabetes will reduce the risk of diabetic retinopathy, but it is not affected by
-- - - - ...
Block Time Elapsed: 00:02:38 ~ II;! rvo Cl @ 0
My Notebook Flashcards Feedback Suspend End Block

)4
"' https://t.me/USMLEWorldStep1 Show all X
=
-
Item _11 of 30
Question Id: 11841
■ t7' Mark <:::
Previous
C>
Next
~~
Full Screen
®
Tutorial
il
Lab Values
~•
Notes Calculator
,
Reverse Color
~
Text Zoom
©
Settings

Open-angle glaucoma is characterized by increased intraocular pressure due to increased secretion or


decreased outflow of aqueous humor. It is a form of optic neuropathy and causes progressive loss of ganglion cell
axons, which may be visualized as a pale optic disc and enlarged optic cup (compared to normal retina).
Symptoms evolve over decades, with progressive loss of peripheral visual fields.

Glaucoma is managed with agents that either decrease production or increase outflow of aqueous humor.
Latanoprost is a topical prostaglandin used in the treatment of glaucoma. It is applied as a prodrug and
converted to the active form by esterases in the cornea. Prostaglandins are the preferred first-line agents for the
treatment of glaucoma, and although the exact mechanism of action is uncertain, they have been found to
decrease the collagen content in the uveoscleral outflow pathway and increase outflow of aqueous humor. They
are also known to cause increased pigmentation in the iris and eyelashes.

(Choices A, C, and E) Carbonic anhydrase inhibitors (eg, dorzolamide) decrease formation of bicarbonate in the
ciliary body, leading to decreased sodium and fluid transport and subsequently decreased production of aqueous
humor. Alpha-adrenergic agonists (eg, brimonidine) inhibit production of aqueous humor due to vasoconstriction in
the ciliary body. Beta blockers (eg, timolol) also decrease production of aqueous humor. Prostaglandins do not
affect production of aqueous humor but are often used in combination with these other agents.

(Choice B) Increased formation of new blood vessels at the optic disc is the characteristic feature of proliferative
diabetic retinopathy. Strict control of diabetes will reduce the risk of diabetic retinopathy, but it is not affected by
topical prostaglandins.

Educational objective:
Glaucoma is a form of optic neuropathy characterized by increased intraocular pressure associated with increased
production or decreased outflow of aqueous humor. Topical prostaglandins (eg , latanoprost) increase outflow of
aqueous via the uveoscleral pathway and are the preferred treatment for open-angle glaucoma.

Block Time Elapsed: 00:02:38 ~ II;! rvo Cl @ 0


My Notebook Flashcards Feedback Suspend End Block

)4
"' https://t.me/USMLEWorldStep1 Show all X
dee
axo
Sy

Gia
Lat
con
trea
dee
are

(Ch
©_ Zoom In 0_ Zoom Out C Reset Cf;, New I lf-J Existi ng (!;I My Notebook
cilia
hu
the ciliary body. Beta blockers (eg, timolol) also decrease production of aqueous humor. Prostaglandins do not

Block Time Elapsed: 00:02:38 ~ II;! rvo Cl @ 0


My Notebook Flashcards Feedback Suspend End Block

)4
"' https://t.me/USMLEWorldStep1 Show all X
Glaucomatous optic atrophy (moderate)

dee
axo
Sy

Gia
Lat
con
trea
dee
are
C)IMlo<ld

(Ch
©_ Zoom In 0_ Zoom Out C Reset Cf;, New I lf-J Existi ng (!;I My Notebook
cilia
hu
the ciliary body. Beta blockers (eg, timolol) also decrease production of aqueous humor. Prostaglandins do not

Block Time Elapsed: 00:02:38 ~ II;! rvo Cl @ 0


My Notebook Flashcards Feedback Suspend End Block

)4
"' https://t.me/USMLEWorldStep1 Show all X
= Item _12 of 30
Question Id: 11675
■ t7' Mark <:::
Previous
C>
Next
~~
Full Screen
®
Tutorial
il
Lab Values

Notes
....I
Calculator
,
Reverse Color
~
Text Zoom
©
Settings

A 62-year-old man with a history of poor medical follow-up comes to the office due to difficulty seeing . He has had
blurry vision for the last 2 weeks. A day ago, he noticed a shadow develop suddenly across the visual field of his
right eye. The patient has had no recent headaches, double vision, vertigo, light sensitivity, or nausea. On
examination, pupils are equal and reactive to light. Visual field testing by confrontation reveals patchy loss of
vision . A flame-shaped retinal hemorrhage in the right eye is noted on funduscopic examination, as shown in the
image below.

Which of the following is the most likely cause of his visual disturbance?

0 A . Central retinal artery occlusion

Block Time Elapsed: 00:02:40 ~ II;! rvo Cl @ 0


My Notebook Flashcards Feedback Suspend End Block

)4
"' https://t.me/USMLEWorldStep1 Show all X
=
-
ltem12of30
Question Id: 11675
■ t:,,
\ Mark
<
Previous
C>
Next
~~
Full Saeen
®
Tutorial
i l
Lab Values
~ ·
Notes
....1
Calculator
,
Reverse Color
~
Text Zoom
©
Settings

A6
, Exhibit Display Bl x
blur
righ
exa
V IS I

ima

©_ Zoom In 0_ Zoom Out C Reset Cf;, New I lf-J Existi ng (!;I My Notebook

)4
https://t.me/USMLEWorldStep1 Show all X
Which of the following is the most likely cause of his visual disturbance?

0 A . Central retinal artery occlusion

0 8 . Cigarette smoking

0 C. Closed-angle glaucoma

0 D. Giant cell arteritis

0 E. Hyperlipidemia

0 F. Hypertension

0 G. Migraine

0 H. Thyroid eye disease

Submit

Block Time Elapsed: 00:02:49 ~ II;! rvo Cl @ 0


My Notebook Flashcards Feedback Suspend End Block

)4
"' https://t.me/USMLEWorldStep1 Show all X
Which of the following is the most likely cause of his visual disturbance?

0 A. Central retinal artery occlusion (18%)

0 8 . Cigarette smoking (4%)

X ® C. Closed-angle glaucoma (4°/o)

0 D. Giant cell arteritis (3%)

0 E. Hyperlipidemia (3%)

✓ Q F. Hypertension (66%)

0 G. Migraine (0%)

0 H. Thyroid eye disease (0%)

Incorrect
Correct answer I 1,1 66% ti\ 15 secs
l!!!!. Answered correctly \::.J Time Spent
F

Block Time Elapsed: 00:02:53 ~ II;! rvo Cl @ 0


My Notebook Flashcards Feedback Suspend End Block

)4
"' https://t.me/USMLEWorldStep1 Show all X
Malignant arterial hypertension
C
F

AV = arteriovenous. Cuworld

©_ Zoom In 0_ Zoom Out C Reset Cf;, New I lf-J Existing (!;I My Notebook

)4
https://t.me/USMLEWorldStep1 Show all X
= Item _12 of 30
Question Id: 11675
■ t7' Mark <:::
Previous
C>
Next
~~
Full Screen
®
Tutorial
il
Lab Values
~-
Notes
....I
Calculator
,
Reverse Color
~
Text Zoom
©
Settings

This patient has an acute retinal hemorrhage (arrow), most likely caused by severe, poorly controlled
hypertension. Hypertensive retinal hemorrhage typically causes painless, unilateral visual disturbances, ranging
from mild obscuration without loss of visual acuity to permanent blindness. Severe hypertension in retinal
precapillary arterioles causes endothelial disruption , leakage of plasma into the arteriolar wall, and fibrinous
necrosis. The necrotic vessels can then bleed into the nerve fiber layers, causing dot- and flame-shaped
hemorrhages.

Diagnosis of hypertensive retinal hemorrhage is usually confirmed by direct ophthalmoscopic evaluation . Other
findings of hypertensive retinopathy include thickening of the arteriolar walls ("copper or silver wiring"),
compression of the associated veins (arteriovenous nicking), and small , white foci of retinal ischemia (cotton-wool
spots).

(Choices A and D) Central retinal artery occlusion is most commonly due to atherosclerosis, cardioembolic
disease, or vasculitis (eg, giant cell arteritis). It causes acute monocular vision loss. Examination can reveal a
cherry red spot at the macula due to diffuse retinal ischemia, but hemorrhage is not typically seen.

(Choices B and E) Smoking and hyperlipidemia can increase the risk of developing ischemic retinal and optic
vascular disease; however, these are not associated with an increased risk of retinal hemorrhage.

(Choice C) Closed-angle glaucoma is characterized by increased intraocular pressure from impaired aqueous
humor drainage. It causes acute headache, eye pain, vision loss, and an afferent pupillary defect.
Ophthalmoscopic examination reveals cupping of the optic disc without hemorrhage.

(Choice G) Ocular migraine and migraine with visual aura can present with transient visual disturbances, with or
without headache. Ophthalmoscopic examination is normal in patients with migraine.

(Choice H) Thyroid eye disease (eg, Graves disease) can present with diplopia and proptosis; however, it is not
•t d "th d • k f f Ih h •
Block Time Elapsed: 00:02:53 ~ II;! rvo Cl @ 0
My Notebook Flashcards Feedback Suspend End Block

)4
"' https://t.me/USMLEWorldStep1 Show all X
= -
Item _12 of 30
Question Id: 11675
• - -
■ t7' Mark
• ,- - - - • --. • - - .. - .-
<:::
Previous
.,- 19r. ■
C>
Next
••• - • - - . • -.
~~
Full Screen
®
Tutorial
il
Lab Values
~-
Notes
....I
Calculator
,
Reverse Color
~
Text Zoom
©
Settings

hemorrhages.

Diagnosis of hypertensive retinal hemorrhage is usually confirmed by direct ophthalmoscopic evaluation. Other
findings of hypertensive reti nopathy include thickening of the arteriolar walls ("copper or silver wiri ng"),
compression of the associated veins (arteriovenous nicking), and small , white foci of retinal ischemia (cotton-wool
spots).

(Choices A and D) Central retinal artery occlusion is most commonly due to atherosclerosis, cardioembolic
disease, or vasculitis (eg, giant cell arteritis). It causes acute monocular vision loss. Examination can reveal a
cherry red spot at the macula due to diffuse retinal ischemia, but hemorrhage is not typically seen.

(Choices B and E) Smoking and hyperlipidemia can increase the risk of developing ischemic retinal and optic
vascular disease; however, these are not associated with an increased risk of retinal hemorrhage.

{Choice C) Closed-angle glaucoma is characterized by increased intraocular pressure from impaired aqueous
humor drainage. It causes acute headache, eye pain, vision loss, and an afferent pupillary defect.
Ophthalmoscopic examination reveals cupping of the optic disc without hemorrhage.

(Choice G) Ocular migraine and migraine with visual aura can present with transient visual disturbances, with or
without headache. Ophthalmoscopic examination is normal in patients with migraine.

(Choice H) Thyroid eye disease (eg, Graves disease) can present with diplopia and proptosis; however, it is not
associated w ith an increased risk of retinal hemorrhage.

Educational objective:
Severe hypertension in retinal precapillary arterioles causes endothelial disruption, leakage of plasma into the
arteriolar wall, and fibrinous necrosis. The necrotic vessels can then bleed into the nerve fiber layer, which can be
seen on examination as dot- or flame-shaped hemorrhages.

Block Time Elapsed: 00:02:53 ~ II;! rvo Cl @ 0


My Notebook Flashcards Feedback Suspend End Block

)4
"' https://t.me/USMLEWorldStep1 Show all X
--- Item 12 of 30
Question Id: 11675
■ ?'Mark <
Previous
C>
Next
r,
L -l
FullSaeen
®
Tutorial
il
Lab Values

Notes
~ I
Calculator
,
Reverse Color
~
Text Zoom Settings

+ • • • • • • • .... • • •

Thi ~
Exhibit Display Bl x

hyp Cotton-wool spots

spo

(Ch
dise
che

(Ch
vas

(Ch
hu
·~mon1y assoc,ated with 11ype<1ens,ve 0< Qal>etJC re«inopathy O,_,.

©_ Zoom In 0_ Zoom Out C Reset Cf;, New I lf-J Existing (!;I My Notebook

Block Time Elapsed: 00:02:53 ~ II;! rvo Cl @ 0


My Notebook Flashcards Feedback Suspend End Block

)4
"' https://t.me/USMLEWorldStep1 Show all X
= Item _12 of 30
Question Id: 11675
'.... • I
■ t7' Mark
,.- - ,- - . - e .e- ,. • 'IM-:1 - I.
<
Previous
- I
C>
Next
• - • ' ■• I I I. - • I I
~~
Full Saeen
®
Tutorial
il
Lab Values

Notes
....I
Calculator
,
Reverse Color
~
Text Zoom
©
Settings

fro
pre • Exhibit Display Bl x
nee
Central retinal artery occlusion
he

co
spo Cherry-red
spot
(Ch
dise
che
(Ch
vas
(Ch
hu

(Ch
with
C)UW0<ld

(Ch ©_ Zoom In 0_ Zoom Out C Reset Cf;, New I lf-J Existi ng (!;I My Notebook

Educational objective:

Block Time Elapsed: 00:02:53 ~ II;! rvo Cl @ 0


My Notebook Flashcards Feedback Suspend End Block

)4
"' https://t.me/USMLEWorldStep1 Show all X
= Item _12 of 30
Question Id: 11675
'.... • I
■ t7' Mark
,.- - ,- - . - e .e- ,. • 'IM-:1 - I.
<
Previous
- I
C>
Next
• - • ' ■• I I I. - • I I
~~
Full Saeen
®
Tutorial
il
Lab Values

Notes
....I
Calculator
,
Reverse Color
~
Text Zoom
©
Settings

fro
pre • Exhibit Display Bl x
nee
Glaucomatous optic atrophy (advanced)
he

co
spo

(Ch
dise
che

(Ch
vas

(Ch
hu

(Ch
with Cl/World

(Ch ©_ Zoom In 0_ Zoom Out C Reset Cf;, New I lf-J Existi ng (!;I My Notebook

Educational objective:

Block Time Elapsed: 00:02:53 ~ II;! rvo Cl @ 0


My Notebook Flashcards Feedback Suspend End Block

)4
"' https://t.me/USMLEWorldStep1 Show all X
= ltem_13of30
Question Id: 173
■ t7' Mark <:::
Previous
C>
Next
~~
Full Screen
®
Tutorial
il
Lab Values

Notes
....I
Calculator
,
Reverse Color
~
Text Zoom
©
Settings

A 63-year-old man comes to the office due to blurred vision in both eyes for the last 2 days . The blurriness is
worse when he reads but is not noticeable when he drives. The patient has no associated headache, double
vision, or weakness. However, he did start taking diphenhydramine several days ago for seasonal allergies. Past
medical history is unremarkable. Vital signs are normal. On physical examination , there is edema and clear
drainage affecting the nasal mucosa. Funduscopic examination is normal. This patient's visual symptoms are
most likely due to blockade of which of the following mediators?

0 A . Acetylcholine

0 8 . Histamine

0 C. Leukotrienes

0 D. Norepinephrine

0 E. Serotonin

Submit

Block Time Elapsed: 00:02:56 ~ II;! rvo Cl @ 0


My Notebook Flashcards Feedback Suspend End Block

)4
"' https://t.me/USMLEWorldStep1 Show all X
= Item _13 of 30
Question Id: 173
■ t7' Mark <:::
Previous
C>
Next
~~
Full Screen
®
Tutorial
il
Lab Values

Notes
....I
Calculator
,
Reverse Color
~
Text Zoom
©
Settings

A 63-year-old man comes to the office due to blurred vision in both eyes for the last 2 days . The blurriness is
worse when he reads but is not noticeable when he drives. The patient has no associated headache, double
vision, or weakness. However, he did start taking diphenhydramine several days ago for seasonal allergies. Past
medical history is unremarkable. Vital signs are normal. On physical examination, there is edema and clear
drainage affecting the nasal mucosa. Funduscopic examination is normal. This patient's visual symptoms are
most likely due to blockade of which of the following mediators?

✓ Q A . Acetylcholine (65%>)

X ® 8 . Histamine (27%)

0 C. Leukotrienes ( 1% )

0 D. Norepinephrine (4%)

0 E. Serotonin (0°/o)

Incorrect
Correct answer I 1,1 65% r i \ 05 secs
~ Answered correctly ~ Time Spent
A

Explanation

This patient has impaired visual accommodation due to anticholinergic (anti-muscarinic) effects of first-
generation antihistamines (eg, chlorpheniramine, diphenhydramine). Antihistamines are designed to relieve the
allergic symptoms caused by release of histamine from mast cells (eg, rhinorrhea, itchy eyes and nose, urticaria).
However, first-generation antihistamines can have prominent side effects due to blockade of other receptors,
Block Time Elapsed: 00:02:58 ~ II;! rvo Cl @ 0
My Notebook Flashcards Feedback Suspend End Block

)4
"' https://t.me/USMLEWorldStep1 Show all X
:= ltem_13of30
Question Id: 173
■ t7' Mark <:::
Previous
C>
Next
~~
Full Screen
®
Tutorial
il
Lab Values
~•
Notes Calculator
,
Reverse Color
~
Text Zoom
©
Settings

This patient has impaired visual accommodation due to anticholinergic (anti-muscarinic) effects of first-
generation antihistamines (eg, chlorpheniramine, diphenhydramine). Antihistamines are designed to relieve the
allergic symptoms caused by release of histamine from mast cells (eg, rhinorrhea, itchy eyes and nose, urticaria).
However, first-generation antihistamines can have prominent side effects due to blockade of other receptors,
including alpha-adrenergic, serotonergic, and cholinergic receptors.

The ciliary muscle attaches to the lens via the zonular fibers. Contraction of the muscle reduces tension on the
fibers, allowing the lens to become more spherical and increasing its refractive power. The ciliary muscle is under
parasympathetic control from the Edinger-Westphal nucleus/ciliary ganglion . Inhibition of this pathway by
antimuscarinic agents will limit accommodation and cause blurring of vision for close objects. The pupillary
sphincter also receives parasympathetic innervation from the Edinger-Westphal nucleus, and patients on
antimuscarinic agents may have mydriasis (pupillary dilation) as well.

{Choice B) First-generation antihistamines easily cross the blood-brain barrier into the central nervous system,
where they interfere with the normal neurotransmitter functions of histamine. Side effects can include drowsiness
and cognitive dysfunction .

(Choice C) Leukotrienes are inflammatory mediators that contribute to nasal, ocular, and respiratory symptoms in
patients with allergic disorders and asthma. Antihistamines do not interfere with leukotriene formation or action .

(Choice D) Alpha-adrenergic blockade causes peripheral vasodilation, resulting in hypotension and postural
dizziness.

(Choice E) Serotonin receptor blockade by antihistamines causes appetite stimulation and weight gain.

Educational objective:
In addition to blocking histamine receptors, first-generation antihistamines (eg, chlorpheniramine,
• • • • • • •
Block Time Elapsed: 00:02:58 ~ II;! rvo Cl @ 0
My Notebook Flashcards Feedback Suspend End Block

)4
"' https://t.me/USMLEWorldStep1 Show all X
= Item _13 of 30
Question Id: 173
■ t7' Mark <:::
Previous
C>
Next
~~
Full Screen
®
Tutorial
il
Lab Values
~-
Notes
....I
Calculator
,
Reverse Color
~
Text Zoom
©
Settings

However, first-generation antihistamines can have prominent side effects due to blockade of other receptors,
incl uding alpha-adrenergic, serotonergic, and cholinergic receptors.

The ciliary muscle attaches to the lens via the zonular fibers. Contraction of the muscle reduces tension on the
fibers, allowing the lens to become more spherical and increasing its refractive power. The ciliary muscle is under
parasympathetic control from the Edinger-Westphal nucleus/ciliary ganglion . Inhi bition of this pathway by
antimuscarinic agents will limit accommodation and cause blurring of vision for close objects. The pupillary
sphincter also receives parasympathetic innervation from the Edinger-Westphal nucleus, and patients on
antimuscarinic agents may have mydriasis (pupillary dilation) as well.

(Choice B) First-generation antihistamines easily cross the blood-brain barrier into the central nervous system,
where they interfere with the normal neurotransmitter functions of histamine. Side effects can include drowsiness
and cognitive dysfunction.

{Choice C) Leukotrienes are inflammatory mediators that contribute to nasal, ocular, and respiratory symptoms in
patients with allergic disorders and asthma. Antihistamines do not interfere with leukotriene formation or action.

(Choice D) Alpha-adrenergic blockade causes peripheral vasodilation, resulting in hypotension and postural
dizziness.

(Choice E) Serotonin receptor blockade by antihistamines causes appetite stimulation and weight gain.

Educational objective:
In addition to blocking histamine receptors, first-generation antihistamines (eg, chlorpheniramine,
diphenhydramine) have antimuscarinic, anti-alpha adrenergic, and anti-serotonergic properties. Anticholinergic
effects on the ocular ciliary muscles impair accommodation and cause blurring of vision for close objects.

Pharmacology Ophthalmology Antihistamines

Block Time Elapsed: 00:02:58 ~ II;! rvo Cl @ 0


My Notebook Flashcards Feedback Suspend End Block

)4
"' https://t.me/USMLEWorldStep1 Show all X
= ltem_13of30
Question Id: 173
■ t7' Mark <
Previous
C>
Next
~~
Full Saeen
®
Tutorial
il
Lab Values

Notes
....I
Calculator
,
Reverse Color
~
Text Zoom
©
Settings

C
Exhibit Display Bl x
A
Eye anatomy

Thi
gen
alle
Iris
Ho

The
Cornea - - - - -
fibe
par
Anterior eye
anti chamber

sphi
anti
Canal of
Schlernm
(Ch
whe
Vitreous body
@UNorkt
and
©_ Zoom In 0_ Zoom Out C Reset Cf;, New I lf-J Existi ng (!;I My Notebook
(Ch
pati
- - .
Block Time Elapsed: 00:02:58 ~ II;! rvo Cl @ 0
My Notebook Flashcards Feedback Suspend End Block

)4
"' https://t.me/USMLEWorldStep1 Show all X
=
-
ltem13of30
Question Id: 173
■ t:,,
\ Mark
<
Previous
C>
Next
~~
Full Screen
®
Tutorial
il
Lab Values

Notes
.... 1
Calculator
,
Reverse Color
~
Text Zoom
©
Settings

C Exhibit Display Bl X
A
Pupillary light reflex

This
gen
Ciliary ganglion -
alle
Ho

The
fibe Oculomotor
par nerve (CN Ill)

anti
sphi
anti

(Ch
whe 7
Lateral geniculate
and nucleus Edinger-Westphal
Pretectal nucleus nucleus
(Ch @UWortd

pati ©_ Zoom In 0_ Zoom Out C Reset Cf;, New I lf-J Existi ng (!;I My Notebook

)4
https://t.me/USMLEWorldStep1 Show all X
= ltem_14of30
Question Id: 15668
■ t7' Mark <:::
Previous
C>
Next
~~
Full Screen
®
Tutorial
il
Lab Values

Notes
....I
Calculator
,
Reverse Color
~
Text Zoom
©
Settings

A 68-year-old woman is evaluated for vision impairment. The patient has a decrease in both distant and near
vision and has had occasional floaters. She has had no pain or redness in her eyes, headache, focal weakness, or
sensory loss. Medical history is notable for type 2 diabetes mellitus, hypertension, and chronic kidney disease.
Blood pressure is 138/84 mm Hg and pulse is 76/min . The pupils are equal and reactive to light bilaterally.
Anterior chambers are clear, and there are no opacities of the cornea or lens. Funduscopic examination reveals
scattered retinal microaneurysms, dot-and-blot hemorrhages, and cotton-wool spots, as well as new blood vessel
formation . Which of the following contributed most to the pathogenesis of this patient's current ocular condition?

O A . Age-related degeneration of retinal pigment epithelium

0 B. Chronic hyperglycemia-induced microvascular injury

0 C. Ganglion cell death due to high intraocular pressure

0 D. Retinal arterial occlusion from atherosclerotic disease

0 E. Vascular injury from increased intraluminal pressure

Submit

Block Time Elapsed: 00:03:04 ~ II;! rvo Cl @ 0


My Notebook Flashcards Feedback Suspend End Block

)4
"' https://t.me/USMLEWorldStep1 Show all X
= ltem_14of30
Question Id: 15668
■ t7' Mark <:::
Previous
C>
Next
~~
Full Screen
®
Tutorial
il
Lab Values

Notes
....I
Calculator
,
Reverse Color
~
Text Zoom
©
Settings

A 68-year-old woman is evaluated for vision impairment. The patient has a decrease in both distant and near
vision and has had occasional floaters. She has had no pain or redness in her eyes, headache, focal weakness, or
sensory loss. Medical history is notable for type 2 diabetes mellitus, hypertension, and chronic kidney disease.
Blood pressure is 138/84 mm Hg and pulse is 76/min. The pupils are equal and reactive to light bilaterally.
Anterior chambers are clear, and there are no opacities of the cornea or lens. Funduscopic examination reveals
scattered retinal microaneurysms, dot-and-blot hemorrhages, and cotton-wool spots, as well as new blood vessel
formation. Which of the following contributed most to the pathogenesis of this patient's current ocular condition?

O A . Age-related degeneration of retinal pigment epithelium (3°/o)

✓ ® B. Chronic hyperglycemia-induced microvascular injury (72%)

0 C. Ganglion cell death due to high intraocular pressure (0%)

0 D. Retinal arterial occlusion from atherosclerotic disease (6%)

0 E. Vascular injury from increased intraluminal pressure (16%)

Correct I 1,1 72% fi"\ 09 secs


I.!!!!. Answered correctly 1..::.J Time Spent

Explanation

Diabetic retinopathy, proliferative (POR)

)4
https://t.me/USMLEWorldStep1 Show all X
Diabetic retinopathy, proliferative (PDR)

© UWol1d

©_ Zoom In 0_ Zoom Out C Reset Cf;, New I lf-J Existi ng (!;I My Notebook

)4
https://t.me/USMLEWorldStep1 Show all X
=
-
ltem_14of30
Question Id: 15668
■ t7' Mark <:::
Previous
C>
Next
~~
Full Screen
®
Tutorial
il
Lab Values
~•
Notes Calculator
,
Reverse Color
~
Text Zoom
©
Settings

This patient, with painless vision loss and retinal neovascularization, has proliferative diabetic retinopathy (DR).
DR is a result of chronic hyperglycemic injury to the small retinal vessels and is considered a microangiopathic
complication of diabetes (along with neuropathy and nephropathy). The prevalence of DR is proportionate to the
duration of diabetes and severity of hyperglycemia over time; tight control of diabetes is associated with a lower
long-term risk of DR.

DR is categorized as follows:

• Nonproliferative (early disease): The earliest morphologic changes include thickening of the basement
membrane and failure of the blood-retinal barrier. Increased permeability allows leakage of fluid into the retina
(macular edema), which can distort vision and leave behind lipid-rich deposits (hard exudates). Arteriolar
obstruction causes ischemic injury to the retina, which manifests as cotton-wool spots. Other findings include
microaneurysms and dot-blot hemorrhages (due to microaneurysm rupture).

• Proliferative (advanced disease): Progressive retinal ischemia stimulates production of angiogenic factors
(eg, vascular endothelial growth factor), leading to formation of new retinal vessels (neovascularization).
The new vessels are fragile and often extend into the adjacent vitreous. Traction from the vitreous can cause
detachment of the retina or laceration of the vessels, leading to acute hemorrhage and vision loss.

(Choice A) Age-related macular degeneration presents with slowly progressive central vision loss. It is due to
oxidative damage to the pigment epithelium and choriocapillaris with abnormal extracellular matrix formation
(drusen). In late-stage disease, patients may develop neovascularization ("wet" macular degeneration), but
associated findings include subretinal drusen and pigment abnormalities.

(Choice C) Open-angle glaucoma is a form of optic neuropathy, characterized by ganglion cell death in
association with elevated intraocular pressure. It causes insidious loss of peripheral vision ; examination findings
include enlargement of the retinal cup and a pale optic disc with thinning of the rim.

Block Time Elapsed: 00:03:07 ~ II;! rvo Cl @ 0


My Notebook Flashcards Feedback Suspend End Block

)4
"' https://t.me/USMLEWorldStep1 Show all X
= ltem_14of30
Question Id: 15668
■ t7' Mark <:::
Previous
C>
Next
~~
Full Screen
®
Tutorial
il
Lab Values
~-
Notes
....I
Calculator
,
Reverse Color
~
Text Zoom
©
Settings
microaneurysms and dot-blot hemorrhages (due to microaneurysm rupture).

• Proliferative (advanced disease): Progressive retinal ischemia stimulates production of angiogenic factors
(eg, vascular endothelial growth factor), leading to formation of new reti nal vessels (neovascularization).
The new vessels are fragile and often extend into the adjacent vitreous. Traction from the vitreous can cause
detachment of the retina or laceration of the vessels, leading to acute hemorrhage and vision loss.

(Choice A) Age-related macular degeneration presents with slowly progressive central vision loss. It is due to
oxidative damage to the pigment epithelium and choriocapillaris with abnormal extracellular matrix formation
(drusen). In late-stage disease, patients may develop neovascularization ("wet" macular degeneration), but
associated findings include subretinal drusen and pigment abnormalities.

{Choice C) Open-angle glaucoma is a form of optic neuropathy, characterized by ganglion cell death in
association with elevated intraocular pressure. It causes insidious loss of peripheral vision; examination findings
include enlargement of the retinal cup and a pale optic disc with thinning of the rim.

(Choice D) Atheroembolic retinal artery occlusion presents with acute monocular vision loss. Examination
findings include retinal pallor and a cherry red spot at the macula.

(Choice E) Hypertensive retinopathy can manifest w ith retinal hemorrhages, cotton-wool spots, and hard
exudates. However, the thickened and stiffened arterial walls are typically visible as arteriolar narrowing and
impingement on the veins where they are crossed by arteries (arteriovenous nicking).

Educational objective:
Chronic hyperglycemia in patients w ith diabetes can lead to increased permeability and arteriolar obstruction in
retinal vessels. The resulting ischemia stimulates production of vascular endothelial growth factor and other
angiogenic factors, leading to neovascularization (proliferative diabetic retinopathy). Complications include retinal
hemorrhage, retinal detachment, and vision loss.

Block Time Elapsed: 00:03:07 ~ II;! rvo Cl @ 0


My Notebook Flashcards Feedback Suspend End Block

)4
"' https://t.me/USMLEWorldStep1 Show all X
= ltem_14of30
Question Id: 15668
■ t7' Mark <
Previous
C>
Next
~~
Full Saeen
®
Tutorial
il
Lab Values

Notes
....I
Calculator
,
Reverse Color
~
Text Zoom
©
Settings

e uwortd

This Exhibit Display Bl X

DR
Cotton-wool spots
com
dur
long

DR

(Ch
oxid
"Commonly assodale<! w,1n hypertensive «01at>e11e retinopalhy C,._,.
(dru
©_ Zoom In 0_ Zoom Out C Reset C:::, New I lf-J Existi ng (!;I My Notebook

association with elevated intraocular pressure. It causes insidious loss of peripheral vision ; examination findings
Block Time Elapsed: 00:03:07 ~ II;! rvo Cl @ 0
My Notebook Flashcards Feedback Suspend End Block

)4
"' https://t.me/USMLEWorldStep1 Show all X
= ltem_14of30
Question Id: 15668
■ t7' Mark <
Previous
C>
Next
~~
Full Saeen
®
Tutorial
il
Lab Values

Notes
....I
Calculator
,
Reverse Color
~
Text Zoom
©
Settings

e uwor1<1

Thi ~ Exhibit Display Bl x


DR
Diabetic retinopathy, nonproliferative

DR


Optic disc ----'►

• Hard exudate

!
(Ch
oxid
(dru
ass ©_ Zoom In 0_ Zoom Out C Reset Cf;, New I lf-J Existi ng (!;I My Notebook

(Ch
association with elevated intraocular pressure. It causes insidious loss of peripheral vision ; examination findings
Block Time Elapsed: 00:03:07 ~ II;! rvo Cl @ 0
My Notebook Flashcards Feedback Suspend End Block

)4
"' https://t.me/USMLEWorldStep1 Show all X
= ltem_14of30
Question Id: 15668
■ t7' Mark
-
<
Previous
C>
Next
-
~~
Full Saeen
®
Tutorial
il
Lab Values

Notes
....I
Calculator
,
Reverse Color
~
Text Zoom
©
Settings

Ion
Age-related macular degeneration (AMD)
DR

(Ch
oxid
(dru
ass

(Ch
@lmortd
ass
©_ Zoom In 0_ Zoom Out C Reset Cf;, New I lf-J Existi ng (!;I My Notebook
incl

(Ch ~ .,.. - - -- ... . - -- - - - . . -- .. .. .. .. - - ,.,_ .. . .-


•• •• • •- - •- •- • -•• - •- a -• •• - • · ••- -

Block Time Elapsed: 00:03:07 ~ II;! rvo Cl @ 0


My Notebook Flashcards Feedback Suspend End Block

)4
"' https://t.me/USMLEWorldStep1 Show all X
= ltem_14of30
Question Id: 15668
■ t7' Mark <
Previous
C>
Next
~~
Full Saeen
®
Tutorial
il
Lab Values

Notes
....I
Calculator
,
Reverse Color
~
Text Zoom
©
Settings

DR
Exhibit Display Bl x

Optic disc in glaucoma

Normal Open-angle glaucoma

(Ch
oxid
(dru
ass

(Ch • Enlarged cup with cup/disc ratio >0.6


• Cup/disc ratio <0.5 • Increase in cup size over time
ass
• Clear disc rim • Thinning of disc rim
incl • Pale disc (optic nerve atrophy)
(Ch euwortd
findi ©_ Zoom In 0_ Zoom Out C Reset Cf;, New I lf-J Existi ng (!;I My Notebook
(Ch
exu

Block Time Elapsed: 00:03:07 ~ II;! rvo Cl @ 0


My Notebook Flashcards Feedback Suspend End Block

)4
"' https://t.me/USMLEWorldStep1 Show all X
= ltem_14of30
Question Id: 15668
■ t7' Mark <
Previous
C>
Next
~~
Full Saeen
®
Tutorial
i l
Lab Values
~
Notes
· ....I
Calculator
,
Reverse Color
~
Text Zoom
©
Settings
membrane and failure of the blood-retinal barrier. Increased permeability allows leakage of fluid into the retina

Exhibit Display Bl x

Central retinal artery occlusion

Optic cup
M acula
(Ch
oxid i
(dru
ass

(Ch
ass
Cherry-red
spot
t t
Optic disc
incl Retinal
whitening
(Ch
findi

(Ch
exu @UW<wld

impi ©_ Zoom In 0_ Zoom Out C Reset Cf;, New I lf-J Existi ng (!;I My Notebook

Ed
Chronic hyperglycemia in patients with diabetes can lead to increased permeability and arteriolar obstruction in

Block Time Elapsed: 00:03:07 ~ II;! rvo Cl @ 0


My Notebook Flashcards Feedback Suspend End Block

)4
"' https://t.me/USMLEWorldStep1 Show all X
= ltem_14of30
Question Id: 15668
■ t7' Mark <
Previous
C>
Next
~~
Full Saeen
®
Tutorial
il
Lab Values

Notes
....I
Calculator
,
Reverse Color
~
Text Zoom
©
Settings
membrane and failure of the blood-retinal barrier. Increased permeability allows leakage of fluid into the retina

Exhibit Display Bl x

Hypertensive retinopathy

Arteriovenous
nicking

(Ch
oxid
\
(dru
ass

(Ch
t
Arteriolar
narrowing
ass
incl

(Ch
Arteriovenous
findi nicking ---1►

(Ch
CUWorld
exu
impi ©_ Zoom In 0_ Zoom Out C Reset Cf;, New I lf-J Existi ng (!;I My Notebook

Ed
Chronic hyperglycemia in patients with diabetes can lead to increased permeability and arteriolar obstruction in

Block Time Elapsed: 00:03:07 ~ II;! rvo Cl @ 0


My Notebook Flashcards Feedback Suspend End Block

)4
"' https://t.me/USMLEWorldStep1 Show all X
= ltem_15of30
Question Id: 862
■ t7' Mark <
Previous
C>
Next
~~
Full Screen
®
Tutorial
il
Lab Values

Notes
....I
Calculator
,
Reverse Color
~
Text Zoom
©
Settings

A 50-year-old woman who has had mild myopia for decades comes to the office for a routine physical examination .
The patient has no chronic medical conditions and eats a healthy diet consisting of a variety of organically grown
foods. She is excited because she rarely has to wear glasses or contacts anymore. Her visual acuity at 20 feet is
20/30 in both eyes without wearing her glasses. Which of the following conditions has the same etiology as the
changes causing this patient's visual improvement?

0 A . Actinic keratosis (10%)

X ® 8 . Angle-closure glaucoma (14%)

0 C. Coronary atherosclerosis (7%)

0 D. Femoral head osteonecrosis (1%>)

✓ Q E. Skin wrinkles (65°/o)

Incorrect
Correct answer I 1,1 65% fi"\ 35 secs
L!!.!.!. Answered correctly \.:.JTimeSpent
E

Explanation

Interaction between presbyopia & myopia

Image focuses
in front of retina

)4
https://t.me/USMLEWorldStep1 Show all X
Interaction betwe<:>n presbyopia & myopia

Image focuses
in front of retin~

Myopia

E
C
E Image focusM
behind re.tina

E Presbyopia
I
E
Impairment
of lens ac:x:ommoclation

Myopia & presbyopia

E
©UWorld

©_ Zoom In 0_ Zoom Out C Reset Cf;, New I lf-J Existi ng (!;I My Notebook

)4
https://t.me/USMLEWorldStep1 Show all X
:= ltem_15of30
Question Id: 862
■ t7' Mark <:::
Previous
C>
Next
~~
Full Screen
®
Tutorial
il
Lab Values
~•
Notes Calculator
,
Reverse Color
~
Text Zoom
©
Settings

The improvement in this patient's mild myopia is likely due to age-related changes. Normally, in accommodation ,
when focusing on near objects (eg, reading), ciliary muscle contraction relaxes the zonular fibers, allowing the lens
to become more convex so the image focuses on the retina.

Starting around age 40-50, almost all individuals develop an inability to focus on near objects. In this condition,
called presbyopia, progressive denaturation of lens proteins and changes in lens curvature cause the lens to
become less elastic and lose its accommodating power. In most patients, this causes the image of near objects to
focus behind the retina resulting in difficulty reading fine print and the need to hold objects farther away in order to
see them clearly. However, in patients with mild myopia (increased axial length with an image focused in front of
the retina), presbyopia can compensate for myopia by displacing the image backward, so that it focuses on the
retina. These patients often note improvement in visual acuity with age as presbyopia develops.

Skin rhytides (wrinkles) are also directly caused by chronological aging (with a contribution from the harmful
effects of ultraviolet light). With aging, skin becomes atrophic and fragile, with reduced elasticity and subdermal
fat. There is dermal and epidermal thinning, flattening of the dermoepidermal junction, a decreased number of
fibroblasts, and reduced synthesis and increased breakdown of collagen and elastin.

(Choices A, B, and C) Although age is a risk factor for actinic keratoses, angle-closure glaucoma, and
atherosclerosis, age-related changes themselves do not cause these conditions. Actinic keratoses (ie,
premalignant lesions increasing squamous cell carcinoma risk) are rough, scaly, erythematous papules on sun-
exposed areas that result from ultraviolet damage. Angle-closure glaucoma occurs due to impaired aqueous
humor flow. Atherosclerosis is primarily attributable to hyperlipidemia, hypertension , and genetic (familial)
predisposition.

(Choice D) Femoral head osteonecrosis is associated with vasculitis (lupus), corticosteroid therapy, sickle cell
disease, and alcohol use disorder. It is not an age-related condition and can occur in children (eg, Legg-Calve-
•----- - -----
Block Time Elapsed: 00:03:42 ~ II;! rvo Cl @ 0
My Notebook Flashcards Feedback Suspend End Block

)4
"' https://t.me/USMLEWorldStep1 Show all X
:= ltem_15of30
Question Id: 862
■ t7' Mark <:::
Previous
C>
Next
~~
Full Screen
®
Tutorial
il
Lab Values
~•
Notes Calculator
,
Reverse Color
~
Text Zoom
©
Settings

focus behind the retina resulting in difficulty reading fine print and the need to hold objects farther away in order to
see them clearly. However, in patients with mild myopia (increased axial length with an image focused in front of
the retina), presbyopia can compensate for myopia by displacing the image backward, so that it focuses on the
retina. These patients often note improvement in visual acuity with age as presbyopia develops.

Skin rhytides (wrinkles) are also directly caused by chronological aging (with a contribution from the harmful
effects of ultraviolet light). With aging, skin becomes atrophic and fragile, with reduced elasticity and subdermal
fat. There is dermal and epidermal thinning, flattening of the dermoepidermal junction, a decreased number of
fibroblasts, and reduced synthesis and increased breakdown of collagen and elastin .

(Choices A, B, and C) Although age is a risk factor for actinic keratoses, angle-closure glaucoma, and
atherosclerosis, age-related changes themselves do not cause these conditions. Actinic keratoses (ie,
premalignant lesions increasing squamous cell carcinoma risk) are rough, scaly, erythematous papules on sun-
exposed areas that result from ultraviolet damage. Angle-closure glaucoma occurs due to impaired aqueous
humor flow. Atherosclerosis is primarily attributable to hyperlipidemia, hypertension , and genetic (familial)
predisposition .

(Choice D) Femoral head osteonecrosis is associated with vasculitis (lupus), corticosteroid therapy, sickle cell
disease, and alcohol use disorder. It is not an age-related condition and can occur in children (eg, Legg-Calve-
Perthes disease).

Educational objective:
Presbyopia and skin wrinkles are age-related changes. Presbyopia occurs due to denaturation of structural
proteins within the lens, leading to loss of lens elasticity which can result in improved vision in patients with mild
myopia. Decreased synthesis and increased breakdown of collagen and elastin contribute to the development of
skin wrinkles.

Block Time Elapsed: 00:03:42 ~ II;! rvo Cl @ 0


My Notebook Flashcards Feedback Suspend End Block

)4
"' https://t.me/USMLEWorldStep1 Show all X
The Changes associated with normal aging

whe System Physiologic effects Medical com plications

to b Cardiovascular
• !Maximum heart rate • !Exercise capacity
• Thickening & calcification of the aorta • Systolic hypertension

• t Gastric acid reflux • Gastroesophageal reflux disease


Gastrointestinal
• ! Colonic motility • Chronic constipation

Urinary
• ! Abi lity to concentrate uri ne • Dehydration
• Impaired emptying of the bladder • Urinary tract infections & incontinence

Immune • ! Numbers of B & T cells • t Respiratory & nosocomial infections, malignancies & autoimmune cond itions
see
the • ! Muscle mass • t Risk of falls
Musculoskeletal
• t Bone mineral loss • t Fracture risk
reti
• Skin atrophy • t Skin fragility
lntegumentary
• ! Subdermal fat • Skin wrinkling

effe • Harden ing of the ocular lens • Presbyopia


Sensory
• Deterioration of auditory pathway • Presbycusis
fat.
fibr

(Ch

q, New I q,J Existing l!;J My Notebook

predisposition.

Block Time Elapsed: 00:03:42 ~ II;! rvo Cl @ 0


My Notebook Flashcards Feedback Suspend End Block

)4
"' https://t.me/USMLEWorldStep1 Show all X
The Ocular accommodation and presbyopia

whe
to b
Relaxed
y-
--
Accommodated
__
..,....... Presbyopia
__
..,.......
Ciliary
muscle

Lens

see
the
reti

effe Zonular
fibers
fat.
fibr

(Ch Ciliary muscle relaxes, Ciliary muscle contracts, Hardened lens is unable
zonular fibers tighten & relaxing the zonular fibers & to thicken upon ciliary
cause flattening of the lens. causing thickening of the lens muscle contraction
C)UWotld

©_ Zoom In 0_ Zoom Out C Reset Cf;, New I lf-J Existing (!;I My Notebook

predisposition.

Block Time Elapsed: 00:03:42 ~ II;! rvo Cl @ 0


My Notebook Flashcards Feedback Suspend End Block

)4
"' https://t.me/USMLEWorldStep1 Show all X
The
whe
to b
Wrinkles
Young skin Aging skin

see
the Dermis
reti
Reticular Elastin

effe
Subcutaneous fat
fat.
fibr CIUWortd

(Ch

©_ Zoom In 0_ Zoom Out C Reset Cf;, New I lf-J Existi ng (!;I My Notebook

predisposition.

Block Time Elapsed: 00:03:42 ~ II;! rvo Cl @ 0


My Notebook Flashcards Feedback Suspend End Block

)4
"' https://t.me/USMLEWorldStep1 Show all X
= ltem_16of30
Question Id: 18588
■ t7' Mark <:::
Previous
C>
Next
~~
Full Screen
®
Tutorial
il
Lab Values

Notes
....I
Calculator
,
Reverse Color
~
Text Zoom
©
Settings

A 58-year-old man comes to the emergency department due to vision disturbances. Over the past several weeks,
the patient has had intermittent episodes in which he sees everything tinted with blue. His vision returned to
normal within a few hours each time. When he woke up today, vision in the left eye was blurry. He has had no eye
pain, conjunctiva! redness, or headache. The patient has tingling and numbness in the feet due to diabetic
neuropathy, which is unchanged, and reports no other focal weakness or sensory loss. Medical history is notable
for hypertension, type 2 diabetes mellitus, hyperlipidemia, and erectile dysfunction. Vital signs are within normal
limits. Physical examination shows an afferent pupillary defect, decreased visual acuity, and optic disc edema in
the left eye. Discontinuing which of the following medications may improve this patient's vision?

0 A . Amitriptyline

0 8 . Atorvastatin

0 C. Canagliflozin

0 D. Metformin

0 E. Ramipril

0 F. Sildenafil

Submit

Block Time Elapsed: 00:03:43 ~ II;! rvo Cl @ 0


My Notebook Flashcards Feedback Suspend End Block

)4
"' https://t.me/USMLEWorldStep1 Show all X
= ltem_16of30
Question Id: 18588
■ t7' Mark <:::
Previous
C>
Next
~~
Full Screen
®
Tutorial
il
Lab Values

Notes
....I
Calculator
,
Reverse Color
~
Text Zoom
©
Settings

A 58-year-old man comes to the emergency department due to vision disturbances. Over the past several weeks,
the patient has had intermittent episodes in which he sees everything tinted with blue. His vision returned to
normal within a few hours each time. When he woke up today, vision in the left eye was blurry. He has had no eye
pain, conjunctiva! redness, or headache. The patient has tingling and numbness in the feet due to diabetic
neuropathy, which is unchanged, and reports no other focal weakness or sensory loss. Medical history is notable
for hypertension, type 2 diabetes mellitus, hyperlipidemia, and erectile dysfunction. Vital signs are within normal
limits. Physical examination shows an afferent pupillary defect, decreased visual acuity, and optic disc edema in
the left eye. Discontinuing which of the following medications may improve this patient's vision?

0 A . Amitriptyline (23%>)

X ® 8 . Atorvastatin (1%)

0 C. Canagliflozin (7%)

0 D. Metformin (2%)

0 E. Ramipril (3%)

✓ Q F. Sildenafil (62%)

Incorrect
Correct answer I 1,1 62% fl\ 04 secs
~ Answered correctly \.::.;Time Spent
F

Explanation

Block Time Elapsed: 00:03:46 ~ II;! rvo Cl @ 0


My Notebook Flashcards Feedback Suspend End Block

)4
"' https://t.me/USMLEWorldStep1 Show all X
= Item _16 of 30
Question Id: 18588
■ t7' Mark <:::
Previous
C>
Next
~~
Full Screen
®
Tutorial
il
Lab Values
~-
Notes
....I
Calculator
,
Reverse Color
~
Text Zoom
©
Settings

Adverse effects of phosphodiesterase 5 inhibitors

Cardiovascular • Hypotension (especially with nitrates, alpha blockers)

• Blue discoloration of vision


Ocular
• Nonarteritic anterior ischemic optic neuropathy

Genitourinary • Priapism

• Flushing
Other • Headache
• Hearing loss

Normal male erectile function involves interplay between psychological and physical stimulation and the vascular,
neurologic, and hormonal systems. On a vascular level, penile erection is induced by increased blood flow to the
corpora cavernosa and corpus spongiosum accompanied by decreased outflow. Endothelial factors required for
erection include formation of nitric oxide, which induces formation of cyclic GMP (cGMP), which in turn mediates
vascular smooth muscle relaxation. cGMP is inactivated by cGMP phosphodiesterase (PDE) (ie, PDES), which
cleaves cGMP and terminates penile erection .

PDES inhibitors (eg, sildenafil, tadalafil) are widely used, first-line agents for treatment of erectile dysfunction
(ED) . PDES inhibitors increase activity of cGMP in the penile corpora, leading to greater and more prolonged
tumescence in men with ED.

However, PDES inhibitors can also inhibit PDE6 in the retina, which is involved in color vision. Patients experience
this effect as a transient bluish discoloration to vision. Less common ocular effects of PDES inhibitors include
nonarteritic anterior ischemic optic neuropathy, which presents with sudden monocular vision loss associated
.. · - -· - - - -- - -- - -- .. -- - ---··
Block Time Elapsed: 00:03:46 ~ II;! rvo Cl @ 0
My Notebook Flashcards Feedback Suspend End Block

)4
"' https://t.me/USMLEWorldStep1 Show all X
= Item _16 of 30
Question Id: 18588
■ t7' Mark
-----------------------------
<:::
Previous
C>
Next
~~
Full Screen
®
Tutorial
il
Lab Values
~-
Notes
....I
Calculator
,
Reverse Color
~
Text Zoom
©
Settings

Normal male erectile function involves interplay between psychological and physical stimulation and the vascular,
neurologic, and hormonal systems. On a vascular level, penile erection is induced by increased blood flow to the
corpora cavernosa and corpus spongiosum accompanied by decreased outflow. Endothelial factors required for
erection include formation of nitric oxide, which induces formation of cyclic GMP (cGMP), which in turn mediates
vascular smooth muscle relaxation. cGMP is inactivated by cGMP phosphodiesterase (PDE) (ie, PDES), which
cleaves cGMP and terminates penile erection.

PDES inhibitors (eg, sildenafil, tadalafil) are widely used, first-line agents for treatment of erectile dysfunction
(ED) . PDES inhibitors increase activity of cGMP in the penile corpora, leading to greater and more prolonged
tumescence in men with ED .

However, PDES inhibitors can also inhibit PDE6 in the retina, which is involved in color vision. Patients experience
this effect as a transient bluish discoloration to vision. Less common ocular effects of PDES inhibitors include
nonarteritic anterior ischemic optic neuropathy, which presents with sudden monocular vision loss associated
with an afferent pupillary defect, decreased visual acuity, and optic disc edema.

(Choice A) The anticholinergic effects of tricyclic antidepressants (eg, amitriptyline) can cause dry eye, blurred
vision, and triggering of angle-closure glaucoma (ie, ocular pain, headache, unilateral vision loss). Discoloration of
vision is not typical.

(Choice B) Important side effects of statins (eg, atorvastatin) include myopathy and hepatic toxicity. Statins may
have a small effect on the risk of cataract but otherwise have few ocular manifestations.

(Choice C) Sodium-glucose cotransporter 2 inhibitors (eg, canagliflozin) induce increased renal excretion of
glucose and water. Most of the side effects (eg, hypotension, urinary tract infection) are related to glucosuria and
fluid shifts; visual symptoms are not common.

I o • ■ ii - • II I II• • 1111 • I • - • • • I - I• • - - I - - • I• - • • • I - • • • • •

Block Time Elapsed: 00:03:46 ~ II;! rvo Cl @ 0


My Notebook Flashcards Feedback Suspend End Block

)4
"' https://t.me/USMLEWorldStep1 Show all X
=
-
ltem_16of30
Question Id: 18588
■ t7' Mark <:::
Previous
C>
Next
~~
Full Screen
®
Tutorial
il
Lab Values
~•
Notes Calculator
,
Reverse Color
~
Text Zoom
©
Settings

However, PDE5 inhibitors can also inhibit PDE6 in the retina, which is involved in color vision . Patients experience
this effect as a transient bluish discoloration to vision . Less common ocular effects of PDE5 inhibitors include
nonarteritic anterior ischemic optic neuropathy, which presents with sudden monocular vision loss associated
with an afferent pupillary defect, decreased visual acuity, and optic disc edema.

(Choice A) The anticholinergic effects of tricyclic antidepressants (eg, amitriptyline) can cause dry eye, blurred
vision , and triggering of angle-closure glaucoma (ie, ocular pain, headache, unilateral vision loss). Discoloration of
vision is not typical.

(Choice B) Important side effects of statins (eg, atorvastatin) include myopathy and hepatic toxicity. Statins may
have a small effect on the risk of cataract but otherwise have few ocular manifestations.

(Choice C) Sodium-glucose cotransporter 2 inhibitors (eg, canagliflozin) induce increased renal excretion of
glucose and water. Most of the side effects (eg, hypotension, urinary tract infection) are related to glucosuria and
fluid shifts; visual symptoms are not common .

(Choice D) Metformin most commonly causes gastrointestinal side effects (eg, nausea, diarrhea) . Lactic acidosis
is a potentially serious but rare complication . Visual side effects are not a significant concern.

(Choice E) Side effects of ACE inhibitors (eg, ramipril) are primarily related to renal function (eg , hyperkalemia,
acute kidney injury) or blood pressure (eg, dizziness, hypotension). Patients may also develop a dry cough due to
an increase in circulating bradykinin.

Educational objective:
Phosphodiesterase 5 inhibitors (eg, sildenafil , tadalafil) can cause a transient bluish discoloration to vision. Less
common ocular effects include sudden monocular vision loss due to nonarteritic anterior ischemic optic
neuropathy; findings include an afferent pupillary defect, decreased visual acuity, and optic disc edema.

Block Time Elapsed: 00:03:46 ~ II;! rvo Cl @ 0


My Notebook Flashcards Feedback Suspend End Block

)4
"' https://t.me/USMLEWorldStep1 Show all X
= ltem_17of30
Question Id: 12257
■ t7' Mark <:::
Previous
C>
Next
~~
Full Screen
®
Tutorial
il
Lab Values

Notes
....I
Calculator
,
Reverse Color
~
Text Zoom
©
Settings

A 31 -year-old woman comes to the office due to transient visual changes that are characterized by "loss of vision
for a minute" and commonly precipitated by bending forward or lifting objects. During the last 3 months, she has
had persistent headaches that she attributes to migraines. The patient takes over-the-counter analgesics as
needed . Blood pressure is 140/90 mm Hg and pulse is 72/min and regular. BM I is 32.4 kg/m2 . Funduscopic
findings of the left eye are shown in the image below, with similar findings in the right eye.

)4
https://t.me/USMLEWorldStep1 Show all X
=
-
ltem17of30
Question Id: 12257
■ t:,,
\ Mark
<
Previous
C>
Next
~~
Full Saeen
®
Tutorial
i l
Lab Values
~ ·
Notes
....1
Calculator
,
Reverse Color
~
Text Zoom
©
Settings

A3
for
had
nee
findi

©_ Zoom In 0_ Zoom Out C Reset Cf;, New I lf-J Existi ng (!;I My Notebook

)4
https://t.me/USMLEWorldStep1 Show all X
Which of the following is the most likely cause of the observed funduscopic findings in this patient?

0 A . Choroidal inflammation

0 8 . Elevated intracranial pressure

0 C. Elevated intraocular pressure

0 D. Optic nerve inflammation

0 E. Retinal ischemia

Submit

Block Time Elapsed: 00:04:00 ~ II;! rvo Cl @ 0


My Notebook Flashcards Feedback Suspend End Block

)4
"' https://t.me/USMLEWorldStep1 Show all X
Which of the following is the most likely cause of the observed funduscopic findings in this patient?

0 A . Choroidal inflammation (0°/o)

✓ (!) 8 . Elevated intracranial pressure (70%)

0 C. Elevated intraocular pressure (17%)

0 D. Optic nerve inflammation (3%)

0 E. Retinal ischemia (8%)

Correct I 1,1 70% ti\ 20 secs


l!!!!. Answered correctly \::.J Time Spent

Block Time Elapsed: 00:04:06 ~ II;! rvo Cl @ 0


My Notebook Flashcards Feedback Suspend End Block

)4
"' https://t.me/USMLEWorldStep1 Show all X
= ltem_17of30
Question Id: 12257
■ t7' Mark <
Previous
C>
Next
~~
Full Saeen
®
Tutorial
il
Lab Values

Notes
....I
Calculator
,
Reverse Color
~
Text Zoom
©
Settings

1 Exhibit Display Bl x

E
Normal optic disc Papilledema

t
Blurred optic
disc margins

CUWortd

Thi
©_ Zoom In 0_ Zoom Out C Reset Cf;, New I lf-J Existi ng (!;I My Notebook

Thi
and transient visual disturbances related to impaired cerebral venous outflow and elevated intracranial

Block Time Elapsed: 00:04:06 ~ II;! rvo Cl @ 0


My Notebook Flashcards Feedback Suspend End Block

)4
"' https://t.me/USMLEWorldStep1 Show all X
:= ltem_17of30
Question Id: 12257
■ t7' Mark <:::
Previous
C>
Next
~~
Full Screen
®
Tutorial
il
Lab Values
~•
Notes Calculator
,
Reverse Color
~
Text Zoom
©
Settings
.,1,,1~,;

This patient's clinical presentation is consistent with idiopathic intracranial hypertension (pseudotumor cerebri).
This condition typically presents in young obese women with daily headache, bilaterally symmetric papilledema,
and transient visual disturbances related to impaired cerebral venous outflow and elevated intracranial
pressure. Symptoms characteristically worsen during the Valsalva maneuver (eg, bending down, coughing)
because intracranial pressure increases.

Increased intracranial pressure is transmitted through the cerebrospinal fluid in the subarachnoid space, which is
continuous with the optic nerve sheath . This buildup of pressure compresses the optic nerves externally, which in
turn impairs axoplasmic flow within the optic nerves, causing bilateral optic disc edema (papilledema).
Funduscopy shows elevation of the optic disc with blurred disc margins.

(Choice A) Choroidal inflammation is characteristic of posterior uveitis, which typically presents with painless
vision loss and floaters. Ophthalmic examination may show inflammation/leukocytes in the vitreous humor. Uveitis
is often associated with systemic inflammatory disorders (eg , inflammatory bowel disease, ankylosing spondylitis).

(Choice C) An acute increase in intraocular pressure is characteristic of angle-closure glaucoma, which typically
presents with painful monocular vision loss, headache, vomiting, and conjunctiva! injection with a poorly reactive
middilated pupil. Chronic increases in intraocular pressure can cause optic disc cupping due to atrophy of the optic
nerve head. Glaucoma usually affects older individuals.

(Choice D) Optic neuritis is frequen tly associated with multiple sclerosis, a demyelinating CNS disease that
typically affects women age 15-50. Although swelling of the optic disc can be present on funduscopy, optic neuritis
usually causes sustained monocular vision loss over several weeks with painful eye movement rather than daily
headaches with transient bilateral vision loss.

... .......
Block Time Elapsed: 00:04:06
. - .... . . ...... ·- ......
~
- - - - .. - ...... -. .. .....
(Choice E) Retinal ischemia can manifest with painless transient monocular vision loss (amaurosis fugax) and is

II;! rvo Cl @ 0
My Notebook Flashcards Feedback Suspend End Block

)4
"' https://t.me/USMLEWorldStep1 Show all X
:= ltem_17of30
Question Id: 12257
■ t7' Mark <::: C>
Previous
Next
~~
Full Screen
®
Tutorial
il
Lab Values
~•
Notes Calculator
,
Reverse Color
~
Text Zoom
©
Settings
-- ·-·- - -- - - - - -- - -- ... - . ---· - ·- - ----- - -- - - -- - - ... -
turn impairs axoplasmic flow within the optic nerves, causing bilateral optic disc edema (papilledema).
Funduscopy shows elevation of the optic disc with blurred disc margins.

(Choice A) Choroidal inflammation is characteristic of posterior uveitis, which typically presents with painless
vision loss and floaters. Ophthalmic examination may show inflammation/leukocytes in the vitreous humor. Uveitis
is often associated with systemic inflammatory disorders (eg , inflammatory bowel disease, ankylosing spondylitis).

(Choice C) An acute increase in intraocular pressure is characteristic of angle-closure glaucoma, which typically
presents with painful monocular vision loss, headache, vomiting, and conjunctiva! injection with a poorly reactive
middilated pupil. Chronic increases in intraocular pressure can cause optic disc cupping due to atrophy of the optic
nerve head. Glaucoma usually affects older individuals.

(Choice D) Optic neuritis is frequently associated w ith multiple sclerosis, a demyelinating CNS disease that
typically affects women age 15-50. Although swelling of the optic disc can be present on funduscopy, optic neuritis
usually causes sustained monocular vision loss over several weeks with painful eye movement rather than daily
headaches with transient bilateral vision loss.

(Choice E) Retinal ischemia can manifest with painless transient monocular vision loss (amaurosis fu gax) and is
most commonly caused by atherosclerotic emboli originating from the ipsilateral carotid artery. Funduscopy may
show embolic plaques and retinal whitening caused by ischemia. This condition typically affects older patients with
vascular risk factors (eg, hypertension , hyperlipidemia, diabetes mellitus).

Educational objective:
Idiopathic intracranial hypertension (pseudotumor cerebri) presents in young obese women with daily headache
(which worsens during the Valsalva maneuver), bilaterally symmetric papilledema, and transient visual
disturbances. Increased intracranial pressure compresses the optic nerves, resulting in impaired axoplasmic flow
and optic disc edema.

Block Time Elapsed: 00:04:06 ~ II;! rvo Cl @ 0


My Notebook Flashcards Feedback Suspend End Block

)4
"' https://t.me/USMLEWorldStep1 Show all X
= ltem_17of30
Question Id: 12257
■ t7' Mark <
Previous
C>
Next
~~
Full Saeen
®
Tutorial
il
Lab Values

Notes
....I
Calculator
,
Reverse Color
~
Text Zoom
©
Settings
• •
VISI •
. Exhibit Display Bl X
IS 0

(Ch Glaucomatous optic atrophy (advanced)

(Ch
typi
usu
hea

(Ch
mos
sho
vas

Edu
ldio
(whi
dist
Cl/World
and
©_ Zoom In 0_ Zoom Out C Reset C:::, New I lf-J Existi ng (!;I My Notebook
Ret
• - - •• - - • •• • -■- .. .. -■ ........ - • -

. - .. ... - -- . - - -. . . .. ···- .......... .


Block Time Elapsed: 00:04:06 ~ II;! rvo Cl @ 0
My Notebook Flashcards Feedback Suspend End Block

)4
"' https://t.me/USMLEWorldStep1 Show all X
= ltem_18of30
Question Id: 360
■ t7' Mark <:::
Previous
C>
Next
~~
Full Screen
®
Tutorial
il
Lab Values

Notes
....I
Calculator
,
Reverse Color
~
Text Zoom
©
Settings

A 64-year-old man comes to the emergency department due to painless loss of vision in the right eye. The
patient's symptoms started suddenly several hours ago and have persisted . He has had no trauma or any similar
episode in the past. He has a history of coronary artery disease, atrial fibrillation, and type 2 diabetes mellitus.
Blood pressure is 144/86 mm Hg and pulse is 92/min . Cardiovascular examination reveals irregularly irregular
rhythm with no murmurs, no peripheral edema, and slightly diminished pedal pulses in both feet symmetrically.
Visual acuity in the right eye is hand motion only and normal in the left eye. Funduscopic examination of the right
eye is shown in the image below.

)4
https://t.me/USMLEWorldStep1 Show all X
=
-
ltem18of30
Question Id: 360
■ t:,,
\ Mark
<
Previous
C>
Next
~~
Full Saeen
®
Tutorial
il
Lab Values
~ ·
Notes
....1
Calculator
,
Reverse Color
~
Text Zoom
©
Settings

A6 >
pat • Exhibit Display .---- - - - - - - - -- - - Bl X

ep1s
Bio

Vis
eye

©_ Zoom In 0_ Zoom Out C Reset Cf;, New I lf-J Existi ng (!;I My Notebook

)4
https://t.me/USMLEWorldStep1 Show all X
Which of the following is the most likely cause of this patient's loss of vision?

0 A . Amaurosis fugax

0 8 . Diabetic retinopathy

0 C. Pituitary adenoma

0 D. Retinal artery occlusion

0 E. Temporal lobe stroke

Submit

Block Time Elapsed: 00:04:35 ~ II;! rvo Cl @ 0


My Notebook Flashcards Feedback Suspend End Block

)4
"' https://t.me/USMLEWorldStep1 Show all X
Which of the following is the most likely cause of this patient's loss of vision?

0 A . Amaurosis fugax (7°/o)

X ® 8 . Diabetic retinopathy (14%)

0 C. Pituitary adenoma (0%)

✓ Q D. Retinal artery occlusion (76%)

0 E. Temporal lobe stroke (1%)

Incorrect
Correct answer I 1,1 76% ti\ 32 secs
I.!!.!!. Answered correctly \..::,; Time Spent
D

Block Time Elapsed: 00:04:38 ~ II;! rvo Cl @ 0


My Notebook Flashcards Feedback Suspend End Block

)4
"' https://t.me/USMLEWorldStep1 Show all X
= ltem_18of30
Question Id: 360
■ t7' Mark <
Previous
C>
Next
~~
Full Saeen
®
Tutorial
il
Lab Values

Notes
.... I
Calculator
,
Reverse Color
~
Text Zoom
©
Settings

Exhibit Display Bl X
'

Central retinal artery occlusion

Ophccup
Macula

i
Cherry-red
spot
t t
Optic disc
Retinal
whitening

Acu ©. Zoom In 0. Zoom Out a Reset q, New I lf-J Existi ng I!;! My Notebook
loss
isch
... ·- ··••-•-· ·- -· .... -- ·- ··-•--•
Block Time Elapsed: 00:04:38 ~ II;! rvo Cl @ 0
My Notebook Flashcards Feedback Suspend End Block

)4
"' https://t.me/USMLEWorldStep1 Show all X
= Item _1s of 30
Question Id: 360
■ t7' Mark <:::
Previous
C>
Next
~~
Full Screen
®
Tutorial
il
Lab Values
~-
Notes
....I
Calculator
,
Reverse Color
~
Text Zoom
©
Settings

Acute and painless monocular vision loss is characteristic of central retinal artery occlusion (CRAO). The vision
loss includes the entire visual field and is often permanent. Funduscopic findings include a pale retina (due to
ischemia and edema) and a cherry-red macula (the fovea and foveola are thin and have a separate blood supply
from the choroid artery; the red choroid is easily visible underneath).

The central retinal artery is a branch of the ophthalmic artery, which arises from the internal carotid artery. Athero-
and thromboembolism are the most common causes of CRAO. Predisposing conditions include atrial fibrillation
and carotid artery stenosis. CRAO may also be caused by vasculitic diseases (eg, giant cell arteritis).

(Choice A) Amaurosis fugax is a painless, transient, monocular vision loss caused by a small embolus to the
ophthalmic artery. It usually does not last more than a few seconds.

(Choice B) Diabetic retinopathy manifests with blurry vision , black spots, floaters, and decreased peripheral
vision . Acute visual loss may occur in the setting of a complication such as vitreous hemorrhage, but this patient
does not have evidence of neovascularization or hemorrhage.

(Choice C) Large pituitary adenomas may compress the central part of the optic chiasm , causing bitemporal
hemianopia.

(Choice E) The lower division of the geniculocalcarine tract (Meyer loop), which carries impulses from the lower
retina, passes through the temporal lobe. Temporal lobe infarcts involving the Meyer loop cause contralateral
upper quadrantanopsia ("pie in the sky" visual defect).

Educational objective:
Central retinal artery occlusion presents with sudden, painless, and permanent monocular blindness. Funduscopic
examination reveals a pale retina and a "cherry-red" macula.

References

Block Time Elapsed: 00:04:38 ~ II;! rvo Cl @ 0


My Notebook Flashcards Feedback Suspend End Block

)4
"' https://t.me/USMLEWorldStep1 Show all X
= ltem_18of30
Question Id: 360
■ t7' Mark <
Previous
C>
Next
~~
Full Saeen
®
Tutorial
il
Lab Values

Notes
....I
Calculator
,
Reverse Color
~
Text Zoom
©
Settings

Acu - • I • • • I - II • I • • • I • I • • - • • I • • I • • I • a. • • I : !. • I - • I

loss Exhibit Display Bl X

isch
Diabetic retinopathy, proliferative (PDR)
fro

The
and
and

(Ch
oph

(Ch
visi
doe

(Ch
he

(Ch
reti
upp

Edu © UWorld
Cen ©_ Zoom In 0_ Zoom Out C Reset Cf;, New I lf-J Existi ng (!;I My Notebook
exa

References

Block Time Elapsed: 00:04:38 ~ ll;I rvo Cl @ 0


My Notebook Flashcards Feedback Suspend End Block

)4
"' https://t.me/USMLEWorldStep1 Show all X
= ltem_19of30
Question Id: 8557
■ t7' Mark <:::
Previous
C>
Next
~~
Full Screen
®
Tutorial
il
Lab Values

Notes
....I
Calculator
,
Reverse Color
~
Text Zoom
©
Settings

A 34-year-old woman comes to the office due to double vision. The patient first experienced difficulty focusing her
eyes while at work yesterday. On awakening this morning, her symptoms had progressed to frank diplopia.
Medical history is insignifican t. A complete physical examination is performed, including a full neurologic
assessment. Shining light into the patient's right eye causes constriction of her right pupil, but not the left pupil.
Shining light into her left eye causes constriction of only her right pupil. Which of the following additional physical
examination fi ndings is likely to be found in this patient?

0 A . Absence of the left corneal reflex

0 8 . Drooping of the left eyelid

0 C. Inability to close the left eye

0 D. Inward deviation of the left eye

0 E. Visual loss in the left eye

Submit

Block Time Elapsed: 00:04:43 ~ II;! rvo Cl @ 0


My Notebook Flashcards Feedback Suspend End Block

)4
"' https://t.me/USMLEWorldStep1 Show all X
= Item _19 of 30
Question Id: 8557
■ t7' Mark <:::
Previous
C>
Next
~~
Full Screen
®
Tutorial
il
Lab Values

Notes
....I
Calculator
,
Reverse Color
~
Text Zoom
©
Settings

A 34-year-old woman comes to the office due to double vision. The patient first experienced difficulty focusing her
eyes while at work yesterday. On awakening this morning, her symptoms had progressed to frank diplopia.
Medical history is insignifican t. A complete physical examination is performed, including a full neurologic
assessment. Shining light into the patient's right eye causes constriction of her right pupil, but not the left pupil.
Shining light into her left eye causes constriction of only her right pupil. Which of the following additional physical
examination fi ndings is likely to be found in this patient?

0 A . Absence of the left corneal reflex (21 %)

✓® 8 . Drooping of the left eyelid (52%>)

0 C. Inability to close the left eye (5%)

0 D. Inward deviation of the left eye (12%>)

0 E. Visual loss in the left eye (8%)

Correct I 1,1 52% ri\ 08 secs


~ Answered correctly ~ Time Spent

Explanation

Eye muscle(s)
Nerve Oculomotor functions Lesion findings
innervated

• Superior rectus • Adduction • Eye deviated downward & laterally

Block Time Elapsed: 00:04:46 ~ II;! rvo Cl @ 0


My Notebook Flashcards Feedback Suspend End Block

)4
"' https://t.me/USMLEWorldStep1 Show all X
= Item _19 of 30
Question Id: 8557
■ t7' Mark <:::
Previous
C>
Next
~~
Full Screen
®
Tutorial
il
Lab Values
~-
Notes
....I
Calculator
,
Reverse Color
~
Text Zoom
©
Settings

Eye muscle(s)
Nerve Oculomotor functions Lesion findings
innervated

• Superior rectus • Adduction • Eye deviated downward & laterally


• Medial rectus • Depression • Diagonal diplopia
Oculomotor • Inferior rectus • Elevation • Dilation of pupil & loss of
(CN Ill) • Inferior oblique • External rotation accommodation
• Levator palpebrae • Ptosis
supenons

• Superior oblique • Abduction • Eye deviated upward


Trochlear • Internal rotation • Vertical & torsional diplopia
(CN IV) • Depression while
adducted

Abducens • Lateral rectus • Abduction • Eye deviated medially


(CN VI) • Horizontal diplopia

This patient has diplopia (suggesting a lesion impacting extraocular muscles) and a left efferent pupillary defect
(absent left pupillary response when light enters either eye). This is likely due to dysfunction of the left oculomotor
nerve (CN Ill), which has multiple functions:

• Innervation of most of the extraocular muscles (superior rectus, medial rectus, inferior rectus, inferior
oblique). Paralysis of these muscles leads to unopposed action of the superior oblique and lateral rectus,
which results in the characteristic "down and out" pupil.

Block Time Elapsed: 00:04:46 ~ II;! rvo Cl @ 0


My Notebook Flashcards Feedback Suspend End Block

)4
"' https://t.me/USMLEWorldStep1 Show all X
= Item _19 of 30
Question Id: 8557
■ t7' Mark
------------------------------------------
<:::
Previous
C>
Next
~~
Full Screen
®
Tutorial
il
Lab Values
~-
Notes
....I
Calculator
,
Reverse Color
~
Text Zoom
©
Settings

This patient has diplopia (suggesting a lesion impacting extraocular muscles) and a left efferent pupillary defect
(absent left pupillary response when light enters either eye). This is likely due to dysfunction of the left oculomotor
nerve (CN Il l), which has multiple functions:

• Innervation of most of the extraocular muscles (superior rectus, medial rectus, inferior rectus, inferior
oblique). Paralysis of these muscles leads to unopposed action of the superior oblique and lateral rectus,
which results in the characteristic "down and out" pupil.

• Parasympathetic fibers innervate the iris sphincter and ciliary muscle. Paralysis of these fibers causes an
efferent pupillary defect, as seen in this patient.

• Somatic innervation of the levator palpebrae superioris. This elevates the upper eyelid, and paralysis of it
leads to ptosis (ie, drooping of the eyelid).

Although the most common causes of acquired oculomotor nerve palsy are diabetes mellitus and hypertensive
ischemia, patients are often evaluated with neuroimaging to rule out an enlarging aneurysm of the posterior-
communicating artery, which can present with isolated CN Ill palsy.

(Choice A) Absence of the left corneal reflex (corneal stimulation causes orbicularis oculi to contract the eyelids)
can result from lesions involving the afferent limb (CN V1) or efferent limb (CN VII).

(Choice C) A lesion involving the left CN VII would result in the inability to close the left eye due to paralysis of the
orbicularis oculi muscle.

(Choice D) Inward deviation of the left eye would result from paralysis of the lateral rectus, which is innervated by
the abducens nerve (CN VI).

(Choice E) The left optic nerve (CN II) is functional in this patient because light shining into the left eye produced a

Block Time Elapsed: 00:04:46 ~ II;! rvo Cl @ 0


My Notebook Flashcards Feedback Suspend End Block

)4
"' https://t.me/USMLEWorldStep1 Show all X
:= ltem_19of30
Question Id: 8557
■ t7' Mark <:::
Previous
C>
Next
~~
Full Screen
®
Tutorial
il
Lab Values
~•
Notes Calculator
,
Reverse Color
~
Text Zoom
©
Settings

• Parasympathetic fibers innervate the iris sphincter and ciliary muscle. Paralysis of these fibers causes an
efferent pupillary defect, as seen in this patient.

• Somatic innervation of the levator palpebrae superioris. This elevates the upper eyelid, and paralysis of it
leads to ptosis (ie, drooping of the eyelid).

Although the most common causes of acquired oculomotor nerve palsy are diabetes mellitus and hypertensive
ischemia, patients are often evaluated with neuroimaging to rule out an enlarging aneurysm of the posterior-
communicating artery, which can present with isolated CN Ill palsy.

(Choice A) Absence of the left corneal reflex (corneal stimulation causes orbicularis oculi to contract the eyelids)
can result from lesions involving the afferent limb (CN V1) or efferent limb (CN VII).

(Choice C) A lesion involving the left CN VII would result in the inability to close the left eye due to paralysis of the
orbicularis oculi muscle.

(Choice D) Inward deviation of the left eye would result from paralysis of the lateral rectus, which is innervated by
the abducens nerve (CN VI).

(Choice E) The left optic nerve (CN II) is functional in this patient because light shining into the left eye produced a
consensual pupillary response in the right eye.

Educational objective:
The oculomotor nerve (CN Ill) provides innervation to many of the extraocular muscles (superior rectus, medial
rectus, inferior rectus, inferior oblique), parasympathetic innervation to the pupil, and somatic innervation of the
levator palpebrae superioris. Dysfunction of CN Ill leads to a characteristic triad of a "down and out" pupil, an
efferent pupillary defect, and ptosis.

Block Time Elapsed: 00:04:46 ~ II;! rvo Cl @ 0


My Notebook Flashcards Feedback Suspend End Block

)4
"' https://t.me/USMLEWorldStep1 Show all X
= ltem_19of30
Question Id: 8557
■ t7' Mark <
Previous
C>
Next
~~
Full Saeen
®
Tutorial
il
Lab Values

Notes
....I
Calculator
,
Reverse Color
~
Text Zoom
©
Settings

• -
Exhibit Display
- Bl X

• Right abducens nerve palsy

Alth
Rightward gaze
isch
com

(Ch
can

(Ch Neutral
orbi

(Ch
the

(Ch
con Leftward gaze

Edu
The ©UWOl1d
rect ©_ Zoom In 0_ Zoom Out C Reset Cf;, New I lf-J Existi ng (!;I My Notebook

effe

Block Time Elapsed: 00:04:46 ~ ll;I rvo Cl @ 0


My Notebook Flashcards Feedback Suspend End Block

"' !!I 51549 (1).mp4


"' https://t.me/USMLEWorldStep1 Show all X
= Item ~O of 30
Question Id: 18951
■ t7' Mark <:::
Previous
C>
Next
~~
Full Screen
®
Tutorial
il
Lab Values

Notes
.. __1
Calculator
,
Reverse Color
~
Text Zoom
©
Settings

A 75-year-old man comes to the office due to a 2-year history of slowly worsening vision in both eyes. The
symptoms are worse at night and the patient has stopped driving at night due to excessive glare from oncoming
headlights. He has worked his entire life as a farmer and continues to drive tractors and other farm equipment
during the day without difficulty. Medical history is unremarkable. Ocular examination shows loss of the red reflex
and poor visualization of retinal detail. Acuity testing shows 20/100 vision in both eyes; visual field testing is
normal. Which of the following is the most likely etiologic factor for this patient's eye condition?

0 A . Accumulation of sorbitol in the lens

0 8 . Age-related oxidative injury

0 C. Decreased collagen fibril production

0 D. lschemia and neovascularization

0 E. Loss of lens distensibility

Submit

Block Time Elapsed: 00:04:47 ~ II;! rvo Cl @ 0


My Notebook Flashcards Feedback Suspend End Block

"' !!I 51549 (1).mp4


"' https://t.me/USMLEWorldStep1 Show all X
= Item ~O of 30
Question Id: 18951
■ t7' Mark <:::
Previous
C>
Next
~~
Full Screen
®
Tutorial
il
Lab Values

Notes
.. __1
Calculator
,
Reverse Color
~
Text Zoom
©
Settings

A 75-year-old man comes to the office due to a 2-year history of slowly worsening vision in both eyes. The
symptoms are worse at night and the patient has stopped driving at night due to excessive glare from oncoming
headlights. He has worked his entire life as a farmer and continues to drive tractors and other farm equipment
during the day without difficulty. Medical history is unremarkable. Ocular examination shows loss of the red reflex
and poor visualization of retinal detail. Acuity testing shows 20/100 vision in both eyes; visual field testing is
normal. Which of the following is the most likely etiologic factor for this patient's eye condition?

0 A . Accumulation of sorbitol in the lens (21%)

✓® 8 . Age-related oxidative injury (55%>)

O C. Decreased collagen fibril production (3%)

0 D. lschemia and neovascularization (4°/o)

0 E. Loss of lens distensibility (14%)

Correct I 1,1 55% ri\ 05 secs i==1 2023


~ Answered correctly ~ Time Spent IE=.J Version

Explanation

Clinical features of age-related cataract

• Age >60
• Chronic sunlight exposure

Block Time Elapsed: 00:04:51 ~ II;! rvo Cl @ 0


My Notebook Flashcards Feedback Suspend End Block

"' !!I 51549 (1).mp4


"' https://t.me/USMLEWorldStep1 Show all X
= Item ~O of 30
Question Id: 18951
■ t7' Mark <:::
Previous
C>
Next
~~
Full Screen
®
Tutorial
il
Lab Values
~-
Notes
....I
Calculator
,
Reverse Color
~
Text Zoom
©
Settings

Clinical features of age-related cataract

• Age >60
• Chronic sunlight exposure
• Diabetes mellitus
Risk factors
• Glucocorticoid use
• Smoking
• HIV infection

• Gradual loss of visual acuity


Symptoms • Excessive glare, halos around bright lights
• Myopic shift

• Opacification of lens
Signs
• Loss of red reflex

This patient has cataracts: progressive opacification of the lenses with chronic loss of visual acuity. Scattering
of light within the lens leads to glare and halos around bright lights, especially at night. Examination reveals
cloudiness in the lens, decreased detail when examining the reti na, and loss of the red reflex.

Transparency of the lens requires an ordered epithelial cell structure and maintenance of intracellular crystallins.
Aging and environmental stressors can disrupt transparency and contribute to cataract formation through the
following mechanisms:

• Nuclear sclerosis: new layers of epithelia form on the cortex of the lens, compacting older layers beneath

• Photooxidative damage and cross-linking of crystallins causes brown/yellow pigmentary changes


f . . If
0 •• d • t fh d •• I fib th t d • t
Block Time Elapsed: 00:04:51 ~ II;! rvo Cl @ 0
My Notebook Flashcards Feedback Suspend End Block

"' !!I 51549 (1).mp4


"' https://t.me/USMLEWorldStep1 Show all X
= Item ~O of 30
Question Id: 18951
■ t7' Mark <:::
Previous
C>
Next
~~
Full Screen
®
Tutorial
il
Lab Values
~-
Notes
....I
Calculator
,
Reverse Color
~
Text Zoom
©
Settings

This patient has cataracts: progressive opacification of the lenses with chronic loss of visual acuity. Scattering
of light within the lens leads to glare and halos around bright lights, especially at night. Examination reveals
cloudiness in the lens, decreased detail when examining the retina, and loss of the red reflex.

Transparency of the lens requires an ordered epithelial cell structure and maintenance of intracellular crystallins.
Aging and environmental stressors can disrupt transparency and contribute to cataract formation through the
following mechanisms:

• Nuclear sclerosis: new layers of epithelia form on the cortex of the lens, compacting older layers beneath

• Photooxidative damage and cross-linking of crystallins causes brown/yellow pigmentary changes

• Osmotic injury resulting in development of hydropic lens fibers that degenerate

Cumulative photooxidative stress is worsened by heavy UV exposure (eg, outdoor occupations), smoking, or
ionizing radiation. Likewise, osmotic injury is accelerated in patients with diabetes mellitus. A disordered balance
of prooxidative versus antioxidative compounds in the lens (eg, decreased glutathione) may also contribute.

(Choice A) In patients with diabetes, excess glucose is converted to sorbitol at a greater rate than sorbitol can be
metabolized to fructose. Osmotic injury due to the higher concentration of sorbitol in the lens can lead to cataract.
However, in patients without diabetes, the conversion of sorbitol to fructose is adequate to prevent the buildup of
sorbitol.

{Choice C) Photoaging of skin is characterized by decreased collagen fibril production and loss of tissue elasticity
in sun-exposed areas. However, defects in collagen production are not a significant contributor to cataract.

(Choice D) Microvascular ischemia in patients with long-standing diabetes can cause neovascularization in the
retina, leading to diabetic retinopathy. Most patients are asymptomatic until late in the course of the disease, when
vitreous hemorrhage or retinal detachment causes acute vision loss.
Block Time Elapsed: 00:04:51 ~ II;! rvo Cl @ 0
My Notebook Flashcards Feedback Suspend End Block

"' !!I 51549 (1).mp4


"' https://t.me/USMLEWorldStep1 Show all X
=
-
ltem~Oof30
Question Id: 18951
■ t7' Mark <:::
Previous
C>
Next
~~
Full Screen
®
Tutorial
il
Lab Values
~•
Notes Calculator
,
Reverse Color
~
Text Zoom
©
Settings

• Nuclear sclerosis: new layers of epithelia form on the cortex of the lens, compacting older layers beneath

• Photooxidative damage and cross-linking of crystallins causes brown/yellow pigmentary changes

• Osmotic injury resulting in development of hydropic lens fibers that degenerate

Cumulative photooxidative stress is worsened by heavy UV exposure (eg, outdoor occupations), smoking, or
ionizing radiation. Likewise, osmotic injury is accelerated in patients with diabetes mellitus. A disordered balance
of prooxidative versus antioxidative compounds in the lens (eg, decreased glutathione) may also contribute.

(Choice A) In patients with diabetes, excess glucose is converted to sorbitol at a greater rate than sorbitol can be
metabolized to fructose. Osmotic injury due to the higher concentration of sorbitol in the lens can lead to cataract.
However, in patients without diabetes, the conversion of sorbitol to fructose is adequate to prevent the buildup of
sorbitol .

{Choice C) Photoaging of skin is characterized by decreased collagen fibril production and loss of tissue elasticity
in sun-exposed areas. However, defects in collagen production are not a significant contributor to cataract.

(Choice D) Microvascular ischemia in patients with long-standing diabetes can cause neovascularization in the
retina, leading to diabetic retinopathy. Most patients are asymptomatic until late in the course of the disease, when
vitreous hemorrhage or retinal detachment causes acute vision loss.

(Choice E) Presbyopia is a type of refractive error caused by loss of the normal distensibility of the lens with age.
It reduces the ability of the eye to focus on near objects but does not cause loss of visual acuity.

Educational objective:
Cataracts are characterized by progressive opacification of the lens with chronic loss of visual acuity. Aging and
environmental exposures (eg, UV light) contribute to cataract formation by inducing nuclear sclerosis,
photooxidative damage to lens crystallins, and osmotic injury.

Block Time Elapsed: 00:04:51 ~ II;! rvo Cl @ 0


My Notebook Flashcards Feedback Suspend End Block

"' !!I 51549 (1).mp4


"' https://t.me/USMLEWorldStep1 Show all X
Cataract

Thi
of Ii
clou

Tra
Agi
folio




Cu 0UWOrld

©_ Zoom In 0_ Zoom Out C Reset Cf;, New I lf-J Existi ng (!;I My Notebook

(Choice A) In patients with diabetes, excess glucose is converted to sorbitol at a greater rate than sorbitol can be

Block Time Elapsed: 00:04:51 ~ II;! rvo [J @ 0


My Notebook Flashcards Feedback Suspend End Block

"' !!I 51549 (1).mp4


"' https://t.me/USMLEWorldStep1 Show all X
Cataract

Decreased
Thi Healthy Lens with acuity
lens cataract
of Ii
clou

Tra
Agi
folio

• Light is sharply focused on fovea Light is scattered, causing


glare and halos around bright lights
@UWo<l<I


Cu
©_ Zoom In 0_ Zoom Out C Reset Cf;, New I lf-J Existi ng (!;I My Notebook

(Choice A) In patients with diabetes, excess glucose is converted to sorbitol at a greater rate than sorbitol can be

Block Time Elapsed: 00:04:51 ~ II;! rvo [J @ 0


My Notebook Flashcards Feedback Suspend End Block

"' !!I 51549 (1).mp4


"' https://t.me/USMLEWorldStep1 Show all X
Normal eyes & white reflex

This
of Ii
clou Normal eyes
Red reflexes & corneal light reflexes are equal.
Tra
Agi
folio




Cu Absent reflex
\Nhite reflex on abnormal eye can result from opacities of the lens
ioni (eg. cataract) or tumor (eg, retinoblastoma).
C>UWortd
ofp
©. Zoom In 0. Zoom Out a Reset q;, New I lf-J Existi ng I!;! My Notebook

!!I 51549 (1).mp4


https://t.me/USMLEWorldStep1 Show all X
= Item ~O of 30
Question Id: 18951
·- - ...
■ t7' Mark
. .. . .·- -. . ,,
<
Previous
C>
. . . . .. . . .
Next
- .
~~
Full Saeen
®
Tutorial
il
Lab Values

Notes
.. __1
Calculator
,
Reverse Color
~
Text Zoom
©
Settings

of Ii

Tra
Agi
folio
Diabetic cataract formation

Fast Slow
• Aldose Sorbitol
reductase dehydrogenase
• Glucose - - ----::,_ -----==~
, - - - - • Sorbitol ~ • Fructose
7
Cu
ioni
NADPH
1
NADP•

Depletion of NADPH
NADH

of p

(Ch Glycation of
lens proteins
1
Oxidative stress
Osmotic
injury
met
I I
Ho
i
Cataract
ClUWorld

(Ch
ms

(Ch ©_ Zoom In 0_ Zoom Out C Reset Cf;, New I lf-J Existi ng (!;I My Notebook

Block Time Elapsed: 00:04:51 ~ II;! rvo [J @ 0


My Notebook Flashcards Feedback Suspend End Block

"' !!I 51549 (1).mp4


"' https://t.me/USMLEWorldStep1 Show all X
= Item ~1 of 30
Question Id: 19126
■ t7' Mark <:::
Previous
C>
Next
~~
Full Screen
®
Tutorial
il
Lab Values

Notes
....I
Calculator
,
Reverse Color
~
Text Zoom
©
Settings

A 17-year-old boy is brought to the clinic due to poor vision at night. The patient developed mild difficulty seeing at
night about 4 months ago, and it has slowly worsened . Visual field examination shows visual loss in the
midperiphery. External eye examination reveals normal tear formation . Funduscopic examination shows bilateral
optic disc pallor, attenuation of retinal vessels, and areas of dark discoloration in the retina. Which of the following
is the most likely mechanism of this patient's visual impairment?

0 A. Biallelic inactivation of RB1 gene

0 B. Genetic progressive retinal dystrophy

0 C. Increased intracranial pressure

0 D. Poor aqueous outflow due to narrow angle

0 E. Retinal artery occlusion

Submit

Block Time Elapsed: 00:04:56 ~ II;! rvo Cl @ 0


My Notebook Flashcards Feedback Suspend End Block

"' !!I 51549 (1).mp4


"' https://t.me/USMLEWorldStep1 Show all X
= Item ~1 of 30
Question Id: 19126
■ t7' Mark <:::
Previous
C>
Next
~~
Full Screen
®
Tutorial
il
Lab Values

Notes
....I
Calculator
,
Reverse Color
~
Text Zoom
©
Settings

A 17-year-old boy is brought to the clinic due to poor vision at night. The patient developed mild difficulty seeing at
night about 4 months ago, and it has slowly worsened . Visual field examination shows visual loss in the
midperiphery. External eye examination reveals normal tear formation. Funduscopic examination shows bilateral
optic disc pallor, attenuation of retinal vessels, and areas of dark discoloration in the retina. Which of the following
is the most likely mechanism of this patient's visual impairment?

0 A. Biallelic inactivation of RB1 gene (15%>)

✓ (!) B. Genetic progressive retinal dystrophy (56%)

0 C. Increased intracranial pressure (8°/o)

0 D. Poor aqueous outflow due to narrow angle (5%)

0 E. Retinal artery occlusion ( 13%)

Correct I 1,1 56% fi"\ 09 secs


L!!.!.!. Answered correctly \.:.JTimeSpent

Explanation

Retinitis pigmentosa

• Genetic dystrophy
Etiology
• Progressive retinal degeneration (photoreceptors, pigmented epithelium)

• • ••
Block Time Elapsed: 00:05:00 ~ II;! rvo Cl @ 0
My Notebook Flashcards Feedback Suspend End Block

"' !!I 51549 (1).mp4


"' https://t.me/USMLEWorldStep1 Show all X
=
-
ltem~1 of30
Question Id: 19126
■ t7' Mark <:::
Previous
C>
Next
~~
Full Screen
®
Tutorial
il
Lab Values
~•
Notes Calculator
,
Reverse Color
~
Text Zoom
©
Settings

Retinitis pigmentosa

• Genetic dystrophy
Etiology
• Progressive retinal degeneration (photoreceptors, pigmented epithelium)

• Loss of rods:
o Night blindness
Clinical features o Peripheral visual field loss
• Loss of cones:
o Decreased central visual acuity (late finding)

• Retinal vessel attenuation (likely due to altered metabolic demand)


Funduscopic changes • Optic disc pallor (optic nerve atrophy and gliosis)
• Pigment accumulation (characteristic bone-spicule pattern around vessels)

This patient has poor night vision, visual field loss, and funduscopic changes (eg, pigment accumulation, optic disc
pallor, retinal vessel attenuation) consistent with retinitis pigmentosa, a genetic disease characterized by
progressive dystrophy of the retinal pigmented epithelium and photoreceptors (eg , rods, cones).

Rods, which are highly metabolically active and most prevalent in the periphery, are affected first, leading to the
initial manifestations of progressive night blindness and loss of peripheral vision. Early funduscopic changes
include retinal vessel attenuation (ie, narrow vessels likely due to alterations in retinal metabolic demand) and a
waxy, pale optic disc (due to optic nerve atrophy and gliosis).

In advanced disease, loss of cones (most prevalent in the central retina) causes decreased central visual acuity.
Degeneration of retinal pigmented epithelial cells releases pigment, which deposits in a characteristic bone-
• I • tt th f th d.
Block Time Elapsed: 00:05:00 ~ II;! rvo Cl @ 0
My Notebook Flashcards Feedback Suspend End Block

"' !!I 51549 (1).mp4


"' https://t.me/USMLEWorldStep1 Show all X
=
-
ltem~1 of30
Question Id: 19126
■ t7' Mark <:::
Previous
C>
Next
~~
Full Screen
®
Tutorial
il
Lab Values
~•
Notes Calculator
,
Reverse Color
~
Text Zoom
©
Settings

This patient has poor night vision, visual field loss, and funduscopic changes (eg, pigment accumulation, optic disc
pallor, retinal vessel attenuation) consistent with retinitis pigmentosa, a genetic disease characterized by
progressive dystrophy of the retinal pigmented epithelium and photoreceptors (eg , rods, cones).

Rods, which are highly metabolically active and most prevalent in the periphery, are affected first, leading to the
initial manifestations of progressive night blindness and loss of peripheral vision. Early funduscopic changes
include retinal vessel attenuation (ie, narrow vessels likely due to alterations in retinal metabolic demand) and a
waxy, pale optic disc (due to optic nerve atrophy and gliosis).

In advanced disease, loss of cones (most prevalent in the central retina) causes decreased central visual acuity.
Degeneration of retinal pigmented epithelial cells releases pigment, which deposits in a characteristic bone-
spicule pattern over the course of the disease.

(Choice A) Retinoblastoma, which is caused by inactivation of both alleles of the RB1 gene (ie, 2-hit hypothesis),
typically presents in the first 3 years of life with leukocoria (ie, white pupillary reflex), strabismus, and a creamy-
white mass on funduscopic examination.

(Choice C) Increased intracranial pressure can be associated with transient visual obscuration and decreased
visual acuity. However, other symptoms (eg, headache, vomiting) are also typically present and papilledema with
blurred optic disc margins would be seen on funduscopy.

(Choice D) Angle-closure glaucoma occurs due to narrowing of the angle between the iris and the cornea, which
blocks aqueous outflow. Patients typically have acute eye pain and vision loss with conjunctiva! erythema, cornea
edema, and a mid-dilated, poorly reactive pupil. Optic nerve atrophy secondary to chronically increased intraocular
pressure results in optic nerve cupping on funduscopy.

(Choice E) Central retinal artery occlusion typically presents with unilateral painless acute vision loss in patients
• • • •
Block Time Elapsed: 00:05:00 ~ II;! rvo Cl @ 0
My Notebook Flashcards Feedback Suspend End Block

"' !!I 51549 (1).mp4


"' https://t.me/USMLEWorldStep1 Show all X
=-
-
'.
ltem~1 of30
.- .. . .. . ..
Question Id: 19126
.
■ t7' Mark
. - .. . - . . .
<:::
Previous
C>
Next
~~
Full Screen
®
Tutorial
il
Lab Values
~•
Notes Calculator
,
Reverse Color
~
Text Zoom
©
Settings

In advanced disease, loss of cones (most prevalent in the central retina) causes decreased central visual acuity.
Degeneration of retinal pigmented epithelial cells releases pigment, which deposits in a characteristic bone-
spicule pattern over the course of the disease.

(Choice A) Retinoblastoma, which is caused by inactivation of both alleles of the RB1 gene (ie, 2-hit hypothesis),
typically presents in the first 3 years of life with leukocoria (ie, white pupillary reflex), strabismus, and a creamy-
white mass on funduscopic examination.

(Choice C) Increased intracranial pressure can be associated with transient visual obscuration and decreased
visual acuity. However, other symptoms (eg, headache, vomiting) are also typically present and papilledema with
blurred optic disc margins would be seen on funduscopy.

(Choice D) Angle-closure glaucoma occurs due to narrowing of the angle between the iris and the cornea, which
blocks aqueous outflow. Patients typically have acute eye pain and vision loss with conjunctiva! erythema, cornea
edema, and a mid-dilated, poorly reactive pupil. Optic nerve atrophy secondary to chronically increased intraocular
pressure results in optic nerve cupping on funduscopy.

(Choice E) Central retinal artery occlusion typically presents with unilateral painless acute vision loss in patients
age >40 with thromboembolic risk factors (eg, hypertension). Retinal whitening and a cherry-red macula are the
characteristic funduscopic changes.

Educational objective:
Retinitis pigmentosa is a genetic condition resulting in progressive dystrophy of retinal pigmented epithelium and
photoreceptors. Patients typically present with progressive night blindness and loss of peripheral vision due to
early loss of rods, which are highly metabolically active. Funduscopic examination may show dark pigments
deposited in a bone-spicule pattern in addition to retinal vessel attenuation and optic disc pallor.

Block Time Elapsed: 00:05:00 ~ II;! rvo Cl @ 0


My Notebook Flashcards Feedback Suspend End Block

"' !!I 51549 (1).mp4


"' https://t.me/USMLEWorldStep1 Show all X
Retinitis pigmentosa
Thi
pall
pro

Rod
initi
incl
wax

In a
Deg
spic

(Ch
typi
CUW0<ld
whit

(Ch
VISU ©_ Zoom In 0_ Zoom Out C Reset Cf;, New I lf-J Existi ng (!;I My Notebook
blur

(Choice D) Angle-closure glaucoma occurs due to narrowing of the angle between the iris and the cornea, which

Block Time Elapsed: 00:05:00 ~ II;! rvo Cl @ 0


My Notebook Flashcards Feedback Suspend End Block

"' !!I 51549 (1).mp4


"' https://t.me/USMLEWorldStep1 Show all X
Knudson's 2-hit hypothesis

Both copies of the gene must be knocked out to promote malignancy.


This
pall Sporadic cancer:
2 acquired mutations
pro

Rod
initi
incl
wax Tumor

In a
Deg Hereditary cancer:
spic 1 inherited & 1 acquired mutation

(Ch
typi
whit

(Ch Tumor
VISU (C)UWo<ld

blur
©_ Zoom In 0_ Zoom Out C Reset Cf;, New I lf-J Existi ng (!;I My Notebook

!!I 51549 (1).m p4


https://t.me/USMLEWorldStep1 Show all X
Normal eyes & white reflex

This
pall
pro

Rod
initi
incl Normal eyes
Red reflexes & corneal light reflexes are equal.
wax

In a
Deg
spic

(Ch
typi
whit
Absent reflex
(Ch \Nhite reflex on abnormal eye can result from opacities of the lens
VISU (eg. cataract) or tumor (eg, retinoblastoma).
C>UWortd
blur
©. Zoom In 0. Zoom Out a Reset q;, New I lf-J Existing I!;! My Notebook

!!I 51549 (1).mp4


https://t.me/USMLEWorldStep1 Show all X
pro

Rod
Normal optic disc Optic disc swelling
initi
incl
wax

In a
Deg
spic

(Ch
typi
whit

(Ch CUW0<ld

VISU

blur

(Ch
blo ©_ Zoom In 0_ Zoom Out C Reset Cf;, New I lf-J Existi ng (!;I My Notebook

pressure results in optic nerve cupping on funduscopy.

Block Time Elapsed: 00:05:00 ~ II;! rvo Cl @ 0


My Notebook Flashcards Feedback Suspend End Block

"' !!I 51549 (1).mp4


"' https://t.me/USMLEWorldStep1 Show all X
=.. Item ~1 of 30
Question Id: 19126
,, -
■ t7' Mark
. . -.. - . - ' -. . . .-
<:::
..
Previous
C>
Next
' - - . -. ..
~~
Full Saeen
®
Tutorial
il
Lab Values

Notes
....I
Calculator
,
Reverse Color
~
Text Zoom
©
Settings

initi . ---·---·----- -·------- -·-- ------ -- - ·- ·---··-·--


incl Exhibit Display Bl X

wax
Optic disc in glaucoma
In a
Deg Normal Open-angle glaucoma

spic

(Ch
typi
whit
Disc
(Ch
VISU

blur

(Ch
bloc
ede • Enlarged cup with cup/disc ratio >0.6
pre • Cup/disc ratio <0.5 • Increase in cup size over time
• Clear disc rim • Thinning of disc rim
(Ch • Pale disc (optic nerve atrophy)
age ~UWortd
cha
©_ Zoom In 0_ Zoom Out C Reset Cf;, New I lf-J Existi ng (!;I My Notebook

Reti
--- - - -- - - -- " - - - -· . - -- - ....... ···- .. - - -- --- -· -- -
Block Time Elapsed: 00:05:00 ~ ll;I rvo Cl @ 0
My Notebook Flashcards Feedback Suspend End Block

"' !!I 51549 (1).mp4


"' https://t.me/USMLEWorldStep1 Show all X
=.. Item ~1 of 30
Question Id: 19126
,, -
■ t7' Mark
. . .. - - . - ' - .... -
<:::
..
Previous
C>
Next
' - - . - . ..
~~
Full Saeen
®
Tutorial
il
Lab Values

Notes
....I
Calculator
,
Reverse Color
~
Text Zoom
©
Settings

initi -···- ------- -······· · ... -- ~ -- - .. ·- -- - - - -

incl Exhibit Display Bl X

wax
Central retinal artery occlusion
In a
Deg
spic

(Ch
Ophccup
typi Macula

whit
i
(Ch
VISU

blur

(Ch
Cherry-red
spot
t t
Optic disc
bloc Retinal
whitening
ede
pre

(Ch
age
cha
©. Zoom In 0. Zoom Out a Reset q, New I lf-J Existi ng I!;! My Notebook

Reti
- - - - - - - - - - - " - - - - - . --
- - ....... ···- .. - - - - - - - -· - - -
Block Time Elapsed: 00:05:00 ~ II;! rvo Cl @ 0
My Notebook Flashcards Feedback Suspend End Block

"' !!I 51549 (1).mp4


"' https://t.me/USMLEWorldStep1 Show all X
= ltem~of30
Question Id: 398
■ t7' Mark <:::
Previous
C>
Next
~~
Full Screen
®
Tutorial
il
Lab Values

Notes
....I
Calculator
,
Reverse Color
~
Text Zoom
©
Settings

Physical examination of a 23-year-old man diagnosed with chronic hepatitis shows corneal changes that are visible
to the naked eye, as shown in the image below.

This patient is most likely to have which of the following conditions?

0 A . Pulmonary emphysema

0 8 . Diabetes mellitus

- =-- • - •• -- ••·
Block Time Elapsed: 00:05:03 ~ II;! rvo Cl @ 0
My Notebook Flashcards Feedback Suspend End Block

"' !!I 51549 (1).mp4


"' https://t.me/USMLEWorldStep1 Show all X
=
-
Item 22 of 30
Question Id: 398
■ t:,,
\ Mark
<
Previous
C>
Next
~~
Full Saeen
®
Tutorial
il
Lab Values

Notes
~
Calculator
,
Reverse Color
~
Text Zoom
©
Settings

Phy
Exhibit Display Bl x
to t

©_ Zoom In 0_ Zoom Out C Reset Cf;, New I lf-J Existi ng (!;I My Notebook

!!I 51549 (1).mp4


https://t.me/USMLEWorldStep1 Show all X
This patient is most likely to have which of the following conditions?

0 A . Pulmonary emphysema

0 8 . Diabetes mellitus

0 C. Basal ganglia atrophy

0 D. Renal cancer

0 E. Hypertrophic cardiomyopathy

0 F. Congenital deafness

Submit

Block Time Elapsed: 00:05:09 ~ II;! rvo Cl @ 0


My Notebook Flashcards Feedback Suspend End Block

"' !!I 51549 (1).mp4


"' https://t.me/USMLEWorldStep1 Show all X
This patient is most likely to have which of the following conditions?

0 A . Pulmonary emphysema (2%)

X ® 8 . Diabetes mellitus (14%)

✓ Q C. Basal ganglia atrophy (68°/o)

0 D. Renal cancer (3%)

0 E. Hypertrophic cardiomyopathy (7%)

0 F. Congenital deafness (3%)

Incorrect
Correct answer I 1,1 68% f'i\ 12 secs
I.!!.!!. Answered correctly \..::,; Time Spent
C

Block Time Elapsed: 00:05:12 ~ II;! rvo Cl @ 0


My Notebook Flashcards Feedback Suspend End Block

"' !!I 51549 (1).mp4


"' https://t.me/USMLEWorldStep1 Show all X
:= ltem~of30
Question Id: 398
■ t7' Mark <:::
Previous
C>
Next
~~
Full Screen
®
Tutorial
il
Lab Values
~-
Notes
....I
Calculator
,
Reverse Color
~
Text Zoom
©
Settings

The image demonstrates a Kayser-Fleischer ring, an ophthalmologic finding most strongly associated with Wilson's
disease (although it can also be seen in chronic cholestatic diseases such as primary biliary cirrhosis). Kayser-
Fleischer rings are best identified on slit-lamp examination, but they can be observed grossly, especially with long-
standing, untreated disease. The rings are formed through the granular deposition of copper within Descemet's
membrane in the cornea.

Wilson's disease is a rare, autosomal recessive disease most often identified in individuals 5-40 years old. The
genetic mutations associated with Wilson's disease hinder copper metabolism by reducing the formation and
secretion of ceruloplasmin and decreasing hepatic secretion of copper into the biliary system. Copper is a pro-
oxidant and causes damage to hepatic tissue through the generation of free radicals. Eventually, it leaks from
injured hepatocytes into the circulation to be deposited in various tissues, including the cornea and basal ganglia.
Atrophy of the basal ganglia then ensues.

Treatment is life-long and focuses on removing accumulated copper in the tissues and preventing its re-
accumulation. First-line medications include copper chelators such as d-penicillamine and trientine.

(Choice A) Pulmonary emphysema commonly develops in patients with alpha-1 antitrypsin deficiency.

(Choice B) Diabetes mellitus commonly develops in patients with advanced hemochromatosis. It has no
association with Wilson's disease.

(Choice D) Renal cancer (clear cell carcinoma) occurs more frequently in patients with von Hippel-Lindau
disease, not Wilson's disease.

(Choice E) Hypertrophic cardiomyopathy presenting in childhood and adolescence is an autosomal dominant


disease involving the cardiac sarcomere. It is not associated with Wilson's disease.

(Choice F) Congenital deafness can develop secondary to numerous hereditary and nonhereditary conditions.

Block Time Elapsed: 00:05:12 ~ II;! rvo Cl @ 0


My Notebook Flashcards Feedback Suspend End Block

"' !!I 51549 (1).mp4


"' https://t.me/USMLEWorldStep1 Show all X
:= ltem~of30
Question Id: 398
■ t7' Mark <:::
Previous
C>
Next
~~
Full Screen
®
Tutorial
il
Lab Values
~-
Notes
....I
Calculator
,
Reverse Color
~
Text Zoom
©
Settings

Wilson's disease is a rare, autosomal recessive disease most often identified in individuals 5-40 years old. The
genetic mutations associated with Wilson's disease hinder copper metabolism by reducing the formation and
secretion of ceruloplasmin and decreasing hepatic secretion of copper into the biliary system. Copper is a pro-
oxidant and causes damage to hepatic tissue through the generation of free radicals. Eventually, it leaks from
injured hepatocytes into the circulation to be deposited in various tissues, incl uding the cornea and basal ganglia.
Atrophy of the basal ganglia then ensues.

Treatment is life-long and focuses on removing accumulated copper in the tissues and preventing its re-
accumulation. First-line medications include copper chelators such as d-penicillamine and trientine.

(Choice A) Pulmonary emphysema commonly develops in patients with alpha-1 antitrypsin deficiency.

{Choice B) Diabetes mellitus commonly develops in patients with advanced hemochromatosis. It has no
association with Wilson's disease.

(Choice D) Renal cancer (clear cell carcinoma) occurs more frequently in patients with von Hippel-Lindau
disease, not Wilson's disease.

(Choice E) Hypertrophic cardiomyopathy presenting in childhood and adolescence is an autosomal dominant


disease involving the cardiac sarcomere. It is not associated with Wilson's disease.

(Choice F) Congenital deafness can develop secondary to numerous hereditary and nonhereditary conditions.
Wilson's disease has no association with congenital deafness.

Educational objective:
The Kayser-Fleischer ring is an ophthalmologic finding most strongly associated with Wilson's disease. It is seen
most frequently in patients with neuropsychiatric complications. Basal ganglia atrophy is typically present in these
patients.

Block Time Elapsed: 00:05:12 ~ II;! rvo Cl @ 0


My Notebook Flashcards Feedback Suspend End Block

"' !!I 51549 (1).mp4


"' https://t.me/USMLEWorldStep1 Show all X
= Item ~3 of 30
Question Id: 107365
■ t7' Mark <:::
Previous
C>
Next
~~
Full Screen
®
Tutorial
il
Lab Values

Notes
....I
Calculator
,
Reverse Color
~
Text Zoom
©
Settings

An 85-year-old woman comes to the emergency department due to worsening eye pain. The patient noticed a
rash around the left eye yesterday and, this morning, began experiencing left eye pain, redness, and decreased
vision . Physical examination shows scattered vesicles on the left side of the forehead and nose and on the left
upper and lower eyelids. Left eye examination shows conjunctiva! erythema, and fluorescein staining
demonstrates linear, branching ulcerations on the cornea. Right eye examination and the remainder of the
physical examination are normal. Which of the following pathogens is most likely responsible for this patient's
current condition?

0 A . Adenovirus

0 8 . Candida albicans

0 C. Herpes simplex virus

0 D. Pseudomonas aeruginosa

0 E. Varicella-zoster virus

Submit

Block Time Elapsed: 00:05:14 ~ II;! rvo Cl @ 0


My Notebook Flashcards Feedback Suspend End Block

"' !!I 51549 (1).mp4


"' https://t.me/USMLEWorldStep1 Show all X
= Item ~3 of 30
Question Id: 107365
■ t7' Mark <:::
Previous
C>
Next
~~
Full Screen
®
Tutorial
il
Lab Values

Notes
....I
Calculator
,
Reverse Color
~
Text Zoom
©
Settings

An 85-year-old woman comes to the emergency department due to worsening eye pain. The patient noticed a
rash around the left eye yesterday and, this morning, began experiencing left eye pain, redness, and decreased
vision . Physical examination shows scattered vesicles on the left side of the forehead and nose and on the left
upper and lower eyelids. Left eye examination shows conjunctiva! erythema, and fluorescein staining
demonstrates linear, branching ulcerations on the cornea. Right eye examination and the remainder of the
physical examination are normal. Which of the following pathogens is most likely responsible for this patient's
current condition?

0 A . Adenovirus (2%)

X ® 8 . Candida albicans ( 1% )

0 C. Herpes simplex virus (28%>)

0 D. Pseudomonas aeruginosa (1%)

✓ Q E. Varicella-zoster virus (65%)

Incorrect
Correct answer I 1,1 65% fi"\ 06 secs
I.!!!!. Answered correctly 1..::.J Time Spent
E

Explanation

This patient has herpes zoster ophthalmicus, a condition caused by reactivation of latent varicella-zoster virus
(VZV) in the ophthalmic division of the trigeminal nerve (CN V1 ).

. . . .. . .. . . - .. - . -··· ·- . ···-· - .. ·- ... -· ... ·-· - .


Block Time Elapsed: 00:05:18 ~ II;! rvo Cl @ 0
My Notebook Flashcards Feedback Suspend End Block

"' !!I 51549 (1).mp4


"' https://t.me/USMLEWorldStep1 Show all X
=
-
ltem~3of30
Question Id: 107365
■ t7' Mark <:::
Previous
C>
Next
~~
Full Screen
®
Tutorial
il
Lab Values
~•
Notes Calculator
,
Reverse Color
~
Text Zoom
©
Settings

This patient has herpes zoster ophthalmicus, a condition caused by reactivation of latent varicella-zoster virus
(VZV) in the ophthalmic division of the trigeminal nerve (CN V1 ).

Following primary VZV infection (ie, chickenpox), the virus is transported retrograde along sensory nerves to
cranial nerve and dorsal spinal ganglia, where it lies dormant for years. Reactivation of the virus most commonly
occurs in elderly or immunosuppressed patients and typically causes sensory disturbances (eg, burning or
shooting pains) and eruption of a painful rash (eg , scattered vesicles) in the associated dermatome (eg ,
unilateral forehead, side of the nose, periorbital area). A prodrome of systemic symptoms (eg, headache, malaise,
fever) can also occur.

Because CN V1 also conveys sensory innervation of the cornea (sensory limb of the corneal/blink reflex),
involvement can cause acute keratitis (ie, corneal inflammation) and corneal ulceration (eg, linear, branching
ulcerations on fluorescein stain). Involvement of the side or tip of the nose (ie, Hutchinson sign) correlates highly
with ocular involvement because the nasociliary branch of CN V1 innervates both the eye and the side/tip of the
nose.

(Choice A) Viral conjunctivitis is most commonly caused by adenovirus. It typically presents with monocular
conjunctiva! erythema and eye discomfort; scattered skin vesicles and corneal ulcerations would not be seen.

(Choice B) Hematogenous dissemination of Candida a/bicans can cause fungal endophthalmitis. The fungus
typically seeds the highly vascular choroid then spreads to the retina and fluid chambers, manifesting as floaters
and progressively decreasing vision. Eye pain is rare until late in the disease process, and corneal ulceration does
not typically occur.

(Choices C and D) Herpes simplex virus is a common cause of keratitis and can occur in healthy adults, although
immunodeficiency increases the risk. Pseudomonas aeruginosa is a common cause of bacterial keratitis,
especially in contact lens users. Both can cause corneal ulceration, leading to eye pain, redness, and decreased

Block Time Elapsed: 00:05:18 ~ II;! rvo Cl @ 0


My Notebook Flashcards Feedback Suspend End Block

"' !!I 51549 (1).mp4


"' https://t.me/USMLEWorldStep1 Show all X
= Item ~3 of 30
Question Id: 107365
ee e ·•
■ t7' Mark
. • e - le e e . I• I • -e If: - -e J.• -
<:::
Previous
• - •
C>
Next
I • .. I I• • • I.• -e
~~
Full Screen
®
Tutorial
il
Lab Values
~-
Notes
....I
Calculator
,
Reverse Color
~
Text Zoom
©
Settings

unilateral forehead, side of the nose, periorbital area). A prodrome of systemic symptoms (eg, headache, malaise,
fever) can also occur.

Because CN V1 also conveys sensory innervation of the cornea (sensory limb of the corneal/blink reflex),
involvement can cause acute keratitis (ie, corneal inflammation) and corneal ulceration (eg, linear, branching
ulcerations on fluorescein stain). Involvement of the side or tip of the nose (ie, Hutchinson sign) correlates highly
with ocular involvement because the nasociliary branch of CN V1 innervates both the eye and the side/tip of the
nose.

{Choice A) Viral conjunctivitis is most commonly caused by adenovirus. It typically presents with monocular
conjunctiva! erythema and eye discomfort; scattered skin vesicles and corneal ulcerations would not be seen.

(Choice B) Hematogenous dissemination of Candida albicans can cause fungal endophthalmitis. The fungus
typically seeds the highly vascular choroid then spreads to the retina and fluid chambers, manifesting as floaters
and progressively decreasing vision. Eye pain is rare until late in the disease process, and corneal ulceration does
not typically occur.

(Choices C and D) Herpes simplex virus is a common cause of keratitis and can occur in healthy adults, although
immunodeficiency increases the risk. Pseudomonas aeruginosa is a common cause of bacterial keratitis,
especially in contact lens users. Both can cause corneal ulceration, leading to eye pain, redness, and decreased
vision. However, neither causes a vesicular, dermatomal rash.

Educational objective:
Herpes zoster ophthalmicus is caused by reactivation of varicella-zoster virus in the ophthalmic division of the
trigeminal nerve (CN V1). It is characterized by a painful, dermatomal rash and ocular involvement (eg, acute
keratitis, corneal ulceration). It most commonly occurs in elderly and immunosuppressed patients.

Block Time Elapsed: 00:05:18 ~ II;! rvo Cl @ 0


My Notebook Flashcards Feedback Suspend End Block

"' !!I 51549 (1).mp4


"' https://t.me/USMLEWorldStep1 Show all X
= Item ~3 of 30
Question Id: 107365
■ t7' Mark <
Previous
C>
Next
~~
Full Saeen
®
Tutorial
il
Lab Values

Notes
....I
Calculator
,
Reverse Color
~
Text Zoom
©
Settings

E Exhibit Display Bl x

Dermatomes of head & neck

(V

Foll
era
Cervical spinal
x Ophthalmic branch of
trigeminal nerve (V1)

ace nerves (dorsal rami)

sho
unil Auricular branch of
vagus nerve (X)
fev Maxillary branch of
trigeminal nerve (V2)
Bee
mvo Facial nerve (VII)

ulce
with
nos Cervical plexus
(ventral rami)
Mandibular branch of
(Ch trigeminal nerve (V3)
conj

(Ch ©UWOt'ld

typi ©. Zoom In 0. Zoom Out a Reset q;, New I lf-J Existi ng I!;! My Notebook
and

Block Time Elapsed: 00:05:18 ~ II;! rvo Cl @ 0


My Notebook Flashcards Feedback Suspend End Block

"' !!I 51549 (1).mp4


"' https://t.me/USMLEWorldStep1 Show all X
= Item ~3 of 30
Question Id: 107365
■ t7' Mark <
Previous
C>
Next
~~
Full Saeen
®
Tutorial
il
Lab Values

Notes
....I
Calculator
,
Reverse Color
~
Text Zoom
©
Settings

E Exhibit Display Bl x

Varicella-zoster virus (VZV) infection

Years later
(V Latency (dormant VZV) Reactivation of VZV by
aging, immunosuppression

Foll
era
ace
sho
...
vzv

unil
fev

Bee
mvo ..
ulce Migration to
senso,y
with • ganglion

nos

(Ch
conj Varicella (chickenpox) Herpes zoster (shingles)
Generalized vesicular rash Unilateral, linear vesicular rash
(Ch <0UWOrld

typi ©_ Zoom In 0_ Zoom Out C Reset Cf;, New I lf-J Existi ng (!;I My Notebook
and

Block Time Elapsed: 00:05:18 ~ II;! rvo Cl @ 0


My Notebook Flashcards Feedback Suspend End Block

,.,, !!I 51549 (1),mp4 ,.,,


https://t.me/USMLEWorldStep1 Show all X
= ltem~4of30
Question Id: 18~6
■ t7' Mark <:::
Previous
C>
Next
~~
Full Screen
®
Tutorial
il
Lab Values

Notes
....I
Calculator
,
Reverse Color
~
Text Zoom
©
Settings

A 14-year-old boy is brought to the clinic by his mother due to changes in vision. For the last few weeks, the
patient has been squinting when trying to read the interactive whiteboard in class and has had to sit closer to the
front of the room. He has also developed headaches. The patient has no chronic medical conditions and takes no
medications. Temperature is 37.2 C (99 F), blood pressure is 116/76 mm Hg, and pulse is 70/min. Examination of
the eyes shows equal and reactive pupils. Extraocular muscles are intact, and visual field testing reveals no
defects. Visual acuity shows 20/70 in both eyes. His ability to read close text is normal. Which of the following is
the most likely cause of this patient's symptoms?

0 A . Chronically increased intraocular pressure

0 8 . Disturbance in development of the visual cortex

0 C. Increased anterior-posterior diameter of the eyes

0 D. Loss of accommodating power of the lens

0 E. Refracted image focused posterior to the retina

Submit

Block Time Elapsed: 00:05:19 ~ II;! rvo Cl @ 0


My Notebook Flashcards Feedback Suspend End Block

"' !!I 51549 (1).mp4


"' https://t.me/USMLEWorldStep1 Show all X
= ltem~4of30
Question Id: 18~6
■ t7' Mark <:::
Previous
C>
Next
~~
Full Screen
®
Tutorial
il
Lab Values

Notes
....I
Calculator
,
Reverse Color
~
Text Zoom
©
Settings

A 14-year-old boy is brought to the clinic by his mother due to changes in vision. For the last few weeks, the
patient has been squinting when trying to read the interactive whiteboard in class and has had to sit closer to the
front of the room. He has also developed headaches. The patient has no chronic medical conditions and takes no
medications. Temperature is 37.2 C (99 F), blood pressure is 116/76 mm Hg, and pulse is 70/min. Examination of
the eyes shows equal and reactive pupils. Extraocular muscles are intact, and visual field testing reveals no
defects. Visual acuity shows 20/70 in both eyes. His ability to read close text is normal. Which of the following is
the most likely cause of this patient's symptoms?

O A . Chronically increased intraocular pressure (2%)

X ® 8 . Disturbance in development of the visual cortex (0%)

✓ Q C. Increased anterior-posterior diameter of the eyes (54%)

0 D. Loss of accommodating power of the lens (7°/o)

0 E. Refracted image focused posterior to the retina (35%)

Incorrect
Correct answer I 1,1 54% fi"\ 03 secs
I.!!!!. Answered correctly 1..::.J Time Spent
C

Explanation

Refractive errors

!!I 51549 (1).mp4


https://t.me/USMLEWorldStep1 Show all X
=
-
ltem24of30
Question Id: 18~6
■ r:,,
\ Mark
<
Previous
C>
Next
~~
Full Saeen
®
Tutorial
il
Lab Values
~-
Notes
.... 1
Calculator
,
Reverse Color
~
Text Zoom
©
Settings

C Exhibit Display Bl x
C
Refractive errors

Image focuses
E on retina

Normal

Image focuses
in front of ,ebna

Myopia

Image focuses
behind relina

Hyperopla
I
E
© UWorld

©. Zoom In 0_ Zoom Out C Reset Cf;, New I lf-J Existi ng (!;I My Notebook

!!I 51549 (1).mp4


https://t.me/USMLEWorldStep1 Show all X
:= ltem~4of30
Question Id: 18~6
■ t7' Mark <:::
Previous
C>
Next
~~
Full Screen
®
Tutorial
il
Lab Values
~•
Notes Calculator
,
Reverse Color
~
Text Zoom
©
Settings

This patient with decreased visual acuity and an otherwise normal ophthalmic examination likely has simple
myopia (nearsightedness) , a refractive error caused by an increased anterior-posterior diameter of the eye.
The risk is greatest for those with a positive family history or those of East Asian descent.

During the adolescent growth spurt, the axial length of the globe also grows rapidly, making the incidence of new-
onset myopia highest in this age group. As the anterior-posterior diameter increases, the focal point of a refracted
image lies anterior to the retina. For images to be seen clearly, they must be projected directly onto the retina;
therefore, the image displacement in myopia causes blurred vision of long-distance objects and decreased
visual acuity on examination. Vision of nearby objects is unaffected and appears clear in myopic patients. The
remainder of the examination, including extraocular movements, peripheral field testing, and funduscopic
examination, is normal.

Management is with prescription glasses with a diverging (concave) lens, which refocuses the refracted image onto
the retina. The degree of myopia usually stabilizes by early adulthood .

(Choice A) Glaucoma, most commonly due to chronically elevated intraocular pressure, is characterized by optic
neuropathy, as evidenced by optic cupping seen on funduscopic examination. Visual acuity may be affected but is
a late manifestation after progressive loss of peripheral vision, which is normal in this patient.

{Choice B) Untreated strabismus (misalignment of the eyes) in early childhood can cause unilateral visual cortex
suppression to avoid diplopia and eventually lead to amblyopia (decreased visual acuity). This patient's recent
symptom onset and normal extraocular movements make this diagnosis unlikely.

{Choice D) Presbyopia is a nonrefractive error that increases with age due to the loss of accommodating power of
the lens from decreased elasticity. Blurry vision of nearby objects typically develops in patients age >40.

(Choice E) Hyperopia (farsightedness) is a refractive error characterized by a reduced anterior-posterior diameter


e ■- - - • le ■ - e • ee I e ■- - • -1 11.1- e 1- l e 11-1 II - I I ■- - ■• I II • I

Block Time Elapsed: 00:05:21 ~ II;! rvo Cl @ 0


My Notebook Flashcards Feedback Suspend End Block

"' !!I 51549 (1).mp4


"' https://t.me/USMLEWorldStep1 Show all X
:= ltem~4of30
Question Id: 18~6
■ t7' Mark <:::
Previous
C>
Next
~~
Full Screen
®
Tutorial
il
Lab Values
~•
Notes Calculator
,
Reverse Color
~
Text Zoom
©
Settings
g g • y, y • •• y
therefore, the image displacement in myopia causes blurred vision of long-distance objects and decreased
visual acuity on examination . Vision of nearby objects is unaffected and appears clear in myopic patients. The
remainder of the examination, including extraocular movements, peripheral field testing, and funduscopic
examination, is normal.

Management is with prescription glasses with a diverging (concave) lens, which refocuses the refracted image onto
the retina. The degree of myopia usually stabilizes by early adulthood .

{Choice A) Glaucoma, most commonly due to chronically elevated intraocular pressure, is characterized by optic
neuropathy, as evidenced by optic cupping seen on funduscopic examination. Visual acuity may be affected but is
a late manifestation after progressive loss of peripheral vision, which is normal in this patient.

(Choice B) Untreated strabismus (misalignment of the eyes) in early childhood can cause unilateral visual cortex
suppression to avoid diplopia and eventually lead to amblyopia (decreased visual acuity). This patient's recent
symptom onset and normal extraocular movements make this diagnosis unlikely.

(Choice D) Presbyopia is a nonrefractive error that increases with age due to the loss of accommodating power of
the lens from decreased elasticity. Blurry vision of nearby objects typically develops in patients age >40.

(Choice E) Hyperopia (farsightedness) is a refractive error characterized by a reduced anterior-posterior diameter


of the eye, causing the focal point of the refracted image to be positioned posterior to the retina. In contrast to
myopia, hyperopia causes blurred vision of nearby objects and is corrected by a converging (convex) lens.

Educational objective:
Myopia (nearsightedness) is a refractive error in which the focal point of an image falls anterior to the retina due to
an increased anterior-posterior diameter of the eyes. Patients have difficulty seeing objects at a distance and have
normal near vision.

Block Time Elapsed: 00:05:21 ~ II;! rvo Cl @ 0


My Notebook Flashcards Feedback Suspend End Block

"' !!I 51549 (1).mp4


"' https://t.me/USMLEWorldStep1 Show all X
Glaucoma pathophysiology

Normal eye
Ciliarybody
& musde

Duri
ons
,ma
ther Optic nerve / -~ ~ - . i ! ,
Retina Normal fluid flow
VISU

rem Glaucoma
exa

Ma
the

(Ch
neu
a la
Blocked fluid flow

(Ch © UWo<td

sup ©_ Zoom In 0_ Zoom Out C Reset Cf;, New I lf-J Existi ng (!;I My Notebook
sym

(Choice D) Presbyopia is a nonrefractive error that increases with age due to the loss of accommodating power of
Block Time Elapsed: 00:05:21 ~ II;! rvo Cl @ 0
My Notebook Flashcards Feedback Suspend End Block

"' !!I 51549 (1),mp4


"' https://t.me/USMLEWorldStep1 Show all X
= ltem~4of30
Question Id: 18~6
- - ------
■ t7' Mark
- --- ---- - - -- - ----
<:::
Previous
-- --
C>
Next
-----
~~
Full Saeen
®
Tutorial
i l
Lab Values
~
Notes
· ....I
Calculator
,
Reverse Color
~
Text Zoom
©
Settings

Dur
Exhibit Display Bl x
ons
Strabismus
ima
ther
VISU

rem
exa

Ma
the

(Ch Normal eyes


neu
a la

(Ch
sup
sym

(Ch
the
Esotropia (strabismus)
(Ch lt>UWortd

my
©_ Zoom In 0_ zoom Out a Reset Cf;, New I lf-J Existing (!;I My Notebook

Block Time Elapsed: 00:05:21 ~ II;! rvo Cl @ 0


My Notebook Flashcards Feedback Suspend End Block

"' !!I 51549 (1).mp4


"' https://t.me/USMLEWorldStep1 Show all X
= ltem~4of30
Question Id: 18~6
■ t7' Mark <
Previous
C>
Next
~~
Full Saeen
®
Tutorial
i l
Lab Values
~
Notes
· ....I
Calculator
,
Reverse Color
~
Text Zoom
©
Settings
g g • y, y • •• y
ther
Exhibit Display Bl X
visu
rem Ocular accommodation and presbyopia
exa Relaxed Accommodated
__
,......... Presbyopia
,.........
_
Ma
Ciliary
the muscle

(Ch
Lens
neu
a la

(Ch
sup
sym

(Ch
Zonular
the I fibers

(Ch
oft
Ciliary muscle relaxes, Ciliary muscle contracts, Hardened lens is unable
myo zonular fibers tighten & relaxing the zonular fibers & to thicken upon ciliary
cause ffattening of the lens. causing thickening of the lens muscle contraction

Edu C)UWotld

Myo ©. Zoom In 0. Zoom O ut a Reset q;, New I lf-J Existi ng I!;! My Notebook
an i

Block Time Elapsed: 00:05:21 ~ ll;I rvo Cl @ 0


My Notebook Flashcards Feedback Suspend End Block

"' !!I 51549 (1).m p4


"' https://t.me/USMLEWorldStep1 Show all X
= ltem~4of30
Question Id: 18~6
■ t7' Mark <
Previous
C>
Next
~~
Full Saeen
®
Tutorial
il
Lab Values

Notes
....I
Calculator
,
Reverse Color
~
Text Zoom
©
Settings
g g • y, y • •• y
ther
Exhibit Display Bl x
visu
rem
exa

Ma Hyperopia (farsightedness)
the
Focal plane Focal plane

(Ch Converging
Lens lens
neu Cornea
a la '\.

(Ch
sup
Nearby
sym object

(Ch Uncorrected Corrected


the • Eye too short or optical power too low • Converging lens increases optical power
(Ch • Image forms behind foca l plane of retina • Image now forms on focal plane of retina

OOWorld

my

Ed
My ©_ Zoom In 0_ Zoom Out C Reset Cf;, New I lf-J Existi ng (!;I My Notebook

Block Time Elapsed: 00:05:21 ~ II;! rvo Cl @ 0


My Notebook Flashcards Feedback Suspend End Block

"' !!I 51549 (1).mp4


"' https://t.me/USMLEWorldStep1 Show all X
= ltem~5of30
Question Id: 7721
■ t7' Mark <:::
Previous
C>
Next
~~
Full Screen
®
Tutorial
il
Lab Values

Notes
....I
Calculator
,
Reverse Color
~
Text Zoom
©
Settings

A 67-year-old smoker comes to the office with a 2-week history of decreased vision in his right eye that he
describes as "blurry" and "distorted." The patient has been having vision problems over the past year, and these
have made it more difficult to drive and require that he use a bright light to read the newspaper. He has no history
of diabetes mellitus or hypertension. The patient uses an albuterol inhaler for occasional wheezing and shortness
of breath. Ophthalmologic examination of the right eye shows a grayish discoloration of the macula with areas of
adjacent hemorrhage. Which of the following should be specifically targeted in treatment of this patient's
condition?

0 A . CD20 lymphocyte glycoprotein

0 8 . Epidermal growth factor receptor

0 C. lnterleukin-2

0 D. Tumor necrosis factor-alpha

O E. Vascular endothelial growth factor

Submit

Block Time Elapsed: 00:05:23 ~ II;! rvo Cl @ 0


My Notebook Flashcards Feedback Suspend End Block

"' !!I 51549 (1).mp4


"' https://t.me/USMLEWorldStep1 Show all X
= ltem~5of30
Question Id: 7721
■ t7' Mark <:::
Previous
C>
Next
~~
Full Screen
®
Tutorial
il
Lab Values

Notes
....I
Calculator
,
Reverse Color
~
Text Zoom
©
Settings

A 67-year-old smoker comes to the office with a 2-week history of decreased vision in his right eye that he
describes as "blurry" and "distorted." The patient has been having vision problems over the past year, and these
have made it more difficult to drive and require that he use a bright light to read the newspaper. He has no history
of diabetes mellitus or hypertension. The patient uses an albuterol inhaler for occasional wheezing and shortness
of breath. Ophthalmologic examination of the right eye shows a grayish discoloration of the macula with areas of
adjacent hemorrhage. Which of the following should be specifically targeted in treatment of this patient's
condition?

0 A . CD20 lymphocyte glycoprotein (1%)

X ® 8 . Epidermal growth factor receptor (6°/o)

0 C. lnterleukin-2 (3%)

0 D. Tumor necrosis factor-alpha (6%)

✓ O E. Vascular endothelial growth factor (82%)

Incorrect
Correct answer I 1,1 82% fi"\ 05 secs i===! 2023
I.!!!!. Answered correctly 1..::.J Time Spent l.:.:.!.J Version
E

Explanation

This patient likely has age-related macular degeneration (AMO), the leading cause of blindness in industrialized
countries. AMO occurs in genetically predisposed individuals as a result of advancing age and environmental
factors (eg, smoking) and is classified into dry and wet subtypes. Dry AMO is characterized by gradual vision loss
Block Time Elapsed: 00:05:26 ~ II;! rvo Cl @ 0
My Notebook Flashcards Feedback Suspend End Block

"' !!I 51549 (1).mp4


"' https://t.me/USMLEWorldStep1 Show all X
:= ltem~5of30
Question Id: 7721
■ t7' Mark <:::
Previous
C>
Next
~~
Full Screen
®
Tutorial
il
Lab Values
~•
Notes Calculator
,
Reverse Color
~
Text Zoom
©
Settings
This patient likely has age-related macular degeneration (AMO), the leading cause of blindness in industrialized
countries. AMD occurs in genetically predisposed individuals as a result of advancing age and environmental
factors (eg, smoking) and is classified into dry and wet subtypes. Dry AMO is characterized by gradual vision loss
in one or both eyes and can cause difficulty with driving/reading. The condition likely results from chronic oxidative
damage to the retinal pigment epithelium and choriocapillaris, leading to subretinal inflammation with abnormal
extracellular matrix formation (eg, confluent drusen, basement membrane thickening). These changes appear on
funduscopy as subretinal drusen deposits with pigment abnormalities.

Progressive extracellular matrix accumulation can eventually result in retinal hypoxia, which stimulates local
vascular endothelial growth factor (VEGF) production and causes subretinal neovascularization with formation of
leaky vessels. This condition is termed wet AMO and presents with acute vision loss (days to weeks) with
metamorphopsia (distortion of straight lines). Funduscopy shows a grayish-green subretinal discoloration with
adjacent fluid/hemorrhage.

Patients with dry or wet AMD may benefit from antioxidant vitamins and zinc, and smokers should receive
smoking cessation counseling to prevent disease progression . Wet AMD usually requires specific treatment with
VEGF inhibitors (eg , ranibizumab, bevacizumab).

(Choice A) Therapy against the CD20 glycoprotein on B cells (eg, the monoclonal antibody rituximab) is used to
treat various lymphomas and active rheumatoid arthritis.

(Choice B) Epidermal growth factor receptor inhibitors (eg, erlotinib, gefitinib) are used to treat advanced non-
small cell lung cancer.

(Choice C) Anti-interleukin-2 therapy is primarily used for immunosuppression in organ transplant patients and in
conditions such as graft versus host disease.

(Choice D) Inhibitors of tumor necrosis factor-alpha are used in many inflammatory autoimmune conditions, such

Block Time Elapsed: 00:05:26 ~ II;! rvo Cl @ 0


My Notebook Flashcards Feedback Suspend End Block

"' !!I 51549 (1).mp4


"' https://t.me/USMLEWorldStep1 Show all X
= ltem~5of30
Question Id: 7721
■ t7' Mark <:::
Previous
C>
Next
~~
Full Screen
®
Tutorial
il
Lab Values
~-
Notes
....I
Calculator
,
Reverse Color
~
Text Zoom
©
Settings

Progressive extracellular matrix accumulation can eventually result in retinal hypoxia, which stimulates local
vascular endothelial growth factor (VEGF) production and causes subretinal neovascularization with formation of
leaky vessels. This condition is termed wet AMO and presents with acute vision loss (days to weeks) with
metamorphopsia (distortion of straight lines). Funduscopy shows a grayish-green subretinal discoloration with
adjacent fluid/hemorrhage.

Patients with dry or wet AMO may benefit from antioxidant vitamins and zinc, and smokers should receive
smoking cessation counseling to prevent disease progression . Wet AMO usually requires specific treatment with
VEGF inhibitors (eg , ranibizumab, bevacizumab).

(Choice A) Therapy against the CO20 glycoprotein on B cells (eg, the monoclonal antibody rituximab) is used to
treat various lymphomas and active rheumatoid arthritis.

(Choice B) Epidermal growth factor receptor inhibitors (eg, erlotinib, gefi tinib) are used to treat advanced non-
small cell lung cancer.

(Choice C) Anti-interleukin-2 therapy is primarily used for immunosuppression in organ transplant patients and in
conditions such as graft versus host disease.

(Choice D) Inhibitors of tumor necrosis factor-alpha are used in many inflammatory autoimmune conditions, such
as rheumatoid arthritis, inflammatory bowel disease, and seronegative spondyloarthropathies.

Educational objective:
Wet age-related macular degeneration is characterized by retinal neovascularization due to increased vascular
endothelial growth factor (VEGF) levels. Patients typically have acute vision loss and metamorphopsia with
funduscopy showing a grayish-green subretinal membrane and/or subretinal hemorrhage. Treatment includes
smoking cessation and VEGF inhibitor therapy (eg, ranibizumab, bevacizumab).

Block Time Elapsed: 00:05:26 ~ II;! rvo Cl @ 0


My Notebook Flashcards Feedback Suspend End Block

"' !!I 51549 (1).mp4


"' https://t.me/USMLEWorldStep1 Show all X
= ltem~5of30
Question Id: 7721
■ t7' Mark <
Previous
C>
Next
~~
Full Saeen
®
Tutorial
il
Lab Values
~ ·
Notes
....I
Calculator
,
Reverse Color
~
Text Zoom
©
Settings

Thi
cou
fact
mo
da

Pro
vas
lea

adj

Pati

(Ch
trea

(Ch ©_ Zoom In 0_ Zoom Out C Reset C:::, New I lf-J Existi ng (!;I My Notebook

(Choice C) Anti-interleukin-2 therapy is primarily used for immunosuppression in organ transplant patients and in

Block Time Elapsed: 00:05:26 ~ II;! rvo Cl @ 0


My Notebook Flashcards Feedback Suspend End Block

"' !!I 51549 (1).mp4


"' https://t.me/USMLEWorldStep1 Show all X
= ltem~5of30 ■ t7' Mark < C> ~~ ® il ~· ....I , ~ ©
.- - .
Question Id: 7721 Previous Next Full Saeen Tutorial Lab Values Notes Calculator Reverse Color Text Zoom Settings

Exhibit Display Bl X

AMO, Calcified Orusen (#1) AMO, Exudat,ve


cou
Age-related macular degeneration (AMO)
fact
mo
da

Pro

me
adja

Pati

(Ch
trea
O\JWOfl d

(Ch ©. Zoom In 0. Zoom Out a Reset q, New I lf-J Existi ng I!;! My Notebook

(Choice C) Anti-interleukin-2 therapy is primarily used for immunosuppression in organ transplant patients and in

Block Time Elapsed: 00:05:26 ~ ll;I rvo Cl @ 0


My Notebook Flashcards Feedback Suspend End Block

"' !!I 51549 (1).mp4


"' https://t.me/USMLEWorldStep1 Show all X
= ltem~5of30
Question Id: 7721
■ t7' Mark <
Previous
C>
Next
~~
Full Saeen
®
Tutorial
il
Lab Values

Notes
....I
Calculator
,
Reverse Color
~
Text Zoom
©
Settings

da

Pro
vas
lea

adj

Pati

(Ch
trea

(Ch
sm

(Ch
con ©_ Zoom In 0_ Zoom Out C Reset Cf;, New I lf-J Existi ng (!;I My Notebook

(Ch
h t 'd rth 'f • fl t b Id' d •r • d I rth • th'
Block Time Elapsed: 00:05:26 ~ II;! rvo Cl @ 0
My Notebook Flashcards Feedback Suspend End Block

"' !!I 51549 (1).mp4


"' https://t.me/USMLEWorldStep1 Show all X
= Item ~6 of 30
Question Id: 1356
■ t7' Mark <:::
Previous
C>
Next
~~
Full Screen
®
Tutorial
il
Lab Values

Notes
....I
Calculator
,
Reverse Color
~
Text Zoom
©
Settings

A 65-year-old man comes to the office due to bilateral visual difficulty that has progressed over the last year.
Funduscopy is shown in the exhibit. Which of the following deficits is most likely to be seen on visual field
examination?

0 A . Arcuate scotoma

0 8 . Binasal hemianopia

0 C. Bitemporal hemianopia

0 D. Central scotoma

0 E. Homonymous hemianopia

Submit

Block Time Elapsed: 00:05:27 ~ II;! rvo Cl @ 0


My Notebook Flashcards Feedback Suspend End Block

"' !!I 51549 (1).mp4


"' https://t.me/USMLEWorldStep1 Show all X
= ltem~6of30
Question Id: 1356
■ t7' Mark <
Previous
C>
Next
~~
Full Saeen
®
Tutorial
j!
Lab Values
~ -
Notes
....I
Calculator
,
Reverse Color
~
Text Zoom
©
Settings

A6

©_ Zoom In 0, Zoom Out C Reset Cf;, New I lf-J Existi ng (!;I My Notebook

Block Time Elapsed: 00:05:33 ~ II;! rvo Cl @ 0


My Notebook Flashcards Feedback Suspend End Block

"' !!I 51549 (1).mp4


"' https://t.me/USMLEWorldStep1 Show all X
= Item ~6 of 30
Question Id: 1356
■ t7' Mark <:::
Previous
C>
Next
~~
Full Screen
®
Tutorial
il
Lab Values

Notes
....I
Calculator
,
Reverse Color
~
Text Zoom
©
Settings

A 65-year-old man comes to the office due to bilateral visual difficulty that has progressed over the last year.
Funduscopy is shown in the exhibit. Which of the following deficits is most likely to be seen on visual field
examination?

0 A . Arcuate scotoma (20%)

X ® 8 . Binasal hemianopia (3%>)

0 C. Bitemporal hemianopia (6%>)

✓ Q D. Central scotoma (63%>)

0 E. Homonymous hemianopia (5%)

Incorrect
Correct answer I 1,1 63% f'i\ 12 secs 2023
D
I.!!.!!. Answered correctly \::.J Time Spent Version

Explanation

Age-related macular degeneration (AMD)

!!I 51549 (1).mp4


https://t.me/USMLEWorldStep1 Show all X
Age-related macular degeneration (AMO)
C
D

CUWorld

©_ Zoom In 0_ Zoom Out C Reset Cf;, New I lf-J Existi ng (!;I My Notebook

!!I 51549 (1).mp4


https://t.me/USMLEWorldStep1 Show all X
= Item ~6 of 30
Question Id: 1356
■ t7' Mark <:::
Previous
C>
Next
~~
Full Screen
®
Tutorial
il
Lab Values
~-
Notes
....I
Calculator
,
Reverse Color
~
Text Zoom
©
Settings

This patient has yellow deposits (ie, drusen) clustered around the macula, which are strongly suspicious for age-
related macular degeneration (AMD). Lesions that impact the macula can lead to loss of a portion of the central
visual field, which is called a central scotoma. A scotoma is any visual defect surrounded by a relatively
unimpaired field of vision. They most often occur due to pathology involving parts of the retina or optic nerve.

The macula is located near the center of the retina. It is characterized histologically by the presence of densely
packed cones, few overlying cells, and no blood vessels. Each macular cone synapses to a single bipolar cell ,
which synapses to a single ganglion cell (in contrast to the peripheral retina, where multiple cones converge onto a
single ganglion cell). Because of this arrangement, visual acuity in the macula, and particularly the fovea, is
greater than in any other area of the retina. Loss of vision in this area is particularly debilitating, and AMD is the
most common cause of blindness in patients >age 50 in the United States.

(Choice A) An arcuate scotoma is an arc-shaped visual field defect extending from the blind spot (the area of the
optic nerve head that does not process visual information) that follows the bending course of the retinal nerve
fibers. It most classically occurs due to glaucoma; funduscopic examination often shows an increased cup/disc
ratio.

(Choice B) Binasal hemianopia is a loss of vision in the nasal fields of both eyes. This information does not cross
at the optic chiasm but travels at its lateral edge. For a lesion to produce binasal hemianopia, it would have to
impact the lateral optic chiasm on both sides.

(Choice C) Bitemporal hemianopia is a loss of vision in the temporal fields of both eyes. Fibers from the nasal
retina cross the midline at the optic chiasm, so lesions that compress the center of the optic chiasm (eg, pituitary
adenoma) can cause bitemporal hemianopia .

(Choice E) Homonymous hemianopia is a loss of vision in the same side of the visual field in both eyes. It occurs
with lesions posterior to the optic chiasm, which would not lead to changes in the funduscopic examination.

Block Time Elapsed: 00:05:38 ~ II;! rvo Cl @ 0


My Notebook Flashcards Feedback Suspend End Block

"' !!I 51549 (1).mp4


"' https://t.me/USMLEWorldStep1 Show all X
= I
Item ~6 of 30
Question Id: 1356
e. -1 - I e
■ t7' Mark
' e -, I I - I I 11- I I. I llt
<:::
Previous
tit I
C>
Next
I. I - - . I II - ,-
~~
Full Screen
®
Tutorial
il
Lab Values
~-
Notes
....I
Calculator
,
Reverse Color
~
Text Zoom
©
Settings

The macula is located near the center of the retina. It is characterized histologically by the presence of densely
packed cones, few overlying cells, and no blood vessels. Each macular cone synapses to a single bipolar cell ,
which synapses to a single ganglion cell (in contrast to the peripheral retina, w here multiple cones converge onto a
single ganglion cell). Because of this arrangement, visual acuity in the macula, and particularly the fovea, is
greater than in any other area of the retina. Loss of vision in this area is particularly debilitating, and AMO is the
most common cause of blindness in patients >age 50 in the United States.

(Choice A) An arcuate scotoma is an arc-shaped visual field defect extending from the blind spot (the area of the
optic nerve head that does not process visual information) that follows the bending course of the retinal nerve
fibers. It most classically occurs due to glaucoma; funduscopic examination often shows an increased cup/disc
ratio.

(Choice B) Binasal hemianopia is a loss of vision in the nasal fields of both eyes. This information does not cross
at the optic chiasm but travels at its lateral edge. For a lesion to produce binasal hemianopia, it would have to
impact the lateral optic chiasm on both sides.

(Choice C) Bitemporal hemianopia is a loss of vision in the temporal fields of both eyes. Fibers from the nasal
retina cross the midline at the optic chiasm, so lesions that compress the center of the optic chiasm (eg, pituitary
adenoma) can cause bitemporal hemianopia.

(Choice E) Homonymous hemianopia is a loss of vision in the same side of the visual field in both eyes. It occurs
with lesions posterior to the optic chiasm, which would not lead to changes in the funduscopic examination.

Educational objective:
A scotoma is a discrete visual field defect. Central scotomas are often caused by age-related macular
degeneration that leads to the deposition of drusen clustered around the macula.

Block Time Elapsed: 00:05:38 ~ II;! rvo Cl @ 0


My Notebook Flashcards Feedback Suspend End Block

"' !!I 51549 (1).mp4


"' https://t.me/USMLEWorldStep1 Show all X
= Item ~7 of 30
Question Id: 106633
■ t7' Mark <:::
Previous
C>
Next
~~
Full Screen
®
Tutorial
il
Lab Values

Notes
....I
Calculator
,
Reverse Color
~
Text Zoom
©
Settings

A 66-year-old hospitalized woman is evaluated due to progressive pain and decreased vision in her left eye. The
patient was admitted 2 weeks ago due to acute diverticulitis with microperforation and localized abscess. CT-
guided abscess drainage was performed, and she has been receiving antibiotics and parenteral nutrition via a
central venous catheter. The patient has no other medical conditions. Temperature is 38.3 C (100.91 F). Left eye
examination shows chorioretinitis and vitreous abscess. Urgent partial vitrectomy is performed, and anti microbial
therapy is administered . Culture of the vitreous humor grows Candida a/bicans. Which of the following is the most
likely route of pathogen invasion during the pathogenesis of this patient's ocular infection?

0 A . Direct extension from anterior chamber

0 8 . Direct extension from paranasal sinuses

0 C. Hematogenous spread

0 D. Retrograde transfer in cranial nerve

0 E. Spread via lymphatic channels

Submit

Block Time Elapsed: 00:05:39 ~ II;! rvo Cl @ 0


My Notebook Flashcards Feedback Suspend End Block

"' !!I 51549 (1).mp4


"' https://t.me/USMLEWorldStep1 Show all X
= Item ~7 of 30
Question Id: 106633
■ t7' Mark <:::
Previous
C>
Next
~~
Full Screen
®
Tutorial
il
Lab Values

Notes
....I
Calculator
,
Reverse Color
~
Text Zoom
©
Settings

A 66-year-old hospitalized woman is evaluated due to progressive pain and decreased vision in her left eye. The
patient was admitted 2 weeks ago due to acute diverticulitis with microperforation and localized abscess. CT-
guided abscess drainage was performed, and she has been receiving antibiotics and parenteral nutrition via a
central venous catheter. The patient has no other medical conditions. Temperature is 38.3 C (100.91 F). Left eye
examination shows chorioretinitis and vitreous abscess. Urgent partial vitrectomy is performed, and anti microbial
therapy is administered . Culture of the vitreous humor grows Candida a/bicans. Which of the following is the most
likely route of pathogen invasion during the pathogenesis of this patient's ocular infection?

O A . Direct extension from anterior chamber (10%>)

X® 8 . Direct extension from paranasal sinuses (19%)

✓O C. Hematogenous spread (62%)

0 D. Retrograde transfer in cranial nerve (1°/o)

0 E. Spread via lymphatic channels (5%)

Incorrect
Correct answer I 1,1 62% fi"\ 03 secs i===! 2023
I.!!!!. Answered correctly 1..::.J Time Spent l.:.:.!.J Version
C

Explanation

Candida chorioretinitis & endophthalmitis


• Transient/persistent fungemia -> hematogenous dissemination to the choroid ->

Block Time Elapsed: 00:05:41 ~ II;! rvo Cl @ 0


My Notebook Flashcards Feedback Suspend End Block

"' !!I 51549 (1).mp4


"' https://t.me/USMLEWorldStep1 Show all X
= Item ~7 of 30
Question Id: 106633
■ t7' Mark <:::
Previous
C>
Next
~~
Full Screen
®
Tutorial
il
Lab Values
~-
Notes
....I
Calculator
,
Reverse Color
~
Text Zoom
©
Settings

Candida chorioretinitis & endophthalmitis


• Transient/persistent fungemia - hematogenous dissemination to the choroid -
invasion of the retina & fluid chambers
Epidemiology
• Risk factors: indwelling central catheter, TPN, broad-spectrum antibiotic use,
gastrointestinal surgery & immunocompromise

• Usually unilateral floaters & progressive vision loss


Manifestations • Pain rare until late in disease process
• Fever may be absent & patient may otherwise feel well

• Blood cultures (often negative)


Diagnosis
• Vitreous fl uid sampling & culture

TPN = total parenteral nutrition.

This patient has Candida endophthalmitis, a purulent infection of the structures and (usually) fl uid chambers of
the eye. Although molds such as Aspergil!us or Fusarium generally spread to the eye via direct extension following
surgery or trauma, yeasts such as Candida typically infect the eye via hematogenous dissemination in the
setting of fungemia. Candida first seeds the highly vascular choroid layer and then spreads to the retina and fluid
chambers.

Most cases occur in hospitalized patients with a central venous catheter, which allows commensal Candida
organisms from the skin to enter the bloodstream . The risk is further increased by total parenteral nutrition ,
broad-spectrum antibiotic therapy, recent abdominal surgery, and immunocompromise (eg, neutropenia,
glucocorticoid therapy).

Manifestations of Candida endophthalmitis are often unilateral and include floaters and a progressive loss in

Block Time Elapsed: 00:05:41 ~ II;! rvo Cl @ 0


My Notebook Flashcards Feedback Suspend End Block

"' !!I 51549 (1).mp4


"' https://t.me/USMLEWorldStep1 Show all X
=
-
ltem~7of30
Question Id: 106633
■ t7' Mark <:::
Previous
C>
Next
~~
Full Screen
®
Tutorial
il
Lab Values
~•
Notes Calculator
,
Reverse Color
~
Text Zoom
©
Settings

Most cases occur in hospitalized patients with a central venous catheter, which allows commensal Candida
organisms from the skin to enter the bloodstream . The risk is further increased by total parenteral nutrition ,
broad-spectrum antibiotic therapy, recent abdominal surgery, and immunocompromise (eg, neutropenia,
glucocorticoid therapy).

Manifestations of Candida endophthalmitis are often unilateral and include floaters and a progressive loss in
vision . Eye pain does not typically occur until late in the disease course. The patient may feel well and be
otherwise asymptomatic or may have systemic symptoms (eg, fever) due to fungemia. Blood cultures should be
obtained, but vitreous fluid sampling is usually required for diagnosis.

(Choice A) Direct extension via the anterior chamber can occur in the setting of a corneal infection following
trauma or surgery. It is rare for Candida to otherwise spread via the anterior chamber.

(Choice B) Rhizopus is a mold that affects primarily those with immunocompromise and diabetes mellitus. It first
attacks the paranasal sinuses and then spreads into adjacent tissue (eg, orbit, eye). This is not a typical pattern for
Candida.

(Choice D) Herpes simplex virus and varicella-zoster virus can reactivate in the optic nerve root and travel in a
retrograde fashion to infect the retina, leading to retinal vasculitis or acute retinal necrosis. Candida does not
spread in this fashion.

(Choice E) Lymphatic spread is common with many atypical bacteria and some endemic fungi. However,
Candida does not usually spread to the eye through the lymphatic system.

Educational objective:
Candida endophthalmitis typically occurs due to hematogenous dissemination to the choroid layer in the setting of
fungemia. Most cases arise in hospitalized patients with indwelling central catheters. Patients usually have
unilateral eye floaters and progressive vision loss.

Block Time Elapsed: 00:05:41 ~ II;! rvo Cl @ 0


My Notebook Flashcards Feedback Suspend End Block

"' !!I 51549 (1).mp4


"' https://t.me/USMLEWorldStep1 Show all X
= Item ~8 of 30
Question Id: 1322
■ t7' Mark <:::
Previous
C>
Next
~~
Full Screen
®
Tutorial
il
Lab Values

Notes
....I
Calculator
,
Reverse Color
~
Text Zoom
©
Settings

A 62-year-old woman is brought to the emergency department because of acute chest pain. She also complains of
diaphoresis, nausea, and lightheadedness. Her blood pressure is 90/60 mm Hg and pulse is 42/min. An ECG
reveals ST-segment elevation in the inferior leads. The appropriate therapy is initiated , incl uding a medication to
treat her bradycardia. After initial treatment, her blood pressure is 120/70 mm Hg and pulse is 76/min. However,
she now complains of severe right-sided eye pain. Which of the following is the most likely cause for this patient's
eye pain?

0 A . lridocyclitis

0 8 . Retinal artery thrombosis

0 C. Acute conjunctivitis

0 D. Glaucoma

0 E. Cataract

0 F. Retinal vein thrombosis

0 G. Retinal detachment

0 H. Vitreous hemorrhage

Submit

Block Time Elapsed: 00:05:44 ~ II;! rvo Cl @ 0


My Notebook Flashcards Feedback Suspend End Block

"' !!I 51549 (1).mp4


"' https://t.me/USMLEWorldStep1 Show all X
= Item ~8 of 30
Question Id: 1322
■ t7' Mark <:::
Previous
C>
Next
~~
Full Screen
®
Tutorial
il
Lab Values

Notes
....I
Calculator
,
Reverse Color
~
Text Zoom
©
Settings

A 62-year-old woman is brought to the emergency department because of acute chest pain. She also complains of
diaphoresis, nausea, and lightheadedness. Her blood pressure is 90/60 mm Hg and pulse is 42/min. An ECG
reveals ST-segment elevation in the inferior leads. The appropriate therapy is initiated , including a medication to
treat her bradycardia. After initial treatment, her blood pressure is 120/70 mm Hg and pulse is 76/min . However,
she now complains of severe right-sided eye pain . Which of the following is the most likely cause for this patient's
eye pain?

0 A . lridocyclitis (2%>)

X ® 8 . Retinal artery thrombosis (18%)

0 C. Acute conjunctivitis (0%)

✓ Q D. Glaucoma (60%>)

0 E. Cataract (0%)

0 F. Retinal vein thrombosis (6°/o)

0 G. Retinal detachment (4%)

0 H. Vitreous hemorrhage (6%)

Incorrect
Correct answer I 1,1 60% fl\ 06 secs 2023
I..!!.!.!. Answered correctly \.:.;Time Spent Version
D

Explanation

Block Time Elapsed: 00:05:47 ~ II;! rvo Cl @ 0


My Notebook Flashcards Feedback Suspend End Block

"' !!I 51549 (1).mp4


"' https://t.me/USMLEWorldStep1 Show all X
= Item ~8 of 30
Question Id: 1322
■ t7' Mark <
Previous
C>
Next
~~
Full Screen
®
Tutorial
il
Lab Values

Notes
....I
Calculator
,
Reverse Color
~
Text Zoom
©
Settings

ST-segment elevation in the inferior leads is diagnostic of an inferior myocardial infarction (Ml). Inferior Mis are
often due to blockage of the right coronary artery, the artery usually responsible for sinoatrial (SA) and
atrioventricular (AV) node perfusion . Thus, inferior Mis are often associated with bradycardia. Atropine blocks
vagal influence on the SA and AV nodes and is effective in increasing heart rate in such patients. Atropine's side
effects are due to muscarinic receptor blockade in other organs. In the eye, atropine causes mydriasis, resulting in
narrowing of the anterior chamber angle and diminished outflow of aqueous humor. This can precipitate angle-
closure glaucoma in patients with shallow anterior chambers or higher than normal intraocular pressures. Acute
closed-angle glaucoma presents with unilateral severe eye pain and visual disturbances (eg, halos). Individuals of
Inuit and Asian descent are at increased risk of angle-closure glaucoma.

The clinical indications for anticholinergic medications are summarized in the table.

System Effect Clinical use


CNS Sedation Motion sickness (scopolami ne)
In toxic doses: psychosis Parkinson's disease (benztropine)
Eye Mydriasis (dilated pupil) Ophthalmologic examination
Cycloplegia (paralysis of Treatment of uveitis
accommodation) Contraindicated in alaucoma!
Heart Tachycardia Sinus bradycardia
Heart block (atropine)
Secretions t lacrimation, sweating, salivation, Premedication before anesthesia
GI and bronchial secretions (atrooine. scooolami nel
Bladder Detrusor relaxation , delays voiding Neurogenic (spastic) bladder
(oxybutyni n)
Bronchi Bronc hodi latati on Asthma and COPD {ipratropi um)

(Choice A) Anticholinergic medications (cycloplegics) reduce pain and prevent the formation of adhesions in
iridocyclitis (uveitis). Atropine does not precipitate iridocyclitis.

(Choices B, E, F, G, and H) These conditions present with either loss of vision or visual disturbances. They are

Block Time Elapsed: 00:05:47 ~ II;! rvo Cl @ 0


My Notebook Flashcards Feedback Suspend End Block

"' !!I 51549 (1).mp4


"' https://t.me/USMLEWorldStep1 Show all X
= Item ~8 of 30
Question Id: 1322
■ t7' Mark <:::
Previous
C>
Next
~~
Full Saeen
®
Tutorial
i l
Lab Values
~ ·
Notes
....I
Calculator
,
Reverse Color
~
Text Zoom
©
Settings

ST-

atri
vag
effe
narr
clos System Effect Clinical use
clos CNS Sedation Motion sickness (scopolamine)
In toxic doses: psychosis Parkinson's disease (benztropine)
lnui Eye Mydriasis (dilated pupil) Ophthalmologic examination
Cycloplegia (paralysis of Treatment of uveitis
The accommodati on) Contraindicated in alaucoma!
Heart Tachycardia Sinus bradycardia
Heart block (atropine)
Secretions t lacrimation, sweating, salivation, Premedication before anesthesia
GI and bronchial secretions (atropine , scopolaminel
Bladder Detrusor relaxation, delays voiding Neurogenic (spastic) bladder
(oxyb utyni n)
Bronchi Bronc hodilatati on Asthma and COPD {ipratropi uml

(Ch ©_ Zoom In 0_ Zoom Out C Reset Cf;, New I lf-J Existi ng (!;I My Notebook
irid

(Choices B, E, F, G, and H) These conditions present with either loss of vision or visual disturbances. They are

Block Time Elapsed: 00:05:47 ~ II;! rvo Cl @ 0


My Notebook Flashcards Feedback Suspend End Block

,,,, !!I 51549 (1),mp4 ,,,,


https://t.me/USMLEWorldStep1 Show all X
= Item ~8 of 30
Question Id: 1322
■ t7' Mark <
Previous
C>
Next
~~
Full Screen
®
Tutorial
il
Lab Values

Notes
....I
Calculator
,
Reverse Color
~
Text Zoom
©
Settings

narrowing of the anterior chamber angle and diminished outflow of aqueous humor. This can precipitate angle-
closure glaucoma in patients with shallow anterior chambers or higher than normal intraocular pressures. Acute
closed-angle glaucoma presents with unilateral severe eye pain and visual disturbances (eg, halos). Individuals of
Inuit and Asian descent are at increased risk of angle-closure glaucoma.

The clinical indications for anticholinergic medications are summarized in the table.

System Effect Clinical use


CNS Sedation Motion sickness (scopolami ne)
In toxic doses: psychosis Parkinson's disease (benztropine)
Eye Mydriasis (dilated pupil) Ophthalmologic examination
Cycloplegia (paralysis of Treatment of uveitis
accommodation) Contraindicated in alaucoma!
Heart Tachycardia Sinus bradycardia
Heart block (atropine)
Secretions t lacrimation, sweating, salivation, Premedication before anesthesia
GI and bronchial secretions (atropine. scooolamine)
Bladder Detrusor relaxation, delays voiding Neurogenic (spastic) bladder
(oxybutyni n)
Bronchi Bronc hodi latati on Asthma and COPD (ioratrooi uml

(Choice A) Anticholinergic medications (cycloplegics) reduce pain and prevent the formation of adhesions in
iridocyclitis (uveitis). Atropine does not precipitate iridocyclitis.

(Choices 8, E, F, G, and H) These conditions present with either loss of vision or visual disturbances. They are
usually not painful and would not be affected by anticholinergic drugs.

Educational objective:
Atropine is indicated for the treatment of bradycardia as it decreases vagal influence on the SA and AV nodes. A
common side effect is increased intraocular pressure. It may precipitate acute closed-angle glaucoma in
susceptible individuals.

Block Time Elapsed: 00:05:47 ~ II;! rvo Cl @ 0


My Notebook Flashcards Feedback Suspend End Block

"' !!I 51549 (1).mp4


"' https://t.me/USMLEWorldStep1 Show all X
= Item ~9 of 30
Question Id: 13554
■ t7' Mark <:::
Previous
C>
Next
~~
Full Screen
®
Tutorial
il
Lab Values

Notes
....I
Calculator
,
Reverse Color
~
Text Zoom
©
Settings

A 62-year-old woman comes to the emergency department due to acute eye pai n. She has a history of seasonal
allergies and developed itchy, watery, red eyes after working outdoors in her garden. The patient used over-the-
counter eye drops to treat her allergy symptoms, and several hours later, she began experiencing severe right eye
pain and headache. On examination , the right eye appears red with a hazy cornea and dilated pupil that responds
poorly to light. Palpation of the globes reveals notable firmness on the right compared to the left. Visual acuity in
the affected eye is severely diminished . Tonometry reveals elevated intraocular pressure. W hich of the following
medications was most likely used by this patient?

0 A . Alpha-adrenergic agonist

0 8 . Beta blocker

0 C. Mast cell stabilizer

0 D. Nonsteroidal anti-inflammatory drug

0 E. Prostaglandin analogue

Submit

Block Time Elapsed: 00:05:48 ~ II;! rvo Cl @ 0


My Notebook Flashcards Feedback Suspend End Block

"' !!I 51549 (1).mp4


"' https://t.me/USMLEWorldStep1 Show all X
= Item ~9 of 30
Question Id: 13554
■ t7' Mark <:::
Previous
C>
Next
~~
Full Screen
®
Tutorial
il
Lab Values

Notes
....I
Calculator
,
Reverse Color
~
Text Zoom
©
Settings

A 62-year-old woman comes to the emergency department due to acute eye pai n. She has a history of seasonal
allergies and developed itchy, watery, red eyes after working outdoors in her garden. The patient used over-the-
counter eye drops to treat her allergy symptoms, and several hours later, she began experiencing severe right eye
pain and headache. On examination , the right eye appears red with a hazy cornea and dilated pupil that responds
poorly to light. Palpation of the globes reveals notable firmness on the right compared to the left. Visual acuity in
the affected eye is severely diminished . Tonometry reveals elevated intraocular pressure. W hich of the following
medications was most likely used by this patient?

✓ 0 A . Alpha-adrenergic agonist (70%)

x ® 8 . Beta blocker (8%>)

0 C. Mast cell stabilizer (5°/o)

0 D. Nonsteroidal anti-inflammatory drug (6%)

O E. Prostaglandin analogue (9%)

Incorrect
Correct answer I 1,1 70% fi"\ 05 secs i===! 2023
I.!!!!. Answered correctly 1..::.J Time Spent l.:.:.!.J Version
A

Explanation

Angle-closure glaucoma

Block Time Elapsed: 00:05:52 ~ - -·- - II;!


My Notebook
rvo
Flashcards
Cl
Feedback
@
Suspend
0
End Block

" !!I 51549 (1).mp4


" https://t.me/USMLEWorldStep1 Show all X
Angle-closure glaucoma

C Normal eye
Ciliary body
A &muscle

Lens

Drainage
channel

Normal fluid flow

Glaucoma

pressure

Blocke<I
drainage
channel
Optic neNe
damage Blocked fluid flow

©UWor1d

©_ Zoom In 0_ Zoom Out C Reset Cf;, New I lf-J Existi ng (!;I My Notebook

Block Time Elapsed: 00:05:52


. . ,,.
~
- · ,_ _ II;!
My Notebook
rvo
Flashcards
Cl
Feedback
@
Suspend
0
End Block

"' !!I 51549 (1).mp4


"' https://t.me/USMLEWorldStep1 Show all X
= Item ~9 of 30
Question Id: 13554
■ t7' Mark <:::
Previous
C>
Next
~~
Full Screen
®
Tutorial
il
Lab Values
~-
Notes
....I
Calculator
,
Reverse Color
~
Text Zoom
©
Settings

This patient is presenting with angle-closure glaucoma (ACG) that was precipitated by use of over-the-counter
eye drops. ACG typically occurs in patients with a predisposing narrow chamber angle and develops when the
normal drainage pathway of aqueous humor becomes blocked .

Aqueous humor is produced by the ciliary body and flows from the posterior chamber of the eye through the pupil
into the anterior chamber and out of the eye via the trabecular network. In susceptible individuals, dilation of the
pupil causes the iris to press against the anterior surface of the lens. This limits flow of aqueous humor through the
pupil, causing the iris to bulge forward. The corresponding reduction (closing) of the iridocorneal angle blocks
drainage of aqueous humor through the trabecular meshwork.

Acute ACG may be precipitated by topical and systemic medications that cause pupillary dilation, such as alpha-
adrenergic agonists (eg , naphazoline) and drugs with strong anticholinergic effects (eg, tricyclic
antidepressants, antihistamines). Acute blockage results in a rapid rise in intraocular pressure that typically causes
severe eye pain, conjunctiva! injection, and corneal edema (hazi ness). If not corrected, the elevated pressure
can damage the optic nerve and cause permanent vision impairment.

(Choice B) Topical beta blockers (eg, timolol) are used to treat open-angle glaucoma. They work by both
reducing the production of aqueous humor and increasing its outflow from the trabecular network.

(Choices C and D) Topical mast cell stabilizers (eg, cromolyn, nedocromil) and nonsteroidal anti-inflammatory
drugs (eg, ketorolac) are used in the treatment of allergic conjunctivitis, but they are not associated with ACG.

(Choice E) Prostaglandin analogues such as latanoprost increase aqueous outflow from the trabecular network
and are used in the treatment of open-angle glaucoma .

Educational objective:
Acute angle-closure glaucoma occurs when the space between the cornea and iris is narrowed, limiting the flow of
• h t f th t • h b d •• • th M d. r th t • •·11
Block Time Elapsed: 00:05:52 ~ II;! rvo Cl @ 0
My Notebook Flashcards Feedback Suspend End Block

"' !!I 51549 (1).mp4


"' https://t.me/USMLEWorldStep1 Show all X
= Item ~9 of 30
Question Id: 13554
■ t7' Mark <:::
Previous
C>
Next
~~
Full Screen
®
Tutorial
il
Lab Values
~-
Notes
....I
Calculator
,
Reverse Color
~
Text Zoom
©
Settings

normal drainage pathway of aqueous humor becomes blocked .

Aqueous humor is produced by the ciliary body and flows from the posterior chamber of the eye through the pupil
into the anterior chamber and out of the eye via the trabecular network. In susceptible individuals, dilation of the
pupil causes the iris to press against the anterior surface of the lens. This limits flow of aqueous humor through the
pupil, causing the iris to bulge forward. The corresponding reduction (closing) of the iridocorneal angle blocks
drainage of aqueous humor through the trabecular meshwork.

Acute ACG may be precipitated by topical and systemic medications that cause pupillary dilation, such as alpha-
adrenergic agonists (eg , naphazoline) and drugs with strong anticholinergic effects (eg, tricyclic
antidepressants, antihistamines). Acute blockage results in a rapid rise in intraocular pressure that typically causes
severe eye pain, conjunctiva! injection, and corneal edema (haziness). If not corrected, the elevated pressure
can damage the optic nerve and cause permanent vision impairment.

{Choice B) Topical beta blockers (eg, timolol) are used to treat open-angle glaucoma. They work by both
reducing the production of aqueous humor and increasing its outflow from the trabecular network.

(Choices C and D) Topical mast cell stabilizers (eg, cromolyn, nedocromil) and nonsteroidal anti-inflammatory
drugs (eg, ketorolac) are used in the treatment of allergic conjunctivitis, but they are not associated with ACG.

(Choice E) Prostaglandin analogues such as latanoprost increase aqueous outflow from the trabecular network
and are used in the treatment of open-angle glaucoma .

Educational objective:
Acute angle-closure glaucoma occurs when the space between the cornea and iris is narrowed, limiting the flow of
aqueous humor out of the anterior chamber and raising pressure in the eye. Medications that cause pupillary
dilation (eg , alpha-adrenergic agonists, anticholinergics) can trigger acute angle-closure glaucoma .

References
Block Time Elapsed: 00:05:52 ~ II;! rvo Cl @ 0
My Notebook Flashcards Feedback Suspend End Block

"' !!I 51549 (1).mp4


"' https://t.me/USMLEWorldStep1 Show all X
= ltem_30of30
Question Id: 1363
■ t7' Mark <:::
Previous
C>
Next
~~
Full Screen
®
Tutorial
il
Lab Values

Notes
....I
Calculator
,
Reverse Color
~
Text Zoom
©
Settings

A 64-year-old woman is brought to the clinic by her husband due to a fall. She was walking in her living room when
she tripped over an electrical cord. The patient has had several other ground-level falls in the last 6 months but
has had no significant injuries. She also gave up driving a year ago following a near-miss motor vehicle accident.
Past medical history is notable for hypertension and type 2 diabetes mellitus, for which she takes appropriate
medications. Mental status and neurologic examinations are normal. Ophthalmic examination reveals reduced
vision in her peripheral visual fields bilaterally and an elevated intraocular pressure. After discussion of treatment
options, the patient is initiated on timolol ophthalmic drops. Which of the following structures is the most likely
target of this medication?

0 A . Ciliary epithelium

0 8 . Ciliary muscle

0 C. Lens

0 D. Pupil sphincter

0 E. Trabecular meshwork

Submit

Block Time Elapsed: 00:05:53 ~ II;! rvo Cl @ 0


My Notebook Flashcards Feedback Suspend End Block

"' !!I 51549 (1).mp4


"' https://t.me/USMLEWorldStep1 Show all X
= ltem_30of30
Question Id: 1363
■ t7' Mark <:::
Previous
C>
Next
~~
Full Screen
®
Tutorial
il
Lab Values

Notes
....I
Calculator
,
Reverse Color
~
Text Zoom
©
Settings

A 64-year-old woman is brought to the clinic by her husband due to a fall. She was walking in her living room when
she tripped over an electrical cord. The patient has had several other ground-level falls in the last 6 months but
has had no significant injuries. She also gave up driving a year ago following a near-miss motor vehicle accident.
Past medical history is notable for hypertension and type 2 diabetes mellitus, for which she takes appropriate
medications. Mental status and neurologic examinations are normal. Ophthalmic examination reveals reduced
vision in her peripheral visual fields bilaterally and an elevated intraocular pressure. After discussion of treatment
options, the patient is initiated on timolol ophthalmic drops. Which of the following structures is the most likely
target of this medication?

✓ Q A . Ciliary epithelium (42%)

X ® 8 . Ciliary muscle (27%>)

0 C. Lens (0%)

0 D. Pupil sphincter (7°/o)

0 E. Trabecular meshwork (20%)

Incorrect
Correct answer I 1,1 42% fi"\ 04 secs 2023
~ Answered correctly \..::.JTimeSpent Version
A

Explanation

Glaucoma medications

Block Time Elapsed: 00:05:56 ~ II;! rvo Cl @ 0


My Notebook Flashcards Feedback Suspend End Block

"' !!I 51549 (1).mp4


"' https://t.me/USMLEWorldStep1 Show all X
Glaucoma medications

E Cornea
Trabecular
meshwork

Trabecular outflow

Schlemm
canal

Sciera - - Aqueous humor Inflow


• Beta blockers
• Alpha-2 AR agonists
• CAls

Alpha-2 AR agonists = alpha-2 adrenergic receptor agonists: CAls = carbonic anhydrase inhibttors. C)UWO<ld

©_ Zoom In 0_ Zoom Out C Reset Cf;, New I lf-J Existing (!;I My Notebook

!!I 51549 (1).mp4


https://t.me/USMLEWorldStep1 Show all X
= ltem_30of30
Question Id: 1363
■ t7' Mark <:::
Previous
C>
Next
~~
Full Screen
®
Tutorial
il
Lab Values
~-
Notes
....I
Calculator
,
Reverse Color
~
Text Zoom
©
Settings

This patient with chronic, progressive loss of peripheral vision has typical features of open-angle glaucoma.
Glaucoma is a type of optic neuropathy characterized by atrophy of the optic nerve head. It is more common with
advancing age and is especially common in certain ethnicities (eg, African Americans). Glaucoma is usually
associated with elevated intraocular pressure (IOP) due to increased production or decreased outflow of aqueous
humor. Aqueous humor is produced by the epithelial cells of the ciliary body. It is secreted into the posterior eye
chamber and transferred through the pupil into the anterior eye chamber. The anterior chamber angle (iridocorneal
angle) contains a trabecular meshwork through which the aqueous humor diffuses into Schlemm's canal (scleral
venous sinus) and subsequently into episcleral and conjunctiva! veins.

Diagnostic features of glaucoma include elevated IOP and abnormal visual field testing with decreased peripheral
vision . Funduscopic examination will show an increased cup-to-disc ratio due to loss of ganglion cell axons. The
goal of treatment in glaucoma is to decrease IOP with drugs that either decrease the production of aqueous humor
or increase its outflow. Timolol and other nonselective beta blockers work by diminishing the secretion of aqueous
humor by the ciliary epithelium.

(Choices B, C, D, and E) The intraocular muscles of the eye, including the ciliary muscle and pupillary sphincter
of the iris, receive parasympathetic innervation from cholinergic fibers of the oculomotor nerve (cranial nerve Ill).
Cholinergic agonists (eg, pilocarpine) cause miosis by promoting contraction of the sphincter of the iris. This
causes the anterior chamber angle to become wider and makes the trabecular meshwork more accessible to
outflow of aqueous humor. Cholinergic agonists also promote ciliary muscle contraction , causing the lens to
become more convex.

Educational objective:
Open-angle glaucoma is characterized by progressive loss of peripheral vision from elevated intraocular pressure.
Timolol and other nonselective beta blockers work by diminishing the secretion of aqueous humor by the ciliary
epithelium. Acetazolamide, a carbonic anhydrase inhibitor, also decreases aqueous humor secretion by the ciliary
Block Time Elapsed: 00:05:56 ~ II;! rvo Cl @ 0
My Notebook Flashcards Feedback Suspend End Block

"' !!I 51549 (1).mp4


"' https://t.me/USMLEWorldStep1 Show all X
= ltem_30of30
Question Id: 1363
■ t7' Mark <:::
Previous
C>
Next
~~
Full Screen
®
Tutorial
il
Lab Values
~-
Notes
....I
Calculator
,
Reverse Color
~
Text Zoom
©
Settings

advancing age and is especially common in certain ethnicities (eg, African Americans). Glaucoma is usually
associated w ith elevated intraocular pressure (IOP) due to increased production or decreased outflow of aqueous
humor. Aqueous humor is produced by the epithelial cells of the ciliary body. It is secreted into the posterior eye
chamber and transferred through the pupil into the anterior eye chamber. The anterior chamber angle (iridocorneal
angle) contains a trabecular meshwork through which the aqueous humor diffuses into Schlemm's canal (scleral
venous sinus) and subsequently into episcleral and conjunctiva! veins.

Diagnostic features of glaucoma include elevated IOP and abnormal visual field testing with decreased peripheral
vision. Funduscopic examination will show an increased cup-to-disc ratio due to loss of ganglion cell axons. The
goal of treatment in glaucoma is to decrease IOP with drugs that either decrease the production of aqueous humor
or increase its outflow. Timolol and other nonselective beta blockers work by diminishing the secretion of aqueous
humor by the ciliary epithelium.

(Choices B, C, D, and E) The intraocular muscles of the eye, including the ciliary muscle and pupillary sphincter
of the iris, receive parasympathetic innervation from cholinergic fibers of the oculomotor nerve (cranial nerve Il l).
Cholinergic agonists (eg, pilocarpine) cause miosis by promoting contraction of the sphincter of the iris. This
causes the anterior chamber angle to become wider and makes the trabecular meshwork more accessible to
outflow of aqueous humor. Cholinergic agonists also promote ciliary muscle contraction, causing the lens to
become more convex.

Educational objective:
Open-angle glaucoma is characterized by progressive loss of peripheral vision from elevated intraocular pressure.
Timolol and other nonselective beta blockers work by diminishing the secretion of aqueous humor by the ciliary
epithelium. Acetazolamide, a carbonic anhydrase inhibitor, also decreases aqueous humor secretion by the ciliary
epithelium. Prostaglandin F2a (eg, latanoprost, travoprost) and cholinomimetics (eg, pilocarpine, carbachol)
decrease intraocular pressure by increasing the outflow of aqueous humor.

Block Time Elapsed: 00:05:56 ~ II;! rvo Cl @ 0


My Notebook Flashcards Feedback Suspend End Block

"' !!I 51549 (1).mp4


"' https://t.me/USMLEWorldStep1 Show all X
Glaucoma pathophysiology

Normal eye

This
Gia
adv
ass Optic nerve / :~~-~,;
Retina Normal fluid flow
hu
Glaucoma

angl
ven

Optic nerve
damage Blocked fluid flow
hu

©_ Zoom In 0_ Zoom Out C Reset Cf;, New I lf-J Existi ng (!;I My Notebook

Ch

Block Time Elapsed: 00:05:56 ~ ll;I rvo Cl @ 0


My Notebook Flashcards Feedback Suspend End Block

"' !!I 51549 (1).mp4


"' https://t.me/USMLEWorldStep1 Show all X
Optic disc in glaucoma

Normal Open-angle glaucoma

This
Gia
adv
ass
hu

angl
ven

• Enlarged cup with cup/disc ratio >0.6


• Cup/disc ratio <0.5 • Increase in cup size over time
• Clear disc rim • Thinning of disc rim
• Pale disc (optic nerve atrophy)
hu euwortd

©_ Zoom In 0_ Zoom Out C Reset C:::, New I lf-J Existi ng (!;I My Notebook

Ch

Block Time Elapsed: 00:05:56 ~ II;! rvo Cl @ 0


My Notebook Flashcards Feedback Suspend End Block

"' !!I 51549 (1).mp4


"' https://t.me/USMLEWorldStep1 Show all X

You might also like